Re: What falsifiability tests has computationalism passed?

2018-01-17 Thread David Nyman
On 18 Jan 2018 00:42, "Brent Meeker" <meeke...@verizon.net> wrote:



On 1/17/2018 2:10 PM, David Nyman wrote:

​I agree. I didn't elaborate in the interests of being short, Nevertheless,
the evolutionary 'selection' argument must, necessarily, rest ultimately on
extrinsic or 3p behaviour (again at the appropriate 'substitution' level).
And on physical-reductionist assumptions, all such behaviour must
necessarily be a proxy for 'fundamental physical law'. Hence my movie/TV
analogy was intended to suggest that the existence of a consistent 'mental
agency' implies the 'epistemic selection' of an (at least) equally
consistent and tightly-constrained 'physical' constitution, depending in
turn on the (undoubtedly long and deep) evolution of a more generalised
environment, based on that foundational 'physical lawfulness'.

This would be the highly non-compressible story both of how the TV and its
viewer came to exist 'physically' and how the viewer came to be
experiencing a movie by means of the complex epistemic relation between its
brain and the TV. For brevity, one can condense this into a purely 3p story
of an evolutionary process that comes about in precisely this way, but as
Tallis crucially points out, we must not take our *experiencing* of
'precisely this way' for granted, as experience, or appearance, is not a 3p
category. Hence in a (logical) sense, the epistemic 'selection' of its own
physical *appearance* would have been a necessary supplementary (1p)
assumption even in the case that (assuming comp, counterfactually) mental
agency could have been be shown to supervene uniquely on some
'pre-ordained', non-quantised 3p physics.

David


I don't think this 3p/1p distinction is a clear as you and Bruno assume.
As I tried to make clear in my axioms of CTM, in your view that only what
is really, really fundamental exists one must say that only 1p
experiences/thoughts exist.  As Descarte and Russell have pointed out, all
the "3p" stuff, including the "person" to whom the experience is attributed
are inferences.  So 3p whatever must be emergent or derivative from the 1p
and hence part of the 1p.


Actually
​Brent,speaking personally, it's not so much what I 'think' in any
cut-and-dried sense, ​but rather what I'm progressively trying to clarify -
at least for for myself - in the form of this and other recent threads. On
that basis, I agree that, on pain of incoherence, we are forced to begin
with 1p for what counts as real. Perhaps this is, at least in part, what
you've been getting at when you've said that epistemology precedes
ontology. What remains is some sort of explanation for the *appearance* of
a such a reality; in modern times, such explanations are customarily
expressed in mathematical form. I think I can be explicit about the 1p/3p
distinction in these terms. What I've been trying to be clear about is the
distinctness of 1p as an *epistemic* category, which we can call knowledge,
thought, or consciousness (the words are not terribly important as long as
the categorical distinction is clear). As such, it is always *about*
something and *on behalf of* someone. That someone might be termed a
self-referential 'mental agent' (again, the name isn't of importance),
initially defined in terms of basic notions of information and computation,
in turn expressible by means of the combinatorial elements of what is
essentially offered as a fundamental language or calculus of thought. Then
the *about* is the agent's 'introspection' of its own epistemic states
(which appear to it, in part, as an 'extraspection' of its spatial and
temporal situation). The agent's epistemic states are 1p and by definition
real; what is offered in explanation of those states is 3p and may be
accounted real by reason of *explanatory power* in accounting for the
former. AFAICS they should be counted none the less real for that.

For the agent to possess the necessary 'physics', assuming comp (always
motivated by CTM), the agent is then conceived as participating in
complexes of computation (which as Bruno says are likely to be
correspondingly long and deep) that both track the disposition and
evolution of its own 'physical' constitution and generalised environment,
and simultaneously emulate (3p)  'dispositional attitudes' that correspond
with its (1p) epistemic states. This 'coincidence' of physical
dispositions, with the corresponding 'attitudinal' computations and their
epistemic entailments, might well strike us as remarkably adventitious
(often attracting the dismissive epithet of 'epiphenomenal') since the
entire (3p) 'dispositional' history, when examined at any given
'substitution level', seems fully capable of operating independently of
whatever 1p story it might be taken to imply. This is what makes the
'epistemic reversal' indispensable to the intelligibility of what is being
proposed. Hence my TV/movie analogy in which, in the relevant sense, it
might well be more intelligible to

Re: What falsifiability tests has computationalism passed?

2018-01-14 Thread David Nyman
On 14 January 2018 at 11:48, Bruno Marchal <marc...@ulb.ac.be> wrote:

>
> On 10 Jan 2018, at 20:23, David Nyman <david.ny...@gmail.com> wrote:
>
>
>
> On 10 Jan 2018 13:48, "Bruno Marchal" <marc...@ulb.ac.be> wrote:
>
>
> On 7 Jan 2018, at 12:42, David Nyman <da...@davidnyman.com> wrote:
>
> On 7 January 2018 at 09:52, Bruno Marchal <marc...@ulb.ac.be> wrote:
>
>>
>> On 6 Jan 2018, at 21:09, David Nyman <david.ny...@gmail.com> wrote:
>>
>>
>>
>> On 6 Jan 2018 19:46, "Bruno Marchal" <marc...@ulb.ac.be> wrote:
>>
>>
>> On 5 Jan 2018, at 21:04, David Nyman <david.ny...@gmail.com> wrote:
>>
>>
>>
>> On 5 Jan 2018 19:27, "Bruno Marchal" <marc...@ulb.ac.be> wrote:
>>
>>
>> On 4 Jan 2018, at 21:07, David Nyman <david.ny...@gmail.com> wrote:
>>
>>
>>
>> On 4 Jan 2018 18:16, "Bruno Marchal" <marc...@ulb.ac.be> wrote:
>>
>>
>> On Jan 4, 2018, at 1:22 PM, David Nyman <da...@davidnyman.com> wrote:
>>
>> On 4 January 2018 at 11:55, Bruno Marchal <marc...@ulb.ac.be> wrote:
>>
>>>
>>> > On Jan 3, 2018, at 10:57 PM, Brent Meeker <meeke...@verizon.net>
>>> wrote:
>>> >
>>> >
>>> >
>>> > On 1/3/2018 5:47 AM, Bruno Marchal wrote:
>>> >>
>>> >> On 03 Jan 2018, at 03:39, Brent Meeker wrote:
>>> >>
>>> >>>
>>> >>>
>>> >>> On 1/2/2018 8:07 AM, Bruno Marchal wrote:
>>> >>>> Now, it
>>> >>>> could be that intelligent behavior implies mind, but as you yourself
>>> >>>> argue, we don't know that.
>>> >>>
>>> >>> Isn't this at the crux of the scientific study of the mind? There
>>> seemed to be universal agreement on this list that a philosophical zombie
>>> is impossible.
>>> >>
>>> >>
>>> >> Precisely: that a philosophical zombie is impossible when we assume
>>> Mechanism.
>>> >
>>> > But the consensus here has been that a philosophical zombie is
>>> impossible because it exhibits intelligent behavior.
>>>
>>> Well, I think the consensus here is that computationalism is far more
>>> plausible than non-computationalism.
>>> Computationalism makes zombies non sensical.
>>>
>>>
>>>
>>> >
>>> >> Philosophical zombie remains logical consistent for a non
>>> computationalist theory of mind.
>>> >
>>> > It's logically consistent with a computationalist theory of brain. It
>>> is only inconsistent with a computationalist theory of mind because use
>>> include as an axiom that computation produces mind.  One can as say that
>>> intelligent behavior entails mind as an axiom of physicalism.  Logic is a
>>> very cheap standard for theories to meet.
>>>
>>> At first sight, zombies seems consistent with computationalism, but the
>>> notion of zombies requires the idea that we attribute mind to bodies
>>> (having the right behavior). But with computationalism, mind is never
>>> associated to a body, but only to the person having the infinity of
>>> (similar enough) bodies relative representation in arithmetic. There are no
>>> “real bodies” or “ontological bodies”, so the notion of zombie becomes
>>> senseless. The consciousness is associated with the person, which is never
>>> determined by one body.
>>>
>>
>> ​So in the light of what you say above, does it then follow that the MGA
>> implies (assuming comp) that a physical system does *not* in fact implement
>> a computation in the relevant sense?
>>
>>
>>
>> The physical world has to be able to implement the computation in the
>> relevant (Turing-Church-Post-Kleene CT) sense. You need this for the YD
>> “act of faith.
>>
>> The physical world is a persistent illusion. It has to be enough
>> persistent that you wake up at the hospital with the digital brain.
>>
>>
>>
>> I ask this because you say mind is *never* associated with a body, but
>> mind *is* associated with computation via the epistemic consequences of
>> universality.
>>
>>
>>
>> A (conscious) third person can associate a mind/person to a body that he
>> perceives. It is polite.
>>
>> The body perceived by that third person is itself a construction 

Re: What falsifiability tests has computationalism passed?

2018-01-12 Thread David Nyman
On 12 Jan 2018 00:36, "Brent Meeker" <meeke...@verizon.net> wrote:



On 1/11/2018 4:11 AM, David Nyman wrote:



On 11 Jan 2018 04:02, "Brent Meeker" <meeke...@verizon.net> wrote:



On 1/10/2018 6:56 PM, David Nyman wrote:



On 11 Jan 2018 02:34, "Brent Meeker" <meeke...@verizon.net> wrote:



On 1/10/2018 6:11 PM, David Nyman wrote:

If you read the rest of Tallis's piece you'll see that he criticises the
characterisation of the physical environment as encoding 'information'
independent of interpretation. This objection can be dealt with by the
reversal,


Can it?  Isn't it just *assumed* that the computational relations, or
number relations, encode information?  That was my objection that the MGA
was missing the necessity of an environment for its computation to be
"about".  Bruno has generally agreed with this and said it just means that
the environment (i.e. physics) is part of what is realized in the
computations of the UD.  But notice that this doesn't answer a Tallis like
objection that "computation is nothing like experience" and "information is
nothing like environment".


But the argument implies that the epistemic entailments of computation,


That a computation has epistemic entailments is an assumption that it's a
computation *about* something.  The argument *assumes* that so far as I can
see.


I think you're right in part, to the extent that Theatatus's criterion of
knowledge, which includes the assumption of the (tautological) truth of the
belief, is indeed an explicit axiom.


Aren't you jumping over the question?  If I have a set of diophantine
equations that instantiate a universal computer and they have an integer
that is a solution, that's a computation.  But why is it "about anything"?
which parts of this process constitute knowledge or belief?  What
proposition is true?


I thought I was answering the question. Theatatus's criterion is modelled
as the conjunction of believing in p and the axiom that p is indeed the
case *in some relevant sense*. IOW true, justified belief. As to what is a
proposition and what it may be about, since logic is fully emulable in
​'universal'
computation,
​in the limit ​
it might be about most anything whatsoever I suppose.
​That apart,

​as Bruce remarked, if we ​
apply a reductionist approach the criticism can be levelled that, both its
very status as a proposition in the first place and the interpretation of
what it is about, are
​by implication *​
external
​*​
to the computation itself.
​But s
ince ex hypothesi this cannot be the case without infinite regress, we are
confronted with the idea that the truth (or 'epistemic entailment')
putatively asserted by the proposition
​*​
supplies its own interpretation
​*​
.

If this is not very intuitive at the level of the toy model, it might be
​a little ​
more so if we
​try to ​
generalise the idea in
​something like ​
the way I suggested. The physical constitution and activity of a brain, in
this way of thinking, would
​then ​
be construed as the manifestation in appearance of some complex of
computations that simultaneously tracks, both the requisite physical
dispositions relating the brain and its wider environment, and the
propositional attitudes of a 'mental agent'.
​(​
By the way, I would be sympathetic to Dennett's idea that this would not
​be expected to ​
play out in the manner of any simplistic notion of a 'Cartesian theatre'.
​)​

Then a propositional attitude attributable to the mental agent might be "I
see an apple" or "I am in intense agony". What the proposition is 'about'
is now
​in terms of
the agent's apprehension of its own state, implying an epistemic truth
​*​
about itself
​*, or perhaps more broadly, about the boundaries of its own physical and
temporal situation​
, one that i
​s ​
​by the same token
not directly communicable. In either case this attitude would necessarily
​have to be
 expressed both physically and epistemically
​; th
e physical express
​ion​
in behaviour, both gross and subtle (e.g. neurocognition); and the
(tautological
​ly truthful​
)
​entailment
 in the fact that the agent in question
​*​
does indeed see an apple or is indeed in intense agony
​*​
, the latter supplying the
​ otherwise​
missing 'interpretation' of the former.
​

As I suggested to Bruce, if one considers the converse, what would it imply
if I believed that, though you were not in any sense lying, your assertion
of such things was nonetheless false? That it only 'seems as if' they are
true, or that their truth is an 'illusion'? By the way, this is apparently
what the likes of Patricia Churchland urge us to take seriously - that we
are nothing other than zombie mechanisms, whose putative mentalistic
assertions are supernumerary to any fundamental understanding of reality.
At the same time o
ne must be careful not to be misled into the idea that
​'​
truth
​'​
here implies that the agent

Re: What falsifiability tests has computationalism passed?

2018-01-11 Thread David Nyman
On 11 Jan 2018 04:02, "Brent Meeker" <meeke...@verizon.net> wrote:



On 1/10/2018 6:56 PM, David Nyman wrote:



On 11 Jan 2018 02:34, "Brent Meeker" <meeke...@verizon.net> wrote:



On 1/10/2018 6:11 PM, David Nyman wrote:

If you read the rest of Tallis's piece you'll see that he criticises the
characterisation of the physical environment as encoding 'information'
independent of interpretation. This objection can be dealt with by the
reversal,


Can it?  Isn't it just *assumed* that the computational relations, or
number relations, encode information?  That was my objection that the MGA
was missing the necessity of an environment for its computation to be
"about".  Bruno has generally agreed with this and said it just means that
the environment (i.e. physics) is part of what is realized in the
computations of the UD.  But notice that this doesn't answer a Tallis like
objection that "computation is nothing like experience" and "information is
nothing like environment".


But the argument implies that the epistemic entailments of computation,


That a computation has epistemic entailments is an assumption that it's a
computation *about* something.  The argument *assumes* that so far as I can
see.


I think you're right in part, to the extent that Theatatus's criterion of
knowledge, which includes the assumption of the (tautological) truth of the
belief, is indeed an explicit axiom. But as is the case with all
hypotheses, the burden is then to persuade that adopting this axiom is a
reasonable step towards shedding some light on the problem we've set out to
address. And one of the defining characteristics of beliefs of the
requisite sort is indeed their indubitably, at least as a first
approximation. IOW, each sentient agent, willy-nilly, is irrevocably bound
(the 'bet' on a reality) to the primary veridicality of phenomena to which
it is thereafter both epistemically and procedurally committed.

And again, the point of studying the self-referential logics in this regard
is to provide the kernel of a model of *aboutness* that could indeed be
understood as  'reaching out', in Tallis's sense, towards such a world. It
constructs, as it were, a space for the relation of the agent and its
phenomenal world that could begin to be seen as possessing the necessary
epistemic and procedural dimensionality, which is arguably what is lacking
in the construction of a 'world' in strictly third personal terms.



not its formal elements, attain this criterion whilst being at the same
time, and unavoidably, directly incommunicable.


The things that are incommunicable, as I understand it, are the things that
are true (given the axiomatic system) but unprovable.  In other words, a
proof is what it communicable.


Yes, and hence, in a sense admittedly highly generalised from the toy
model, proof constitutes the procedural or actionable polarity in the
epistemic construction of a 'world'. It is what is available for public
inspection. But at the same time a system of proofs or theorems of
sufficient power implies further truths that are not formally describable.
In essence, again in a highly generalised extrapolation, this forms the
analogy with the epistemic situation of a sentient agent in relation to its
phenomenal world.

It is publicly and procedurally committed to a system of proofs or beliefs
which can never exceed the reliability of relatively confirmed conjectures.
However the epistemic consequences of such beliefs present themselves in
the form of categorically distinct phenomena that elude this mode of direct
communication. So, in another reversal, the substantive 'world ' now lies
at the interior or phenomenal pole of this relation and its procedural or
processual construction at the 'publicly inferrable' or externalisable one.




This category therefore subsumes that of physical phenomenology. So in
principle this is capable of answering the objection, at least in the limit
of what can be explicitly characterised.

  I find these "nothing like" arguments facile.


I know you do. And I've tried to present a less 'facile' view of the issues
as best I can.


I appreciate the effort.


Thanks.

David



Brent


But there I fear our views of the matter diverge, probably irreconcilably.

David



Brent
-- 
You received this message because you are subscribed to the Google Groups
"Everything List" group.
To unsubscribe from this group and stop receiving emails from it, send an
email to everything-list+unsubscr...@googlegroups.com.
To post to this group, send email to everything-list@googlegroups.com.
Visit this group at https://groups.google.com/group/everything-list.
For more options, visit https://groups.google.com/d/optout.


-- 
You received this message because you are subscribed to the Google Groups
"Everything List" group.
To unsubscribe from this group and stop receiving emails from it, send an
email to everyt

Re: What falsifiability tests has computationalism passed?

2018-01-10 Thread David Nyman
On 11 Jan 2018 02:34, "Brent Meeker" <meeke...@verizon.net> wrote:



On 1/10/2018 6:11 PM, David Nyman wrote:

If you read the rest of Tallis's piece you'll see that he criticises the
characterisation of the physical environment as encoding 'information'
independent of interpretation. This objection can be dealt with by the
reversal,


Can it?  Isn't it just *assumed* that the computational relations, or
number relations, encode information?  That was my objection that the MGA
was missing the necessity of an environment for its computation to be
"about".  Bruno has generally agreed with this and said it just means that
the environment (i.e. physics) is part of what is realized in the
computations of the UD.  But notice that this doesn't answer a Tallis like
objection that "computation is nothing like experience" and "information is
nothing like environment".


But the argument implies that the epistemic entailments of computation, not
its formal elements, attain this criterion whilst being at the same time,
and unavoidably, directly incommunicable. This category therefore subsumes
that of physical phenomenology. So in principle this is capable of
answering the objection, at least in the limit of what can be explicitly
characterised.

  I find these "nothing like" arguments facile.


I know you do. And I've tried to present a less 'facile' view of the issues
as best I can. But there I fear our views of the matter diverge, probably
irreconcilably.

David



Brent

-- 
You received this message because you are subscribed to the Google Groups
"Everything List" group.
To unsubscribe from this group and stop receiving emails from it, send an
email to everything-list+unsubscr...@googlegroups.com.
To post to this group, send email to everything-list@googlegroups.com.
Visit this group at https://groups.google.com/group/everything-list.
For more options, visit https://groups.google.com/d/optout.

-- 
You received this message because you are subscribed to the Google Groups 
"Everything List" group.
To unsubscribe from this group and stop receiving emails from it, send an email 
to everything-list+unsubscr...@googlegroups.com.
To post to this group, send email to everything-list@googlegroups.com.
Visit this group at https://groups.google.com/group/everything-list.
For more options, visit https://groups.google.com/d/optout.


Re: What falsifiability tests has computationalism passed?

2018-01-10 Thread David Nyman
On 10 January 2018 at 23:49, Bruce Kellett <bhkell...@optusnet.com.au>
wrote:

> On 11/01/2018 9:09 am, Brent Meeker wrote:
>
> On 1/10/2018 11:23 AM, David Nyman wrote:
>
> Searle makes his position even more vulnerable by arguing that not only
> are neural activity and the experience of perception the same but that the
> former *causes *the latter just as water is “caused” by H2O. This is
> desperate stuff: one could hardly expect some thing A to cause some thing B
> with which it is identical, because nothing can cause itself. In any event,
> the bottom line is that the molecules of H2O and the wet stuff that is
> water are two *appearances *of the same thing — two conscious takes on
> the same stuff. They cannot be analogous to, respectively, that which
> supposedly *causes* conscious experiences (neural impulses) and conscious
> experiences *themselves*.​"
> Here's a link to the original piece:
>
>
> <https://www.thenewatlantis.com/publications/what-neuroscience-cannot-tell-us-about-ourselves>
> https://www.thenewatlantis.com/publications/what-neuroscienc
> e-cannot-tell-us-about-ourselves
>
>
> I understand those criticisms of Searle and they may be right.  But note
> that arithmetic and computation are nothing like experience either and all
> the same criticisms apply to CTM; something that goes unchallenged on this
> list because CTM is always taken as a given and Bruno says, "We must assume
> something to begin our theorizing."  But Searle would reject CTM (and has)
> for exactly the same criticisms directed against him above.
>
>
> I think Tallis's argument against Searle is entirely specious. Searle
> would appear to be arguing that the properties of H2O molecules, such as
> the large dipole moment, etc, cause the observed bulk properties of water.
> It is hard to fault such an argument -- what else could possibly lie behind
> the bulk properties of water other than the properties of its constituent
> molecules? By analogy, then, the bulk properties of a brain, such as
> consciousness, thought, memory, and so on
>

​None of these can be claimed as 'bulk properties of a brain', except by a
brute and unargued-for a posteriori assumption (or identity thesis). On the
contrary, they all fall properly into the category of phenomena or
appearance, to employ Tallis's taxonomy. I think you have managed entirely
to miss the point of his argument.​

arise from the properties of the individual neurons and other structures
> that make up the physical brain
>

​The only necessary entailment is that what arises from the properties of
the individual neurons and other structures are the bulk properties of a
brain - i.e. neurocognition and so forth.​ So far, so circular.
​

> -- mind supervenes on the physical brain.
>

​Yes - in the sense of covariance. No - in any further, explicitly
argued-for sense​, except by brute extrapolation or identity.


> As you say, Tallis's argument can be raised against the CTM account --
> after all, consciousness is not the same thing as a computation.
>

​I don't believe so. The point of Bruno's hypothesis is to associate
consciousness with specific, first-person *epistemic entailments* of
computation​, not directly with computation in any purely third-person
sense. Now, you may criticise this by saying that it is likewise
unargued-for and if I'm honest this is something that troubled me for some
time about CTM in this formulation. Hence my characterisation of the
reversal as a complete starting-over with formal elements in terms of which
both information and computation can, in principle, be directly encoded or
described. A 'language of thought', if you will.

If you read the rest of Tallis's piece you'll see that he criticises the
characterisation of the physical environment as encoding 'information'
independent of interpretation. This objection can be dealt with by the
reversal, the consequence being that physics itself comes to be understood
emergently as a complex and tightly constrained set of epistemic phenomena
or appearances. But we are then more justified in claiming that such
appearances can - indeed ubiquitously - be understood as 'encoding'
information and computation, including that associated (albeit indirectly)
with mind.

But to make the final step to mind, we need to mark 'something' out that,
whilst being inextricably 'entangled' with computation and its physical
manifestation, can at the same time be understood to be categorically
distinct from either. The suggestion is then that this something - this
species of categorical distinction, if you like - is to be found at the
juncture of an ontology - in this case the elements and processes of
computation - with its justified or 'truthful' epistemic derivatives, or
'interpretation'. IOW, the idea is that as (the possessor of) a mind, I
apprehend, as

Re: What falsifiability tests has computationalism passed?

2018-01-10 Thread David Nyman
On 10 Jan 2018 13:48, "Bruno Marchal" <marc...@ulb.ac.be> wrote:


On 7 Jan 2018, at 12:42, David Nyman <da...@davidnyman.com> wrote:

On 7 January 2018 at 09:52, Bruno Marchal <marc...@ulb.ac.be> wrote:

>
> On 6 Jan 2018, at 21:09, David Nyman <david.ny...@gmail.com> wrote:
>
>
>
> On 6 Jan 2018 19:46, "Bruno Marchal" <marc...@ulb.ac.be> wrote:
>
>
> On 5 Jan 2018, at 21:04, David Nyman <david.ny...@gmail.com> wrote:
>
>
>
> On 5 Jan 2018 19:27, "Bruno Marchal" <marc...@ulb.ac.be> wrote:
>
>
> On 4 Jan 2018, at 21:07, David Nyman <david.ny...@gmail.com> wrote:
>
>
>
> On 4 Jan 2018 18:16, "Bruno Marchal" <marc...@ulb.ac.be> wrote:
>
>
> On Jan 4, 2018, at 1:22 PM, David Nyman <da...@davidnyman.com> wrote:
>
> On 4 January 2018 at 11:55, Bruno Marchal <marc...@ulb.ac.be> wrote:
>
>>
>> > On Jan 3, 2018, at 10:57 PM, Brent Meeker <meeke...@verizon.net> wrote:
>> >
>> >
>> >
>> > On 1/3/2018 5:47 AM, Bruno Marchal wrote:
>> >>
>> >> On 03 Jan 2018, at 03:39, Brent Meeker wrote:
>> >>
>> >>>
>> >>>
>> >>> On 1/2/2018 8:07 AM, Bruno Marchal wrote:
>> >>>> Now, it
>> >>>> could be that intelligent behavior implies mind, but as you yourself
>> >>>> argue, we don't know that.
>> >>>
>> >>> Isn't this at the crux of the scientific study of the mind? There
>> seemed to be universal agreement on this list that a philosophical zombie
>> is impossible.
>> >>
>> >>
>> >> Precisely: that a philosophical zombie is impossible when we assume
>> Mechanism.
>> >
>> > But the consensus here has been that a philosophical zombie is
>> impossible because it exhibits intelligent behavior.
>>
>> Well, I think the consensus here is that computationalism is far more
>> plausible than non-computationalism.
>> Computationalism makes zombies non sensical.
>>
>>
>>
>> >
>> >> Philosophical zombie remains logical consistent for a non
>> computationalist theory of mind.
>> >
>> > It's logically consistent with a computationalist theory of brain. It
>> is only inconsistent with a computationalist theory of mind because use
>> include as an axiom that computation produces mind.  One can as say that
>> intelligent behavior entails mind as an axiom of physicalism.  Logic is a
>> very cheap standard for theories to meet.
>>
>> At first sight, zombies seems consistent with computationalism, but the
>> notion of zombies requires the idea that we attribute mind to bodies
>> (having the right behavior). But with computationalism, mind is never
>> associated to a body, but only to the person having the infinity of
>> (similar enough) bodies relative representation in arithmetic. There are no
>> “real bodies” or “ontological bodies”, so the notion of zombie becomes
>> senseless. The consciousness is associated with the person, which is never
>> determined by one body.
>>
>
> ​So in the light of what you say above, does it then follow that the MGA
> implies (assuming comp) that a physical system does *not* in fact implement
> a computation in the relevant sense?
>
>
>
> The physical world has to be able to implement the computation in the
> relevant (Turing-Church-Post-Kleene CT) sense. You need this for the YD
> “act of faith.
>
> The physical world is a persistent illusion. It has to be enough
> persistent that you wake up at the hospital with the digital brain.
>
>
>
> I ask this because you say mind is *never* associated with a body, but
> mind *is* associated with computation via the epistemic consequences of
> universality.
>
>
>
> A (conscious) third person can associate a mind/person to a body that he
> perceives. It is polite.
>
> The body perceived by that third person is itself a construction of its
> own mind, and with computationalism (but also with QM), we know that such a
> body is an (evolving) map of where, and in which states, we could find, sy,
> the electron and proton of that body, and such snapshot is only a
> computational state among infinitely many others which would works as well,
> with respect to the relevant computations which brought its conscious state.
> Now, the conscious first person cannot associate itself to any particular
> body or computation.
>
> Careful: sometimes I say that a machine can think, or maybe (I usually
> avoid) that a computation can think or be

Re: What falsifiability tests has computationalism passed?

2018-01-07 Thread David Nyman
On 7 January 2018 at 09:52, Bruno Marchal <marc...@ulb.ac.be> wrote:

>
> On 6 Jan 2018, at 21:09, David Nyman <david.ny...@gmail.com> wrote:
>
>
>
> On 6 Jan 2018 19:46, "Bruno Marchal" <marc...@ulb.ac.be> wrote:
>
>
> On 5 Jan 2018, at 21:04, David Nyman <david.ny...@gmail.com> wrote:
>
>
>
> On 5 Jan 2018 19:27, "Bruno Marchal" <marc...@ulb.ac.be> wrote:
>
>
> On 4 Jan 2018, at 21:07, David Nyman <david.ny...@gmail.com> wrote:
>
>
>
> On 4 Jan 2018 18:16, "Bruno Marchal" <marc...@ulb.ac.be> wrote:
>
>
> On Jan 4, 2018, at 1:22 PM, David Nyman <da...@davidnyman.com> wrote:
>
> On 4 January 2018 at 11:55, Bruno Marchal <marc...@ulb.ac.be> wrote:
>
>>
>> > On Jan 3, 2018, at 10:57 PM, Brent Meeker <meeke...@verizon.net> wrote:
>> >
>> >
>> >
>> > On 1/3/2018 5:47 AM, Bruno Marchal wrote:
>> >>
>> >> On 03 Jan 2018, at 03:39, Brent Meeker wrote:
>> >>
>> >>>
>> >>>
>> >>> On 1/2/2018 8:07 AM, Bruno Marchal wrote:
>> >>>> Now, it
>> >>>> could be that intelligent behavior implies mind, but as you yourself
>> >>>> argue, we don't know that.
>> >>>
>> >>> Isn't this at the crux of the scientific study of the mind? There
>> seemed to be universal agreement on this list that a philosophical zombie
>> is impossible.
>> >>
>> >>
>> >> Precisely: that a philosophical zombie is impossible when we assume
>> Mechanism.
>> >
>> > But the consensus here has been that a philosophical zombie is
>> impossible because it exhibits intelligent behavior.
>>
>> Well, I think the consensus here is that computationalism is far more
>> plausible than non-computationalism.
>> Computationalism makes zombies non sensical.
>>
>>
>>
>> >
>> >> Philosophical zombie remains logical consistent for a non
>> computationalist theory of mind.
>> >
>> > It's logically consistent with a computationalist theory of brain. It
>> is only inconsistent with a computationalist theory of mind because use
>> include as an axiom that computation produces mind.  One can as say that
>> intelligent behavior entails mind as an axiom of physicalism.  Logic is a
>> very cheap standard for theories to meet.
>>
>> At first sight, zombies seems consistent with computationalism, but the
>> notion of zombies requires the idea that we attribute mind to bodies
>> (having the right behavior). But with computationalism, mind is never
>> associated to a body, but only to the person having the infinity of
>> (similar enough) bodies relative representation in arithmetic. There are no
>> “real bodies” or “ontological bodies”, so the notion of zombie becomes
>> senseless. The consciousness is associated with the person, which is never
>> determined by one body.
>>
>
> ​So in the light of what you say above, does it then follow that the MGA
> implies (assuming comp) that a physical system does *not* in fact implement
> a computation in the relevant sense?
>
>
>
> The physical world has to be able to implement the computation in the
> relevant (Turing-Church-Post-Kleene CT) sense. You need this for the YD
> “act of faith.
>
> The physical world is a persistent illusion. It has to be enough
> persistent that you wake up at the hospital with the digital brain.
>
>
>
> I ask this because you say mind is *never* associated with a body, but
> mind *is* associated with computation via the epistemic consequences of
> universality.
>
>
>
> A (conscious) third person can associate a mind/person to a body that he
> perceives. It is polite.
>
> The body perceived by that third person is itself a construction of its
> own mind, and with computationalism (but also with QM), we know that such a
> body is an (evolving) map of where, and in which states, we could find, sy,
> the electron and proton of that body, and such snapshot is only a
> computational state among infinitely many others which would works as well,
> with respect to the relevant computations which brought its conscious state.
> Now, the conscious first person cannot associate itself to any particular
> body or computation.
>
> Careful: sometimes I say that a machine can think, or maybe (I usually
> avoid) that a computation can think or be conscious. It always mean,
> respectively, that a machine can make a person capable of manifesting
> itself relatively to you. But the machine and the b

Re: What falsifiability tests has computationalism passed?

2018-01-06 Thread David Nyman
On 6 Jan 2018 19:46, "Bruno Marchal" <marc...@ulb.ac.be> wrote:


On 5 Jan 2018, at 21:04, David Nyman <david.ny...@gmail.com> wrote:



On 5 Jan 2018 19:27, "Bruno Marchal" <marc...@ulb.ac.be> wrote:


On 4 Jan 2018, at 21:07, David Nyman <david.ny...@gmail.com> wrote:



On 4 Jan 2018 18:16, "Bruno Marchal" <marc...@ulb.ac.be> wrote:


On Jan 4, 2018, at 1:22 PM, David Nyman <da...@davidnyman.com> wrote:

On 4 January 2018 at 11:55, Bruno Marchal <marc...@ulb.ac.be> wrote:

>
> > On Jan 3, 2018, at 10:57 PM, Brent Meeker <meeke...@verizon.net> wrote:
> >
> >
> >
> > On 1/3/2018 5:47 AM, Bruno Marchal wrote:
> >>
> >> On 03 Jan 2018, at 03:39, Brent Meeker wrote:
> >>
> >>>
> >>>
> >>> On 1/2/2018 8:07 AM, Bruno Marchal wrote:
> >>>> Now, it
> >>>> could be that intelligent behavior implies mind, but as you yourself
> >>>> argue, we don't know that.
> >>>
> >>> Isn't this at the crux of the scientific study of the mind? There
> seemed to be universal agreement on this list that a philosophical zombie
> is impossible.
> >>
> >>
> >> Precisely: that a philosophical zombie is impossible when we assume
> Mechanism.
> >
> > But the consensus here has been that a philosophical zombie is
> impossible because it exhibits intelligent behavior.
>
> Well, I think the consensus here is that computationalism is far more
> plausible than non-computationalism.
> Computationalism makes zombies non sensical.
>
>
>
> >
> >> Philosophical zombie remains logical consistent for a non
> computationalist theory of mind.
> >
> > It's logically consistent with a computationalist theory of brain. It is
> only inconsistent with a computationalist theory of mind because use
> include as an axiom that computation produces mind.  One can as say that
> intelligent behavior entails mind as an axiom of physicalism.  Logic is a
> very cheap standard for theories to meet.
>
> At first sight, zombies seems consistent with computationalism, but the
> notion of zombies requires the idea that we attribute mind to bodies
> (having the right behavior). But with computationalism, mind is never
> associated to a body, but only to the person having the infinity of
> (similar enough) bodies relative representation in arithmetic. There are no
> “real bodies” or “ontological bodies”, so the notion of zombie becomes
> senseless. The consciousness is associated with the person, which is never
> determined by one body.
>

​So in the light of what you say above, does it then follow that the MGA
implies (assuming comp) that a physical system does *not* in fact implement
a computation in the relevant sense?



The physical world has to be able to implement the computation in the
relevant (Turing-Church-Post-Kleene CT) sense. You need this for the YD
“act of faith.

The physical world is a persistent illusion. It has to be enough persistent
that you wake up at the hospital with the digital brain.



I ask this because you say mind is *never* associated with a body, but mind
*is* associated with computation via the epistemic consequences of
universality.



A (conscious) third person can associate a mind/person to a body that he
perceives. It is polite.

The body perceived by that third person is itself a construction of its own
mind, and with computationalism (but also with QM), we know that such a
body is an (evolving) map of where, and in which states, we could find, sy,
the electron and proton of that body, and such snapshot is only a
computational state among infinitely many others which would works as well,
with respect to the relevant computations which brought its conscious state.
Now, the conscious first person cannot associate itself to any particular
body or computation.

Careful: sometimes I say that a machine can think, or maybe (I usually
avoid) that a computation can think or be conscious. It always mean,
respectively, that a machine can make a person capable of manifesting
itself relatively to you. But the machine and the body are local relative
representation.

A machine cannot think, and a computation (which is the (arithmetical)
dynamic 3p view of the sequence of the relative static machine/state)
cannot think. Only a (first) person can think, and to use that thinking
with respect to another person, a machine is handy, like brain or a
physical computer.

The person is in heaven (arithmetical truth) and on earth (sigma_1
arithmetical truth), simultaneously. But this belongs to G*, and I should
stay mute, or insist that we are in the “after-act-of-faith” position of
the one betting that comp is true, and … assuming comp is true. It is
subtle to talk on those th

Re: Fermi Paradox defined and solved (in 15 minutes)

2018-01-06 Thread David Nyman
On 4 Jan 2018 20:16, "Lawrence Crowell" 
wrote:

On Thursday, January 4, 2018 at 1:44:26 PM UTC-6, agrays...@gmail.com wrote:
>
>
>
> On Thursday, January 4, 2018 at 8:11:28 AM UTC-7, Lawrence Crowell wrote:
>>
>> On Thursday, January 4, 2018 at 8:26:33 AM UTC-6, agrays...@gmail.com
>> wrote:
>>>
>>>
>>> Insofar as you proceed from ignorance by refusing to view a short video,
>>> you have the mistaken impression that those witnesses find some religious
>>> solace in their belief of an alien visitation. Anyway, who cares what some
>>> religious zealots think about UFO's. I am merely laying out a case for the
>>> existence of alien visitations. AG
>>>
>>
>> I had sometime back a person call me ignorant for not wanting to watch a
>> video on ancient alien astronauts --- Eric von Daniken rubbish. I more
>> recently had somebody call me foolish for wishing to remain ignorant of the
>> good news of Jesus etc. So maybe ignorance is a relative term. I am
>> ignorant of what it is like to experience heroin, and I prefer to keep it
>> that way. I am sorry, but I do not have a great interest in eye witness
>> testimony over obscure events 70 years ago.
>>
>> LC
>>
>
> Imagination challenged. If the crash occurred, it's the tip of the
> iceberg; not to say numerous crashes but an on-going presence, which is
> what the project manager  essentially concluded in his comments. Forget it.
> Enjoy your calculations of low densities. And Brent and Clark can enjoy
> their erroneous statements about eye-witness reports. They must know that
> context is important in assessing the value of eye witness reports, but
> refuse to acknowledge same. AG
>

This  is a pretty imaginative
take on the subject.


Blimey! I turned into a Scotsman myself almost 70 years ago. I always
wondered why.

David


LC

 https://www.youtube.com/watch?v=sVbb6pZLfzU

-- 
You received this message because you are subscribed to the Google Groups
"Everything List" group.
To unsubscribe from this group and stop receiving emails from it, send an
email to everything-list+unsubscr...@googlegroups.com.
To post to this group, send email to everything-list@googlegroups.com.
Visit this group at https://groups.google.com/group/everything-list.
For more options, visit https://groups.google.com/d/optout.

-- 
You received this message because you are subscribed to the Google Groups 
"Everything List" group.
To unsubscribe from this group and stop receiving emails from it, send an email 
to everything-list+unsubscr...@googlegroups.com.
To post to this group, send email to everything-list@googlegroups.com.
Visit this group at https://groups.google.com/group/everything-list.
For more options, visit https://groups.google.com/d/optout.


Re: What falsifiability tests has computationalism passed?

2018-01-05 Thread David Nyman
On 5 Jan 2018 23:42, "Brent Meeker" <meeke...@verizon.net> wrote:



On 1/5/2018 3:02 PM, David Nyman wrote:



On 5 January 2018 at 22:52, Brent Meeker <meeke...@verizon.net> wrote:

>
>
> On 1/5/2018 2:17 PM, David Nyman wrote:
>
> To take a more realistic example.
>>
>
> ​I do so love your appropriation of ​the terms 'real' and 'realistic' to
> your own theories.
>
>
I'll take that as rhetorical snark.


Well it may have been snarky, but it wasn't meant rhetorically. Realism is
one of the key things at issue, so it isn't playing fair to appropriate it
to your own theories as by right.




> I think a Mars Rover reporting its battery is low is more realistic than
> it seeing an apple in the most common sense of "realistic".
>

A Mars Rover reporting that its battery is low is roughly analogous to your
judgement that I might look like I could do with a good meal.


No, I'm using "reporting" in the common sense that the rover sent a message
in Engligsh, "My battery is low".  That's a perception by the rover, just
as "I see an apple." is a report of an perception.


You've just equated the two different things that are at issue. If you had
said a *report* of a perception by the rover, that might be analogous to my
reporting or judging that "I see an apple". The question in the case of the
rover would then be if it were the type of 'perception' that both entailed,
and is capable of referring to, a phenomenal counterpart. Does the rover
feel hungry or low in energy? In what ways is it aware of this? If some
future evolution of the rover were capable of truthfully communicating
reports of its phenomenal states, such states being entailments of internal
judgments (as distinct from some superadded mimicry, as with a marionette)
then I would have little principled reason to disbelieve it.




'Reporting' in this sense is the more or less straightforward 'reading' by
you of some state, in this case of charge. It has no semantic content
independent of that fact, any more than my 'looking hungry'.


And "seeing an apple" is different how (aside from the fact there aren't
apples on Mars?


Ah. Assuming you are serious, at this point I think the conversation has to
terminate.

David



Brent



David
 ​

>
>
> Brent
> --
> You received this message because you are subscribed to the Google Groups
> "Everything List" group.
> To unsubscribe from this group and stop receiving emails from it, send an
> email to everything-list+unsubscr...@googlegroups.com.
> To post to this group, send email to everything-list@googlegroups.com.
> Visit this group at https://groups.google.com/group/everything-list.
> For more options, visit https://groups.google.com/d/optout.
>

-- 
You received this message because you are subscribed to the Google Groups
"Everything List" group.
To unsubscribe from this group and stop receiving emails from it, send an
email to everything-list+unsubscr...@googlegroups.com.
To post to this group, send email to everything-list@googlegroups.com.
Visit this group at https://groups.google.com/group/everything-list.
For more options, visit https://groups.google.com/d/optout.


-- 
You received this message because you are subscribed to the Google Groups
"Everything List" group.
To unsubscribe from this group and stop receiving emails from it, send an
email to everything-list+unsubscr...@googlegroups.com.
To post to this group, send email to everything-list@googlegroups.com.
Visit this group at https://groups.google.com/group/everything-list.
For more options, visit https://groups.google.com/d/optout.

-- 
You received this message because you are subscribed to the Google Groups 
"Everything List" group.
To unsubscribe from this group and stop receiving emails from it, send an email 
to everything-list+unsubscr...@googlegroups.com.
To post to this group, send email to everything-list@googlegroups.com.
Visit this group at https://groups.google.com/group/everything-list.
For more options, visit https://groups.google.com/d/optout.


Re: What falsifiability tests has computationalism passed?

2018-01-05 Thread David Nyman
On 5 January 2018 at 22:52, Brent Meeker <meeke...@verizon.net> wrote:

>
>
> On 1/5/2018 2:17 PM, David Nyman wrote:
>
> To take a more realistic example.
>>
>
> ​I do so love your appropriation of ​the terms 'real' and 'realistic' to
> your own theories.
>
>
> I think a Mars Rover reporting its battery is low is more realistic than
> it seeing an apple in the most common sense of "realistic".
>

A Mars Rover reporting that its battery is low is roughly analogous to your
judgement that I might look like I could do with a good meal. 'Reporting'
in this sense is the more or less straightforward 'reading' by you of some
state, in this case of charge. It has no semantic content independent of
that fact, any more than my 'looking hungry'.

David
 ​

>
>
> Brent
>
> --
> You received this message because you are subscribed to the Google Groups
> "Everything List" group.
> To unsubscribe from this group and stop receiving emails from it, send an
> email to everything-list+unsubscr...@googlegroups.com.
> To post to this group, send email to everything-list@googlegroups.com.
> Visit this group at https://groups.google.com/group/everything-list.
> For more options, visit https://groups.google.com/d/optout.
>

-- 
You received this message because you are subscribed to the Google Groups 
"Everything List" group.
To unsubscribe from this group and stop receiving emails from it, send an email 
to everything-list+unsubscr...@googlegroups.com.
To post to this group, send email to everything-list@googlegroups.com.
Visit this group at https://groups.google.com/group/everything-list.
For more options, visit https://groups.google.com/d/optout.


Re: What falsifiability tests has computationalism passed?

2018-01-05 Thread David Nyman
On 5 January 2018 at 20:41, Brent Meeker <meeke...@verizon.net> wrote:

>
>
> On 1/5/2018 4:00 AM, David Nyman wrote:
>
>
>
> On 5 Jan 2018 03:22, "Bruce Kellett" <bhkell...@optusnet.com.au> wrote:
>
> On 4/01/2018 11:59 pm, Bruno Marchal wrote:
>
>> On Jan 4, 2018, at 12:50 PM, Bruce Kellett <bhkell...@optusnet.com.au>
>>> wrote:
>>>
>>> On 4/01/2018 12:30 am, Bruno Marchal wrote:
>>>
>>>> On 29 Dec 2017, at 01:29, Bruce Kellett wrote:
>>>>
>>>>> On 29/12/2017 10:14 am, Russell Standish wrote:
>>>>>
>>>>>> This is computationalism - the idea that our human consciousness _is_
>>>>>> a computation (and nothing but a computation).
>>>>>>
>>>>> What distinguishes a conscious computation within the class of all
>>>>> computations? After all, not all computations are conscious.
>>>>>
>>>> Universality seems enough.
>>>>
>>> What is a universal computation? From what you say below, universality
>>> appears to be a property of a machine, not of a computation.
>>>
>> OK, universality is an attribute of a machine, relatively to some
>> universal machinery, like arithmetic or physics.
>>
>>
>> But just universality gives rise only to a highly non standard,
>>>> dissociative, form of consciousness. It might correspond to the cosmic
>>>> consciousness alluded by people living highly altered state of
>>>> consciousness.
>>>>
>>>> You need Löbianity to get *self-consciousness*, or reflexive
>>>> consciousness. A machine is Löbian when its universality is knowable by it.
>>>> Equivalently, when the machine is universal and can prove its own "Löb's
>>>> formula". []([]p -> p) -> []p. Note that the second incompleteness theorem
>>>> is the particular case with p = f (f = "0≠1").
>>>>
>>> Löbanity is a property of the machine, not of the computation.
>>>
>> Yes. The same. I was talking about machine or about the person supported
>> by those machine. No machine (as conceived as a code, number, physical
>> object) can ever be conscious or think. It is always a more abstract notion
>> implemented through some machinery which do the thinking.
>>
>> Similarly a computation cannot be conscious, but it can support a person,
>> which is the one having genuinely the thinking or conscious attribute.
>>
>
> The original suggestion by Russell was that "our human consciousness _is_
> a computation (and nothing but a computation)."
>
> You seem to be steering away from Russell's straightforward position. If
> human consciousness is a computation, then the computation is conscious (it
> is an identity thesis). You say that the computation cannot be conscious,
> but can support a person. It is difficult to see this as anything other
> than the introduction of a dualistic element: the computation supports a
> conscious person, but is not itself conscious? So wherein does
> consciousness exist? You are introducing some unspecified magic into the
> equation. And what characterizes those computations that can support a
> person from those that cannot?
>
>
> Thanks for the attempt to make this clearer (if not clear).  It is helpful.
>

​Was this addressed to Bruce or to me?

>
> Let me see if I can attempt some sort of answer Bruce. The utility of the
> notion of the 'universal' mechanism is precisely its ability to emulate all
> other finitely computable mechanisms. But not all such mechanisms can be
> associated with persons. What distinguishes this particular class, as
> exemplified by Bruno's modal logic toy models is, in the first instance,
> self-referentiality. This first-personal characteristic is, if you like,
> the fixed point on which every other feature is centred.
>
>
> Is this really a fixed point in the computer science sense of recursion?
> Or are you using this in a metaphorical way?
>

​Metaphorical. It might have some technical application but I'm less than
competent to judge.
​

>
>
> Then, with respect to this fixed point of view, a distinction can be made
> between what is 'believed' (essentially, provably true as a theorem and
> hence communicable) by the machine and what is true *about* the machine
> (essentially, not provable as a theorem and hence not communicable, but
> nonetheless 'epistemically' true).
>
>
> As I understood it the things that are true put unprovable are not about
> the machine (although they may characterize it in some sense) th

Re: What falsifiability tests has computationalism passed?

2018-01-05 Thread David Nyman
On 5 January 2018 at 21:51, Brent Meeker <meeke...@verizon.net> wrote:

>
>
> On 1/5/2018 6:48 AM, David Nyman wrote:
>
> On 5 January 2018 at 14:06, Jason Resch <jasonre...@gmail.com> wrote:
>
>>
>>
>> On Friday, January 5, 2018, David Nyman <david.ny...@gmail.com> wrote:
>>
>>>
>>>
>>> On 5 Jan 2018 03:22, "Bruce Kellett" <bhkell...@optusnet.com.au> wrote:
>>>
>>> On 4/01/2018 11:59 pm, Bruno Marchal wrote:
>>>
>>>> On Jan 4, 2018, at 12:50 PM, Bruce Kellett <bhkell...@optusnet.com.au>
>>>>> wrote:
>>>>>
>>>>> On 4/01/2018 12:30 am, Bruno Marchal wrote:
>>>>>
>>>>>> On 29 Dec 2017, at 01:29, Bruce Kellett wrote:
>>>>>>
>>>>>>> On 29/12/2017 10:14 am, Russell Standish wrote:
>>>>>>>
>>>>>>>> This is computationalism - the idea that our human consciousness
>>>>>>>> _is_
>>>>>>>> a computation (and nothing but a computation).
>>>>>>>>
>>>>>>> What distinguishes a conscious computation within the class of all
>>>>>>> computations? After all, not all computations are conscious.
>>>>>>>
>>>>>> Universality seems enough.
>>>>>>
>>>>> What is a universal computation? From what you say below, universality
>>>>> appears to be a property of a machine, not of a computation.
>>>>>
>>>> OK, universality is an attribute of a machine, relatively to some
>>>> universal machinery, like arithmetic or physics.
>>>>
>>>>
>>>> But just universality gives rise only to a highly non standard,
>>>>>> dissociative, form of consciousness. It might correspond to the cosmic
>>>>>> consciousness alluded by people living highly altered state of
>>>>>> consciousness.
>>>>>>
>>>>>> You need Löbianity to get *self-consciousness*, or reflexive
>>>>>> consciousness. A machine is Löbian when its universality is knowable by 
>>>>>> it.
>>>>>> Equivalently, when the machine is universal and can prove its own "Löb's
>>>>>> formula". []([]p -> p) -> []p. Note that the second incompleteness 
>>>>>> theorem
>>>>>> is the particular case with p = f (f = "0≠1").
>>>>>>
>>>>> Löbanity is a property of the machine, not of the computation.
>>>>>
>>>> Yes. The same. I was talking about machine or about the person
>>>> supported by those machine. No machine (as conceived as a code, number,
>>>> physical object) can ever be conscious or think. It is always a more
>>>> abstract notion implemented through some machinery which do the thinking.
>>>>
>>>> Similarly a computation cannot be conscious, but it can support a
>>>> person, which is the one having genuinely the thinking or conscious
>>>> attribute.
>>>>
>>>
>>> The original suggestion by Russell was that "our human consciousness
>>> _is_ a computation (and nothing but a computation)."
>>>
>>> You seem to be steering away from Russell's straightforward position. If
>>> human consciousness is a computation, then the computation is conscious (it
>>> is an identity thesis). You say that the computation cannot be conscious,
>>> but can support a person. It is difficult to see this as anything other
>>> than the introduction of a dualistic element: the computation supports a
>>> conscious person, but is not itself conscious? So wherein does
>>> consciousness exist? You are introducing some unspecified magic into the
>>> equation. And what characterizes those computations that can support a
>>> person from those that cannot?
>>>
>>>
>>> Let me see if I can attempt some sort of answer Bruce. The utility of
>>> the notion of the 'universal' mechanism is precisely its ability to emulate
>>> all other finitely computable mechanisms. But not all such mechanisms can
>>> be associated with persons. What distinguishes this particular class, as
>>> exemplified by Bruno's modal logic toy models is, in the first instance,
>>> self-referentiality. This first-personal characteristic is, if you like,
>>> the fixed point on which every other feature is centred. Then, wit

Re: What falsifiability tests has computationalism passed?

2018-01-05 Thread David Nyman
On 5 Jan 2018 19:27, "Bruno Marchal" <marc...@ulb.ac.be> wrote:


On 4 Jan 2018, at 21:07, David Nyman <david.ny...@gmail.com> wrote:



On 4 Jan 2018 18:16, "Bruno Marchal" <marc...@ulb.ac.be> wrote:


On Jan 4, 2018, at 1:22 PM, David Nyman <da...@davidnyman.com> wrote:

On 4 January 2018 at 11:55, Bruno Marchal <marc...@ulb.ac.be> wrote:

>
> > On Jan 3, 2018, at 10:57 PM, Brent Meeker <meeke...@verizon.net> wrote:
> >
> >
> >
> > On 1/3/2018 5:47 AM, Bruno Marchal wrote:
> >>
> >> On 03 Jan 2018, at 03:39, Brent Meeker wrote:
> >>
> >>>
> >>>
> >>> On 1/2/2018 8:07 AM, Bruno Marchal wrote:
> >>>> Now, it
> >>>> could be that intelligent behavior implies mind, but as you yourself
> >>>> argue, we don't know that.
> >>>
> >>> Isn't this at the crux of the scientific study of the mind? There
> seemed to be universal agreement on this list that a philosophical zombie
> is impossible.
> >>
> >>
> >> Precisely: that a philosophical zombie is impossible when we assume
> Mechanism.
> >
> > But the consensus here has been that a philosophical zombie is
> impossible because it exhibits intelligent behavior.
>
> Well, I think the consensus here is that computationalism is far more
> plausible than non-computationalism.
> Computationalism makes zombies non sensical.
>
>
>
> >
> >> Philosophical zombie remains logical consistent for a non
> computationalist theory of mind.
> >
> > It's logically consistent with a computationalist theory of brain. It is
> only inconsistent with a computationalist theory of mind because use
> include as an axiom that computation produces mind.  One can as say that
> intelligent behavior entails mind as an axiom of physicalism.  Logic is a
> very cheap standard for theories to meet.
>
> At first sight, zombies seems consistent with computationalism, but the
> notion of zombies requires the idea that we attribute mind to bodies
> (having the right behavior). But with computationalism, mind is never
> associated to a body, but only to the person having the infinity of
> (similar enough) bodies relative representation in arithmetic. There are no
> “real bodies” or “ontological bodies”, so the notion of zombie becomes
> senseless. The consciousness is associated with the person, which is never
> determined by one body.
>

​So in the light of what you say above, does it then follow that the MGA
implies (assuming comp) that a physical system does *not* in fact implement
a computation in the relevant sense?



The physical world has to be able to implement the computation in the
relevant (Turing-Church-Post-Kleene CT) sense. You need this for the YD
“act of faith.

The physical world is a persistent illusion. It has to be enough persistent
that you wake up at the hospital with the digital brain.



I ask this because you say mind is *never* associated with a body, but mind
*is* associated with computation via the epistemic consequences of
universality.



A (conscious) third person can associate a mind/person to a body that he
perceives. It is polite.

The body perceived by that third person is itself a construction of its own
mind, and with computationalism (but also with QM), we know that such a
body is an (evolving) map of where, and in which states, we could find, sy,
the electron and proton of that body, and such snapshot is only a
computational state among infinitely many others which would works as well,
with respect to the relevant computations which brought its conscious state.
Now, the conscious first person cannot associate itself to any particular
body or computation.

Careful: sometimes I say that a machine can think, or maybe (I usually
avoid) that a computation can think or be conscious. It always mean,
respectively, that a machine can make a person capable of manifesting
itself relatively to you. But the machine and the body are local relative
representation.

A machine cannot think, and a computation (which is the (arithmetical)
dynamic 3p view of the sequence of the relative static machine/state)
cannot think. Only a (first) person can think, and to use that thinking
with respect to another person, a machine is handy, like brain or a
physical computer.

The person is in heaven (arithmetical truth) and on earth (sigma_1
arithmetical truth), simultaneously. But this belongs to G*, and I should
stay mute, or insist that we are in the “after-act-of-faith” position of
the one betting that comp is true, and … assuming comp is true. It is
subtle to talk on those things, and it is important to admit that we don’t
know the truth (or we do get inconsistent and fall in the theological trap).



If so, according to com

Re: What falsifiability tests has computationalism passed?

2018-01-05 Thread David Nyman
On 5 January 2018 at 14:06, Jason Resch <jasonre...@gmail.com> wrote:

>
>
> On Friday, January 5, 2018, David Nyman <david.ny...@gmail.com> wrote:
>
>>
>>
>> On 5 Jan 2018 03:22, "Bruce Kellett" <bhkell...@optusnet.com.au> wrote:
>>
>> On 4/01/2018 11:59 pm, Bruno Marchal wrote:
>>
>>> On Jan 4, 2018, at 12:50 PM, Bruce Kellett <bhkell...@optusnet.com.au>
>>>> wrote:
>>>>
>>>> On 4/01/2018 12:30 am, Bruno Marchal wrote:
>>>>
>>>>> On 29 Dec 2017, at 01:29, Bruce Kellett wrote:
>>>>>
>>>>>> On 29/12/2017 10:14 am, Russell Standish wrote:
>>>>>>
>>>>>>> This is computationalism - the idea that our human consciousness _is_
>>>>>>> a computation (and nothing but a computation).
>>>>>>>
>>>>>> What distinguishes a conscious computation within the class of all
>>>>>> computations? After all, not all computations are conscious.
>>>>>>
>>>>> Universality seems enough.
>>>>>
>>>> What is a universal computation? From what you say below, universality
>>>> appears to be a property of a machine, not of a computation.
>>>>
>>> OK, universality is an attribute of a machine, relatively to some
>>> universal machinery, like arithmetic or physics.
>>>
>>>
>>> But just universality gives rise only to a highly non standard,
>>>>> dissociative, form of consciousness. It might correspond to the cosmic
>>>>> consciousness alluded by people living highly altered state of
>>>>> consciousness.
>>>>>
>>>>> You need Löbianity to get *self-consciousness*, or reflexive
>>>>> consciousness. A machine is Löbian when its universality is knowable by 
>>>>> it.
>>>>> Equivalently, when the machine is universal and can prove its own "Löb's
>>>>> formula". []([]p -> p) -> []p. Note that the second incompleteness theorem
>>>>> is the particular case with p = f (f = "0≠1").
>>>>>
>>>> Löbanity is a property of the machine, not of the computation.
>>>>
>>> Yes. The same. I was talking about machine or about the person supported
>>> by those machine. No machine (as conceived as a code, number, physical
>>> object) can ever be conscious or think. It is always a more abstract notion
>>> implemented through some machinery which do the thinking.
>>>
>>> Similarly a computation cannot be conscious, but it can support a
>>> person, which is the one having genuinely the thinking or conscious
>>> attribute.
>>>
>>
>> The original suggestion by Russell was that "our human consciousness _is_
>> a computation (and nothing but a computation)."
>>
>> You seem to be steering away from Russell's straightforward position. If
>> human consciousness is a computation, then the computation is conscious (it
>> is an identity thesis). You say that the computation cannot be conscious,
>> but can support a person. It is difficult to see this as anything other
>> than the introduction of a dualistic element: the computation supports a
>> conscious person, but is not itself conscious? So wherein does
>> consciousness exist? You are introducing some unspecified magic into the
>> equation. And what characterizes those computations that can support a
>> person from those that cannot?
>>
>>
>> Let me see if I can attempt some sort of answer Bruce. The utility of the
>> notion of the 'universal' mechanism is precisely its ability to emulate all
>> other finitely computable mechanisms. But not all such mechanisms can be
>> associated with persons. What distinguishes this particular class, as
>> exemplified by Bruno's modal logic toy models is, in the first instance,
>> self-referentiality. This first-personal characteristic is, if you like,
>> the fixed point on which every other feature is centred. Then, with respect
>> to this fixed point of view, a distinction can be made between what is
>> 'believed' (essentially, provably true as a theorem and hence communicable)
>> by the machine and what is true *about* the machine (essentially, not
>> provable as a theorem and hence not communicable, but nonetheless
>> 'epistemically' true).
>>
>
>
> Given the above, what would be the shortest program with these properties?
>
> Is the Mars Rover conscious?
>

​Don't forget t

Re: What falsifiability tests has computationalism passed?

2018-01-05 Thread David Nyman
On 5 Jan 2018 03:22, "Bruce Kellett"  wrote:

On 4/01/2018 11:59 pm, Bruno Marchal wrote:

> On Jan 4, 2018, at 12:50 PM, Bruce Kellett 
>> wrote:
>>
>> On 4/01/2018 12:30 am, Bruno Marchal wrote:
>>
>>> On 29 Dec 2017, at 01:29, Bruce Kellett wrote:
>>>
 On 29/12/2017 10:14 am, Russell Standish wrote:

> This is computationalism - the idea that our human consciousness _is_
> a computation (and nothing but a computation).
>
 What distinguishes a conscious computation within the class of all
 computations? After all, not all computations are conscious.

>>> Universality seems enough.
>>>
>> What is a universal computation? From what you say below, universality
>> appears to be a property of a machine, not of a computation.
>>
> OK, universality is an attribute of a machine, relatively to some
> universal machinery, like arithmetic or physics.
>
>
> But just universality gives rise only to a highly non standard,
>>> dissociative, form of consciousness. It might correspond to the cosmic
>>> consciousness alluded by people living highly altered state of
>>> consciousness.
>>>
>>> You need Löbianity to get *self-consciousness*, or reflexive
>>> consciousness. A machine is Löbian when its universality is knowable by it.
>>> Equivalently, when the machine is universal and can prove its own "Löb's
>>> formula". []([]p -> p) -> []p. Note that the second incompleteness theorem
>>> is the particular case with p = f (f = "0≠1").
>>>
>> Löbanity is a property of the machine, not of the computation.
>>
> Yes. The same. I was talking about machine or about the person supported
> by those machine. No machine (as conceived as a code, number, physical
> object) can ever be conscious or think. It is always a more abstract notion
> implemented through some machinery which do the thinking.
>
> Similarly a computation cannot be conscious, but it can support a person,
> which is the one having genuinely the thinking or conscious attribute.
>

The original suggestion by Russell was that "our human consciousness _is_ a
computation (and nothing but a computation)."

You seem to be steering away from Russell's straightforward position. If
human consciousness is a computation, then the computation is conscious (it
is an identity thesis). You say that the computation cannot be conscious,
but can support a person. It is difficult to see this as anything other
than the introduction of a dualistic element: the computation supports a
conscious person, but is not itself conscious? So wherein does
consciousness exist? You are introducing some unspecified magic into the
equation. And what characterizes those computations that can support a
person from those that cannot?


Let me see if I can attempt some sort of answer Bruce. The utility of the
notion of the 'universal' mechanism is precisely its ability to emulate all
other finitely computable mechanisms. But not all such mechanisms can be
associated with persons. What distinguishes this particular class, as
exemplified by Bruno's modal logic toy models is, in the first instance,
self-referentiality. This first-personal characteristic is, if you like,
the fixed point on which every other feature is centred. Then, with respect
to this fixed point of view, a distinction can be made between what is
'believed' (essentially, provably true as a theorem and hence communicable)
by the machine and what is true *about* the machine (essentially, not
provable as a theorem and hence not communicable, but nonetheless
'epistemically' true).

Any machine possessing the foregoing features is in principle conscious, in
the sense of having implicit self-referential epistemic access to
non-communicable truths that are nonetheless entailed by its explicit and
communicable 'beliefs'. Of course it's a long step from the toy model to
the human person, but I think one can still discern the thread. The
machine's 'beliefs' can now be represented as communicable and explicit
third-person behaviour with respect to a 'physical' environment in which it
is embedded; however, associated with this behaviour there are true but
non-communicable epistemic phenomena to which the behaviour indirectly
refers (i.e. they are true *about* the machine). An example of this would
be any statement (or judgment, in the usual terminology of the field) you
might make about your own phenomenal experience, as in "I see an apple". In
behavioral terms, this statement or judgment is cashed out purely as
physical action (neurocognitive, neuromuscular, etc). In epistemic terms
however it cashes out as a truth (tautological and hence undoubtable)
that's implied by that same behaviour - in other words that it is in fact
the case that you really do see an apple.

It's important to take into account that all the terms employed are given
precise technical sense in mathematical logic, emulable in computation.
Bruno's computational schema is an attempt (motivated by 

Re: What falsifiability tests has computationalism passed?

2018-01-04 Thread David Nyman
On 4 Jan 2018 21:04, "Brent Meeker" <meeke...@verizon.net> wrote:



On 1/4/2018 5:13 AM, David Nyman wrote:

On 3 January 2018 at 21:57, Brent Meeker <meeke...@verizon.net> wrote:

>
>
> On 1/3/2018 5:47 AM, Bruno Marchal wrote:
>
>>
>> On 03 Jan 2018, at 03:39, Brent Meeker wrote:
>>
>>
>>>
>>> On 1/2/2018 8:07 AM, Bruno Marchal wrote:
>>>
>>>> Now, it
>>>> could be that intelligent behavior implies mind, but as you yourself
>>>> argue, we don't know that.
>>>>
>>>
>>> Isn't this at the crux of the scientific study of the mind? There seemed
>>> to be universal agreement on this list that a philosophical zombie is
>>> impossible.
>>>
>>
>>
>> Precisely: that a philosophical zombie is impossible when we assume
>> Mechanism.
>>
>
> But the consensus here has been that a philosophical zombie is impossible
> because it exhibits intelligent behavior.
>
> Philosophical zombie remains logical consistent for a non computationalist
>> theory of mind.
>>
>
> It's logically consistent with a computationalist theory of brain. It is
> only inconsistent with a computationalist theory of mind because use
> include as an axiom that computation produces mind.  One can as say that
> intelligent behavior entails mind as an axiom of physicalism.  Logic is a
> very cheap standard for theories to meet.


​ISTM that you are failing to take account of an important distinction
here, despite having acknowledged it in previous conversations.​ I don't
think of comp as really taking it as axiomatic that computation produces
mind. Of course, CTM, which comp takes as its nominal point of departure,
does do precisely that. But the comp theory seeks then to provide a
*persuasive* model of an 'internalised' epistemic access (i.e to knowledge)
that can be emulated via computation. This form of subjective access to
knowledge - as distinct from information, mechanism, or behaviour in
general - can, as you know, be represented by a toy model deploying a range
of self-referential modal logics. Of course this model doesn't directly
allow one to infer a priori the entire phenomenal spectrum of
consciousness. But it is adequate to demonstrate a range of distinctively
first-personal characteristics of mind, such as internal/external,
shareable/non-shareable, doubtable/undoubtable, that are otherwise
effectively indistinguishable from a purely third-personal perspective.
Hence part of its role is to *persuade* us that the distinction between
body and mind is coterminous with that between a 'universal' mechanistic
ontology and its possible epistemic consequences.

Of these, ISTM the most important is actually the first mentioned. The idea
that intelligent behaviour entails mind is equivalent to mind's being
something entirely extrinsic.


No, that's a logical fallacy.  That X is entailed by Y and Y is extrinsic,
does not imply that X is entirely extrinsic.


No, but if X is to be understood as intrinsic, it demands a convincing
explication of why that claim of intrinsicality is warranted, other than as
a brute a posteriori assertion. IOW, if we already have a perfectly
adequate extrinsic account of all the relevant behaviour, what reason would
you propose for the additional assumption that this somehow additionally
entails all the phenomena of subjective internality, other than this is
(inconveniently) what remains to be explained?



Neither behaviour nor matter possess, or require in any explanatory role,
an 'interior' aspect. Neither matter nor behaviour have an 'inside' - plumb
their depths as you will and all you will discover is more 'outsides'. So
the axiom that intelligent behaviour entails mind would seem either to be
an effective elimination of the concept as redundant (popular in certain
recently discussed circles) or a theoretically ex nihilo evocation of
first-personal epistemic access on the exclusive basis of third-personal
action. The former option undercuts itself at the start; the latter seems
to lack any theoretical motivation other than a tacit (or on occasion
explicit) dismissal of the viability of any alternative approach to the
matter.


That's the same argument you've made in several different forms: Any third
person explanation of mind implies that there can be no first person
experience.  I don't see that it follows.


It doesn't. It's just that it's unargued for except as a brute 'identity'.
That's my point. That's why I contrasted it with the explicitly epistemic
elements of Bruno's theory. That's the sum and total of the distinction I'm
pointing out.

  Any reduction banishes the thing reduced.


Not at all. But that's just the point. In third person terms, when you
build a house from bricks, you don't then have the bricks plus a house
somehow evoked ex nihilo. 

Re: What falsifiability tests has computationalism passed?

2018-01-04 Thread David Nyman
On 4 Jan 2018 18:16, "Bruno Marchal" <marc...@ulb.ac.be> wrote:


On Jan 4, 2018, at 1:22 PM, David Nyman <da...@davidnyman.com> wrote:

On 4 January 2018 at 11:55, Bruno Marchal <marc...@ulb.ac.be> wrote:

>
> > On Jan 3, 2018, at 10:57 PM, Brent Meeker <meeke...@verizon.net> wrote:
> >
> >
> >
> > On 1/3/2018 5:47 AM, Bruno Marchal wrote:
> >>
> >> On 03 Jan 2018, at 03:39, Brent Meeker wrote:
> >>
> >>>
> >>>
> >>> On 1/2/2018 8:07 AM, Bruno Marchal wrote:
> >>>> Now, it
> >>>> could be that intelligent behavior implies mind, but as you yourself
> >>>> argue, we don't know that.
> >>>
> >>> Isn't this at the crux of the scientific study of the mind? There
> seemed to be universal agreement on this list that a philosophical zombie
> is impossible.
> >>
> >>
> >> Precisely: that a philosophical zombie is impossible when we assume
> Mechanism.
> >
> > But the consensus here has been that a philosophical zombie is
> impossible because it exhibits intelligent behavior.
>
> Well, I think the consensus here is that computationalism is far more
> plausible than non-computationalism.
> Computationalism makes zombies non sensical.
>
>
>
> >
> >> Philosophical zombie remains logical consistent for a non
> computationalist theory of mind.
> >
> > It's logically consistent with a computationalist theory of brain. It is
> only inconsistent with a computationalist theory of mind because use
> include as an axiom that computation produces mind.  One can as say that
> intelligent behavior entails mind as an axiom of physicalism.  Logic is a
> very cheap standard for theories to meet.
>
> At first sight, zombies seems consistent with computationalism, but the
> notion of zombies requires the idea that we attribute mind to bodies
> (having the right behavior). But with computationalism, mind is never
> associated to a body, but only to the person having the infinity of
> (similar enough) bodies relative representation in arithmetic. There are no
> “real bodies” or “ontological bodies”, so the notion of zombie becomes
> senseless. The consciousness is associated with the person, which is never
> determined by one body.
>

​So in the light of what you say above, does it then follow that the MGA
implies (assuming comp) that a physical system does *not* in fact implement
a computation in the relevant sense?



The physical world has to be able to implement the computation in the
relevant (Turing-Church-Post-Kleene CT) sense. You need this for the YD
“act of faith.

The physical world is a persistent illusion. It has to be enough persistent
that you wake up at the hospital with the digital brain.



I ask this because you say mind is *never* associated with a body, but mind
*is* associated with computation via the epistemic consequences of
universality.



A (conscious) third person can associate a mind/person to a body that he
perceives. It is polite.

The body perceived by that third person is itself a construction of its own
mind, and with computationalism (but also with QM), we know that such a
body is an (evolving) map of where, and in which states, we could find, sy,
the electron and proton of that body, and such snapshot is only a
computational state among infinitely many others which would works as well,
with respect to the relevant computations which brought its conscious state.
Now, the conscious first person cannot associate itself to any particular
body or computation.

Careful: sometimes I say that a machine can think, or maybe (I usually
avoid) that a computation can think or be conscious. It always mean,
respectively, that a machine can make a person capable of manifesting
itself relatively to you. But the machine and the body are local relative
representation.

A machine cannot think, and a computation (which is the (arithmetical)
dynamic 3p view of the sequence of the relative static machine/state)
cannot think. Only a (first) person can think, and to use that thinking
with respect to another person, a machine is handy, like brain or a
physical computer.

The person is in heaven (arithmetical truth) and on earth (sigma_1
arithmetical truth), simultaneously. But this belongs to G*, and I should
stay mute, or insist that we are in the “after-act-of-faith” position of
the one betting that comp is true, and … assuming comp is true. It is
subtle to talk on those things, and it is important to admit that we don’t
know the truth (or we do get inconsistent and fall in the theological trap).



If so, according to comp, it would follow that (the material appearance and
behaviour of) a body cannot be considered *causally* relevant to the
computation-mind polarity,



Yes, 

Re: What falsifiability tests has computationalism passed?

2018-01-04 Thread David Nyman
On 4 January 2018 at 11:55, Bruno Marchal  wrote:

>
> > On Jan 3, 2018, at 10:57 PM, Brent Meeker  wrote:
> >
> >
> >
> > On 1/3/2018 5:47 AM, Bruno Marchal wrote:
> >>
> >> On 03 Jan 2018, at 03:39, Brent Meeker wrote:
> >>
> >>>
> >>>
> >>> On 1/2/2018 8:07 AM, Bruno Marchal wrote:
>  Now, it
>  could be that intelligent behavior implies mind, but as you yourself
>  argue, we don't know that.
> >>>
> >>> Isn't this at the crux of the scientific study of the mind? There
> seemed to be universal agreement on this list that a philosophical zombie
> is impossible.
> >>
> >>
> >> Precisely: that a philosophical zombie is impossible when we assume
> Mechanism.
> >
> > But the consensus here has been that a philosophical zombie is
> impossible because it exhibits intelligent behavior.
>
> Well, I think the consensus here is that computationalism is far more
> plausible than non-computationalism.
> Computationalism makes zombies non sensical.
>
>
>
> >
> >> Philosophical zombie remains logical consistent for a non
> computationalist theory of mind.
> >
> > It's logically consistent with a computationalist theory of brain. It is
> only inconsistent with a computationalist theory of mind because use
> include as an axiom that computation produces mind.  One can as say that
> intelligent behavior entails mind as an axiom of physicalism.  Logic is a
> very cheap standard for theories to meet.
>
> At first sight, zombies seems consistent with computationalism, but the
> notion of zombies requires the idea that we attribute mind to bodies
> (having the right behavior). But with computationalism, mind is never
> associated to a body, but only to the person having the infinity of
> (similar enough) bodies relative representation in arithmetic. There are no
> “real bodies” or “ontological bodies”, so the notion of zombie becomes
> senseless. The consciousness is associated with the person, which is never
> determined by one body.
>

​So in the light of what you say above, does it then follow that the MGA
implies (assuming comp) that a physical system does *not* in fact implement
a computation in the relevant sense? I ask this because you say mind is
*never* associated with a body, but mind *is* associated with computation
via the epistemic consequences of universality. If so, according to comp,
it would follow that (the material appearance and behaviour of) a body
cannot be considered *causally* relevant to the computation-mind polarity,
but instead must be regarded as a consistent *consequence* of it.

David​


>
>
>
> >
> >>
> >>
> >>
> >>
> >>> Of course that doesn't mean it's true. But it seems as good a working
> hypothesis as "Yes, doctor".  And in fact it's the working hypothesis of
> most studies of neurocognition, intelligence, and mind.
> >>
> >> Neuroscience and AI often bet, more or less explicitly, on mechanism,
> or on its "strong AI" weakenings.
> >>
> >> (Note that UDA use mechanism, but its translation in arithmetic needs
> only strong-AI. Note that if strong AI is true, and comp false, we get
> infinitely many zombies in arithmetic.
> >
> > How do you know that?
>
>
> I was wrong. Wrote to quickly. It is only if weak AI is true, and strong
> AI or comp false, that there will be infinitely many zombies in arithmetic.
> Of course, if strong AI is false, comp is false too.
>
>
>
>
>
> >
> >> very curious one, which lacks body and mind, but act like you and me.
> They are quite similar with the "Bohm's zombies", the beings in the
> branches of the universal quantum wave which have no particles.
> >>
> >>
> >>> If it's true then it provides a link from intelligent behavior to mind.
> >>
> >> The "non-zombie" principle is a consequence of comp, but I doubt that
> it implies comp. It is not related to finiteness, as comp and strong AI are.
> >>
> >>
> >>
> >>> We already have links from from physics to brain to intelligent
> behavior.  So why isn't this the physics based theory of mind that Bruno et
> al keep saying is impossible?
> >>
> >> This is a bit ambiguous and misleading. Comp makes physics necessary,
> and that is why with Occam, physics cannot be assumed primitively if we
> want to use actual physics to verify or refute comp.
> >
> > That very much depends on what physics comp makes necessary.
>
> Well, if it violate our empirical physics, comp is refuted.
>
>
>
> >
> >> We can of course assume physics when doing physics, but not when doing
> computationalist theory of mind.
> >
> > No, but we can assume physics when doing physicalist theory of mind.
>
> Yes, but then the point is that a physicalist theory of mind (like with
> consciousness reducing the wave) will be non-computationalist.
>
>
> >
> >>
> >> OK? "physics" is necessary for machine/numbers is what makes the
> physical assumption eliminable, and is what makes computationalism testable.
> >
> > But it doesn't seem to be testable because the conclusions drawn from it
> are extremely 

Re: Dreamless Sleep and the Dream Argument

2018-01-01 Thread David Nyman
On 1 January 2018 at 19:34, Brent Meeker <meeke...@verizon.net> wrote:

>
>
> On 1/1/2018 4:39 AM, David Nyman wrote:
>
> https://www.frontiersin.org/articles/10.3389/fpsyg.2017.01924/full
>
> This link
> ​could serve as quite​
> an illuminating adjunct to the dreamless sleep thread. The authors begin
> by asserting that 'consciousness' - by which they do
> ​indeed ​
> appear to mean phenomenal awareness as distinct from any
> ​of its
> reductive correlates - has no causal powers distinct from those correlates
> or
> ​any of the ​deeper
> processes giving rise to
> ​them​
> . But
> ​from this starting assertion they ​
> then
> ​g​
> o on to speculate about the 'evolutionary utility' of the 'narratives'
> thus created. They don't seem to have noticed the tacit elision from
> 'narrative' to (presumably) the correlates of narrative, thus bypassing
> entirely the notion of consciousness in their original sense, however much
> they might wish to analogise it as a 'rainbow effect'.
> ​ ​
> This is about as classic an example of the tacit switching between
> categories that characterises
> ​discussion of emergence in the philosophy of mind
> as you could
> ​wish to ​
> find.
>
>
> I think you are imagining a clean distinction between levels of discourse
> that cannot exist.
>

​Of course I agree that it cannot exist. What I don't seem to have conveyed
to you however is that my point is precisely that the apparently seamless
bottom-up causality of the physical narrative seduces people towards making
such a distinction where it suits them and evading it where it doesn't,
Both tendencies are exampled in this paper. The bottom-up effectiveness of
neurocognition (as a proxy for physical causality) is called upon to
explain away that of phenomenal consciousness. Then, no sooner has this
been stated, but the power of the conscious narrative is evoked as part of
an explanation at the level of evolution (equally a proxy for physical
causality). As I pointed out, to be consistent, 'conscious narrative' must
here be assumed to have tacitly elided from the phenomenal to the neural
correlative version. As you point out below, we do this sort of thing all
the time, usually without danger of losing the plot. But it won't do here,
because this elision simply results in the erasure of any principled
distinction between any putative 'neural correlates' of consciousness and
those of any other aspect of brain function. Either category, in the final
analysis, serves as a proxy for the physical causality of which each is
ultimately a re-description (aka emergent). Consequently we have now
contrived to lose the category of phenomenal consciousness altogether other
than as a so-called 'rainbow effect' (i.e. the notorious 'seemings' or
'illusions') or as an implicit brain-mind 'identity'.

  When we talk about the effect of a law, such as legalizing marijuana, we
> may discuss it in terms of action of the local Sheriff.  That's a tacit
> switch in categories, but it's not some intellectual sin.  So I see no deep
> problem in discussing the evolutionary utility of conscious narrative even
> if the narrative is an emergent epiphenomenon.  My theory is that the
> conscious narrative is a construct which summarizes and makes consistent an
> account of what is happening and it's utility is learning, including
> formulating questions and exchanges of social learning.  The link seems to
> subscribe to a version of the Cartesian theater, which Dennett sharply
> criticizes and instead proposes the mutliple-drafts model.  That's pretty
> close of Jeff Hawkins six layered hierarchical model of the function of the
> cortex in which consciousness accompanies integration of disparate inputs
> into a coherent thought at the top level.  I can imagine constructing an AI
> Mars Rover which integrates the input from all its sensors and their
> correlates from associative memory into a kind of log-book entry which then
> goes into memory for future reference when some unusual problem arises or
> when some long range plan is to be formulated.
>

​Forgive me, but these points are all too obvious. You must consider me a
terrible duffer for you to think it worth your while to keep re-explaining
them to me. No doubt you're correct about my dufferhood, but in this
particular case you can save yourself further effort. ​The reason that I
restated the thing in the form of the Dream Argument was in an attempt to
anchor the conversation at the level of the phenomenal. Thereafter the task
becomes one of reconciling that narrative with the highly-constrained
features of material causality with which the dreamer is confronted. For
example, how and why do emergent (non-reducible) 'material phenomena'
*appear* to be capable of stepwise structural and behavioural correlation
with nested sets of ever

Re: Dreamless Sleep and the Dream Argument

2018-01-01 Thread David Nyman
On 1 January 2018 at 15:02, Bruno Marchal <marc...@ulb.ac.be> wrote:

>
> On 01 Jan 2018, at 13:39, David Nyman wrote:
>
> https://www.frontiersin.org/articles/10.3389/fpsyg.2017.01924/full
>
> This link
> ​could serve as quite​
> an illuminating adjunct to the dreamless sleep thread. The authors begin
> by asserting that 'consciousness' - by which they do
> ​indeed ​
> appear to mean phenomenal awareness as distinct from any
> ​of its
> reductive correlates - has no causal powers distinct from those correlates
> or
> ​any of the ​deeper
> processes giving rise to
> ​them​
> .
>
>
> I guess that you agree that this is already a sort of mistake; It is
> basically the same mistake that the lawyer who justify its client actions
> by saying that his/her client has no causal power distinct from the laws of
> physics. This eliminates consciousness  and responsibility, which is close
> to non-sense.
>

​Yes, I agree of course. But this is indeed the state of affairs if one
follows, without tacit additions, what is *strictly* proposed by physics -
or rather, by physicalism (i.e. the implicit metaphysics of physics). This
then leads directly to the zombie problem. In fact it leads even beyond
this, because as you go on to say vis-a-vis 'number reductionism', strictly
speaking there would be no independent justification for the zombie as an
'emergent' causal entity, since ex hypothesi all 'causality' has already
been accounted for at the level of elementary particles, fields, strings
are what you will.


> Of course, a mechanist knows that at its substitution level, he has the
> same causal power than its components when betting on some reality, but we
> can never know what are those components in a rational way, and our causal
> power (free will, or will) is a higher construct, not present in any
> subpart of any third person description of ourself.
>

​Yes. But the difference between mechanism and physicalism is that with the
former one can infer a rationale​, via self-reference and its consequences,
for the appearance and specific characteristics of these higher 'emergent'
constructs and thereby test to what extent they match the phenomena we seek
to explain. Physicalism, by contrast, has so far discovered no such a
priori rationale and so is forced into falling back on the (often tacit)
assertion of a unique and mysterious species of 'non-identical' identity
thesis in a purely a posteriori attempt to account for 'emergent'
phenomena. Alternatively it tries to sweep those very phenomena under the
rug with terminology such as 'seemings' and 'illusions'. There's nothing in
that move however that prevents us from demanding an explication of the how
these so-called seemings or illusions produce the very particular
impressions they do (i.e. our entire phenomenal reality) unless we're being
asked in effect to believe in magic.


>  If they did the same error when assuming Mechanism and its immaterialist
> consequence, they would eliminate not only consciousness, but the
> appearance of matter as well.
>
They would become "number reductionist", which is correct for the ontology,
> but nonsensical for the phenomenology. No consciousness and no matter!
>

​Actually,
​AFAICS, ​
this applies to physicalism also, if you remove all the additional a
posteriori ​assumptions, such as
​'​
emergence
​' (other than as an explanatory device)​
. A fully-reduced physicalism does *not* give you the appearance of matter,
even when 'visualised'
​as​
the View from Nowhere. That view is always an implicit interpretation of
the reduced state of affairs, converting it once again into the 'emergent'
forms afforded by perception. But to present that as the 'solution' to the
Body Problem is to beg the question in the most egregious manner.


> We don't have that problem with mechanism. Consciousness has a big role,
> like speeding up our relative means of actions, computations, etc. (That is
> not obvious, but comes from a theorem by Gödel on the length of proof).
> Then matter is a sort of consciousness construct, but not a human
> consciousness construct, as all machine/number are involved below the
> substitution level.
>
> But
> ​from this starting assertion they ​
> then
> ​g​
> o on to speculate about the 'evolutionary utility' of the 'narratives'
> thus created. They don't seem to have noticed the tacit elision from
> 'narrative' to (presumably) the correlates of narrative, thus bypassing
> entirely the notion of consciousness in their original sense, however much
> they might wish to analogise it as a 'rainbow effect'.
> ​ ​
> This is about as classic an example of the tacit switching between
> categories that characterises
> ​discussion of emergence in the philosophy of mind
> as you could
> ​wish to ​
> find.
>
>
> Indeed. It 

Dreamless Sleep and the Dream Argument

2018-01-01 Thread David Nyman
https://www.frontiersin.org/articles/10.3389/fpsyg.2017.01924/full

This link
​could serve as quite​
an illuminating adjunct to the dreamless sleep thread. The authors begin by
asserting that 'consciousness' - by which they do
​indeed ​
appear to mean phenomenal awareness as distinct from any
​of its
reductive correlates - has no causal powers distinct from those correlates
or
​any of the ​deeper
processes giving rise to
​them​
. But
​from this starting assertion they ​
then
​g​
o on to speculate about the 'evolutionary utility' of the 'narratives' thus
created. They don't seem to have noticed the tacit elision from 'narrative'
to (presumably) the correlates of narrative, thus bypassing entirely the
notion of consciousness in their original sense, however much they might
wish to analogise it as a 'rainbow effect'.
​ ​
This is about as classic an example of the tacit switching between
categories that characterises
​discussion of emergence in the philosophy of mind
as you could
​wish to ​
find.

That apart, their
​struggle to find a 'purpose' or 'utility' in consciousness might motivate
a return to the Dream Argument as a point of departure, or at least an
overarching metaphor, for computationalism. If we take Berkeley as the
exemplar of this tendency in the Western canon, ​what was missing in his
treatment was any attempt to understand in detail how the multiplicity of
thoughts in the 'divine mind' could come to be correlated in the kind of
consistent system of 'physical' action we observe (other than by divine
decree, of course). Essentially, he re-proposed the antique Dream Argument
in a Christian context but without too much of an an eye to its consilience
with the other, ultimately more influential rationalist trends of his time.
But the attraction of this point of departure still remains. For one thing
it need not tempt us to deny the 'evidence of our own eyes'. But for
another, it may enable us to reframe the problem that the authors of the
above paper get so confused about - which is to say the 'purpose' or
'utility' of consciousness. But that is to set off down a rabbit hole that
leads to a very different construction (literally) of things. And indeed to
a reversal of, or perhaps more correctly the idea of a two-way reciprocity
between, the notions of mental and physical 'causality'.

David

-- 
You received this message because you are subscribed to the Google Groups 
"Everything List" group.
To unsubscribe from this group and stop receiving emails from it, send an email 
to everything-list+unsubscr...@googlegroups.com.
To post to this group, send email to everything-list@googlegroups.com.
Visit this group at https://groups.google.com/group/everything-list.
For more options, visit https://groups.google.com/d/optout.


Re: Dreamless Sleep?

2017-12-30 Thread David Nyman
On 30 December 2017 at 23:38, Brent Meeker <meeke...@verizon.net> wrote:

>
>
> On 12/29/2017 4:51 PM, David Nyman wrote:
>
> On 23 December 2017 at 02:13, Brent Meeker <meeke...@verizon.net> wrote:
>
>>
>>
>> On 12/22/2017 4:50 PM, David Nyman wrote:
>>
>>
>>
>> On 22 Dec 2017 23:16, "Brent Meeker" <meeke...@verizon.net> wrote:
>>
>>
>>
>> On 12/22/2017 6:31 AM, David Nyman wrote:
>>
>>
>>
>> On 22 Dec 2017 11:22, "Telmo Menezes" <te...@telmomenezes.com> wrote:
>>
>> On Thu, Dec 21, 2017 at 2:01 PM, David Nyman <da...@davidnyman.com>
>> wrote:
>> > On 21 December 2017 at 11:34, Telmo Menezes <te...@telmomenezes.com>
>> wrote:
>> >>
>> >> > So we are told.  But what if someone could look at a recorded MRI of
>> you
>> >> > brain and tell you what you were thinking?
>> >>
>> >> Why do you need the MRI? You can look at the text that I write and
>> >> know what I'm thinking. We've been doing that all along.
>> >> The text I write comes from my fingers hitting the keyboard, and the
>> >> fingers move in a certain pattern because the muscles are activated by
>> >> nerves that are connected to my brain and completely correlated to my
>> >> neural activity. What does the MRI add beyond precision? How does this
>> >> help solve the mystery that I am conscious, instead of a zombie?
>> >
>> >
>> > Well put.
>> >
>> > However if we follow Bruno in taking the antique Dream Argument as our
>> point
>> > of departure (which to a certain extent can be made distinct from an
>> > explicitly computationalist hypothesis) then the question becomes:
>> >
>> > Starting from the position that these present thoughts and sensations
>> (i.e.
>> > the 'waking' dream) are beyond doubt, and that they appear also to
>> refer to
>> > events in an externalised field of action, how does it come to be the
>> case
>> > that all this appears to play out in the very particular way it does?
>> >
>> > When the question is asked in some such way, it should perhaps not then
>> be
>> > unexpected that brains, nervous systems and bodies, as intrinsic
>> components
>> > of the field of action in question, appear precisely to be mechanisms
>> (in
>> > the generalised sense for now) for translating transactions, between
>> > themselves and the remainder of that field, into action. And also
>> > unsurprising that this continues to generalise whatever detailed level
>> of
>> > analysis is applied to the field in question, whether 'narrower' or
>> 'wider'
>> > in focus (i.e. the consistency requirement). And further that this is
>> just
>> > the sort of tightly-constrained and consistent set of mechanisms that we
>> > might expect to be picked out from an even more generalised
>> 'mechanistic'
>> > environment, owing to the very particular requirements of the
>> > 'self-observation' with which we began.
>> >
>> > So far, perhaps so un-Hard. But the question then still remains of the
>> > precise relation between the phenomena of the dream itself and the
>> > transactional mechanisms that make their appearance within it,
>> including and
>> > especially the aforementioned brains. If we turn for a moment to an
>> analogy,
>> > it doesn't surprise us, when watching a movie play out on an LCD screen,
>> > that the mechanism that implements this playing out fails to resemble
>> point
>> > for point, although is obviously systematically correlated with, the
>> > ultimate phenomena it stimulates the viewer into realising. But the
>> reason
>> > of course for our lack of surprise is that we consider the bulk of the
>> > burden of such realisation to be shouldered by the viewer's brain, not
>> by
>> > the LCD device alone. So for that reason, no such loophole seems
>> possible
>> > for the final relation between the phenomena of the dream and the
>> mechanisms
>> > of the brain itself. It must somehow shoulder the final burden of
>> > 'self-observation' and 'self-interpretation'; the matter can no longer
>> be
>> > 'externalised'.
>> >
>> > Hence to explicate the matter further, what is needed is a conceptual
>> > apparatus - i.e. in the Western tradition, a mathematical theory -
>> adequate
>> &

Re: Dreamless Sleep?

2017-12-30 Thread David Nyman
On 30 Dec 2017 21:38, "Brent Meeker" <meeke...@verizon.net> wrote:



On 12/30/2017 7:41 AM, Bruno Marchal wrote:


On 26 Dec 2017, at 22:06, Brent Meeker wrote:



On 12/26/2017 6:48 AM, Bruno Marchal wrote:


On 23 Dec 2017, at 03:13, Brent Meeker wrote:



On 12/22/2017 4:50 PM, David Nyman wrote:



On 22 Dec 2017 23:16, "Brent Meeker" <meeke...@verizon.net> wrote:



On 12/22/2017 6:31 AM, David Nyman wrote:



On 22 Dec 2017 11:22, "Telmo Menezes" <te...@telmomenezes.com> wrote:

On Thu, Dec 21, 2017 at 2:01 PM, David Nyman <da...@davidnyman.com> wrote:
> On 21 December 2017 at 11:34, Telmo Menezes <te...@telmomenezes.com>
wrote:
>>
>> > So we are told.  But what if someone could look at a recorded MRI of
you
>> > brain and tell you what you were thinking?
>>
>> Why do you need the MRI? You can look at the text that I write and
>> know what I'm thinking. We've been doing that all along.
>> The text I write comes from my fingers hitting the keyboard, and the
>> fingers move in a certain pattern because the muscles are activated by
>> nerves that are connected to my brain and completely correlated to my
>> neural activity. What does the MRI add beyond precision? How does this
>> help solve the mystery that I am conscious, instead of a zombie?
>
>
> Well put.
>
> However if we follow Bruno in taking the antique Dream Argument as our
point
> of departure (which to a certain extent can be made distinct from an
> explicitly computationalist hypothesis) then the question becomes:
>
> Starting from the position that these present thoughts and sensations
(i.e.
> the 'waking' dream) are beyond doubt, and that they appear also to refer
to
> events in an externalised field of action, how does it come to be the case
> that all this appears to play out in the very particular way it does?
>
> When the question is asked in some such way, it should perhaps not then be
> unexpected that brains, nervous systems and bodies, as intrinsic
components
> of the field of action in question, appear precisely to be mechanisms (in
> the generalised sense for now) for translating transactions, between
> themselves and the remainder of that field, into action. And also
> unsurprising that this continues to generalise whatever detailed level of
> analysis is applied to the field in question, whether 'narrower' or
'wider'
> in focus (i.e. the consistency requirement). And further that this is just
> the sort of tightly-constrained and consistent set of mechanisms that we
> might expect to be picked out from an even more generalised 'mechanistic'
> environment, owing to the very particular requirements of the
> 'self-observation' with which we began.
>
> So far, perhaps so un-Hard. But the question then still remains of the
> precise relation between the phenomena of the dream itself and the
> transactional mechanisms that make their appearance within it, including
and
> especially the aforementioned brains. If we turn for a moment to an
analogy,
> it doesn't surprise us, when watching a movie play out on an LCD screen,
> that the mechanism that implements this playing out fails to resemble
point
> for point, although is obviously systematically correlated with, the
> ultimate phenomena it stimulates the viewer into realising. But the reason
> of course for our lack of surprise is that we consider the bulk of the
> burden of such realisation to be shouldered by the viewer's brain, not by
> the LCD device alone. So for that reason, no such loophole seems possible
> for the final relation between the phenomena of the dream and the
mechanisms
> of the brain itself. It must somehow shoulder the final burden of
> 'self-observation' and 'self-interpretation'; the matter can no longer be
> 'externalised'.
>
> Hence to explicate the matter further, what is needed is a conceptual
> apparatus - i.e. in the Western tradition, a mathematical theory -
adequate
> to the explication of an entirely 'internal' relation between the dream
> phenomena and their transactional mechanisms.  At this point, enter the
> Computationalist Hypothesis, or of course any other theory that cares to
> test its mettle for the purpose. ISTM that formulating the matter in this
> way genuinely makes any putatively remaining 'Hard' problems seem less
> intractable, at the cost of putting the 'Aristotelian' position on matter
> into question (but arguably this is already a lost cause even within
physics
> itself). However in a sense it's also a different form of WYSIWYG, in that
> the dream always and forever is both what you see and what you get. But if
> you want to study its detailed mechanisms of action you need to delve into
> the realms of unobservable abstraction. The s

Re: Dreamless Sleep?

2017-12-29 Thread David Nyman
On 23 December 2017 at 02:13, Brent Meeker <meeke...@verizon.net> wrote:

>
>
> On 12/22/2017 4:50 PM, David Nyman wrote:
>
>
>
> On 22 Dec 2017 23:16, "Brent Meeker" <meeke...@verizon.net> wrote:
>
>
>
> On 12/22/2017 6:31 AM, David Nyman wrote:
>
>
>
> On 22 Dec 2017 11:22, "Telmo Menezes" <te...@telmomenezes.com> wrote:
>
> On Thu, Dec 21, 2017 at 2:01 PM, David Nyman <da...@davidnyman.com> wrote:
> > On 21 December 2017 at 11:34, Telmo Menezes <te...@telmomenezes.com>
> wrote:
> >>
> >> > So we are told.  But what if someone could look at a recorded MRI of
> you
> >> > brain and tell you what you were thinking?
> >>
> >> Why do you need the MRI? You can look at the text that I write and
> >> know what I'm thinking. We've been doing that all along.
> >> The text I write comes from my fingers hitting the keyboard, and the
> >> fingers move in a certain pattern because the muscles are activated by
> >> nerves that are connected to my brain and completely correlated to my
> >> neural activity. What does the MRI add beyond precision? How does this
> >> help solve the mystery that I am conscious, instead of a zombie?
> >
> >
> > Well put.
> >
> > However if we follow Bruno in taking the antique Dream Argument as our
> point
> > of departure (which to a certain extent can be made distinct from an
> > explicitly computationalist hypothesis) then the question becomes:
> >
> > Starting from the position that these present thoughts and sensations
> (i.e.
> > the 'waking' dream) are beyond doubt, and that they appear also to refer
> to
> > events in an externalised field of action, how does it come to be the
> case
> > that all this appears to play out in the very particular way it does?
> >
> > When the question is asked in some such way, it should perhaps not then
> be
> > unexpected that brains, nervous systems and bodies, as intrinsic
> components
> > of the field of action in question, appear precisely to be mechanisms (in
> > the generalised sense for now) for translating transactions, between
> > themselves and the remainder of that field, into action. And also
> > unsurprising that this continues to generalise whatever detailed level of
> > analysis is applied to the field in question, whether 'narrower' or
> 'wider'
> > in focus (i.e. the consistency requirement). And further that this is
> just
> > the sort of tightly-constrained and consistent set of mechanisms that we
> > might expect to be picked out from an even more generalised 'mechanistic'
> > environment, owing to the very particular requirements of the
> > 'self-observation' with which we began.
> >
> > So far, perhaps so un-Hard. But the question then still remains of the
> > precise relation between the phenomena of the dream itself and the
> > transactional mechanisms that make their appearance within it, including
> and
> > especially the aforementioned brains. If we turn for a moment to an
> analogy,
> > it doesn't surprise us, when watching a movie play out on an LCD screen,
> > that the mechanism that implements this playing out fails to resemble
> point
> > for point, although is obviously systematically correlated with, the
> > ultimate phenomena it stimulates the viewer into realising. But the
> reason
> > of course for our lack of surprise is that we consider the bulk of the
> > burden of such realisation to be shouldered by the viewer's brain, not by
> > the LCD device alone. So for that reason, no such loophole seems possible
> > for the final relation between the phenomena of the dream and the
> mechanisms
> > of the brain itself. It must somehow shoulder the final burden of
> > 'self-observation' and 'self-interpretation'; the matter can no longer be
> > 'externalised'.
> >
> > Hence to explicate the matter further, what is needed is a conceptual
> > apparatus - i.e. in the Western tradition, a mathematical theory -
> adequate
> > to the explication of an entirely 'internal' relation between the dream
> > phenomena and their transactional mechanisms.  At this point, enter the
> > Computationalist Hypothesis, or of course any other theory that cares to
> > test its mettle for the purpose. ISTM that formulating the matter in this
> > way genuinely makes any putatively remaining 'Hard' problems seem less
> > intractable, at the cost of putting the 'Aristotelian' position on matter
> > into question (but arguably this is already a lost cause even within
> physics
> > it

Re: Dreamless Sleep?

2017-12-22 Thread David Nyman
On 22 Dec 2017 23:16, "Brent Meeker" <meeke...@verizon.net> wrote:



On 12/22/2017 6:31 AM, David Nyman wrote:



On 22 Dec 2017 11:22, "Telmo Menezes" <te...@telmomenezes.com> wrote:

On Thu, Dec 21, 2017 at 2:01 PM, David Nyman <da...@davidnyman.com> wrote:
> On 21 December 2017 at 11:34, Telmo Menezes <te...@telmomenezes.com>
wrote:
>>
>> > So we are told.  But what if someone could look at a recorded MRI of
you
>> > brain and tell you what you were thinking?
>>
>> Why do you need the MRI? You can look at the text that I write and
>> know what I'm thinking. We've been doing that all along.
>> The text I write comes from my fingers hitting the keyboard, and the
>> fingers move in a certain pattern because the muscles are activated by
>> nerves that are connected to my brain and completely correlated to my
>> neural activity. What does the MRI add beyond precision? How does this
>> help solve the mystery that I am conscious, instead of a zombie?
>
>
> Well put.
>
> However if we follow Bruno in taking the antique Dream Argument as our
point
> of departure (which to a certain extent can be made distinct from an
> explicitly computationalist hypothesis) then the question becomes:
>
> Starting from the position that these present thoughts and sensations
(i.e.
> the 'waking' dream) are beyond doubt, and that they appear also to refer
to
> events in an externalised field of action, how does it come to be the case
> that all this appears to play out in the very particular way it does?
>
> When the question is asked in some such way, it should perhaps not then be
> unexpected that brains, nervous systems and bodies, as intrinsic
components
> of the field of action in question, appear precisely to be mechanisms (in
> the generalised sense for now) for translating transactions, between
> themselves and the remainder of that field, into action. And also
> unsurprising that this continues to generalise whatever detailed level of
> analysis is applied to the field in question, whether 'narrower' or
'wider'
> in focus (i.e. the consistency requirement). And further that this is just
> the sort of tightly-constrained and consistent set of mechanisms that we
> might expect to be picked out from an even more generalised 'mechanistic'
> environment, owing to the very particular requirements of the
> 'self-observation' with which we began.
>
> So far, perhaps so un-Hard. But the question then still remains of the
> precise relation between the phenomena of the dream itself and the
> transactional mechanisms that make their appearance within it, including
and
> especially the aforementioned brains. If we turn for a moment to an
analogy,
> it doesn't surprise us, when watching a movie play out on an LCD screen,
> that the mechanism that implements this playing out fails to resemble
point
> for point, although is obviously systematically correlated with, the
> ultimate phenomena it stimulates the viewer into realising. But the reason
> of course for our lack of surprise is that we consider the bulk of the
> burden of such realisation to be shouldered by the viewer's brain, not by
> the LCD device alone. So for that reason, no such loophole seems possible
> for the final relation between the phenomena of the dream and the
mechanisms
> of the brain itself. It must somehow shoulder the final burden of
> 'self-observation' and 'self-interpretation'; the matter can no longer be
> 'externalised'.
>
> Hence to explicate the matter further, what is needed is a conceptual
> apparatus - i.e. in the Western tradition, a mathematical theory -
adequate
> to the explication of an entirely 'internal' relation between the dream
> phenomena and their transactional mechanisms.  At this point, enter the
> Computationalist Hypothesis, or of course any other theory that cares to
> test its mettle for the purpose. ISTM that formulating the matter in this
> way genuinely makes any putatively remaining 'Hard' problems seem less
> intractable, at the cost of putting the 'Aristotelian' position on matter
> into question (but arguably this is already a lost cause even within
physics
> itself). However in a sense it's also a different form of WYSIWYG, in that
> the dream always and forever is both what you see and what you get. But if
> you want to study its detailed mechanisms of action you need to delve into
> the realms of unobservable abstraction. The slogan might then be: The
> concrete is the subjective reflection of the abstract.

David, excellent text.

Taking the cue of your slogan (which I love), see if you agree:

A possible model of what is happening is that there is an objective
reality that is independent from any of us, and that is made of
matter.


OK

Re: Dreamless Sleep?

2017-12-22 Thread David Nyman
On 22 Dec 2017 19:57, "Brent Meeker"  wrote:



On 12/22/2017 2:25 AM, Telmo Menezes wrote:

> On Thu, Dec 21, 2017 at 8:11 PM, Brent Meeker 
> wrote:
>
>>
>> On 12/21/2017 3:34 AM, Telmo Menezes wrote:
>>
>>> So we are told.  But what if someone could look at a recorded MRI of you
 brain and tell you what you were thinking?

>>> Why do you need the MRI? You can look at the text that I write and
>>> know what I'm thinking. We've been doing that all along.
>>> The text I write comes from my fingers hitting the keyboard, and the
>>> fingers move in a certain pattern because the muscles are activated by
>>> nerves that are connected to my brain and completely correlated to my
>>> neural activity. What does the MRI add beyond precision? How does this
>>> help solve the mystery that I am conscious, instead of a zombie?
>>>
>>
>> Well, you can't lie to the MRI.  But otherwise I agree.  Except that I
>> then
>> ask, "What mystery?"  If having thoughts, however expressed or detected,
>> is
>> consciousness then problem solved...or more accurately pushed back to why
>> do
>> we believe a philosophical zombie is impossible.
>>
> Alright, I think we can agree on some important things. I would say
> that we are both inclined to believe that:
>
> "Certain configuration of matter are correlated with certain states of
> consciousness, and it must be so."
>
> Yes?
>
> The mystery here is: why must it be so? It is a perfectly legitimate
> scientific question, I would say.
>

Any question is legitimate if you can think of a what an answer might be or
how to test it.  But haven't you ever been engage with someone who has a
naive but enthusiastic view of science and so asks lots of questions like
"Why is the speed of light constant?" or "Why are there only two electric
charges?" or "Why did the universe expand?"   At the fundamental level
science doesn't answer "why" questions, because an answer would have to
invoke a more basic level (hence my virtuous circle model of explanation).
Of course you can never know that you're at the fundamental level.



The point I'm gently trying to make is that the "hard problem of
consciousness" is a why question, as you've posed it above, and scientific
progress is made by answering "how" questions.


Gently? OK buster, that's enough. Who are you and why are you pretending
to be Brent Meeker?


It seems to me that people who want an answer to the "the hard problem" are
asking why can't we explain consciousness the way we explain gravity and
metabolism and atoms.  I'm saying we can - it's just that all those
explanations are how explanations and so let's get some "how" explanations
of consciousness - the engineering approach.


Well, whoever you are, that just sounds too bloody reasonable to disagree
with.

David


Brent
The sciences do not try to explain, they hardly even try to interpret, they
mainly make models. By a model is meant a mathematical construct which,
with the addition of certain verbal interpretations, describes observed
phenomena. The justification of such a mathematical construct is solely and
precisely that it is expected to work.
--—John von Neumann


-- 
You received this message because you are subscribed to the Google Groups
"Everything List" group.
To unsubscribe from this group and stop receiving emails from it, send an
email to everything-list+unsubscr...@googlegroups.com.
To post to this group, send email to everything-list@googlegroups.com.
Visit this group at https://groups.google.com/group/everything-list.
For more options, visit https://groups.google.com/d/optout.

-- 
You received this message because you are subscribed to the Google Groups 
"Everything List" group.
To unsubscribe from this group and stop receiving emails from it, send an email 
to everything-list+unsubscr...@googlegroups.com.
To post to this group, send email to everything-list@googlegroups.com.
Visit this group at https://groups.google.com/group/everything-list.
For more options, visit https://groups.google.com/d/optout.


Re: Dreamless Sleep?

2017-12-22 Thread David Nyman
On 22 Dec 2017 11:22, "Telmo Menezes" <te...@telmomenezes.com> wrote:

On Thu, Dec 21, 2017 at 2:01 PM, David Nyman <da...@davidnyman.com> wrote:
> On 21 December 2017 at 11:34, Telmo Menezes <te...@telmomenezes.com>
wrote:
>>
>> > So we are told.  But what if someone could look at a recorded MRI of
you
>> > brain and tell you what you were thinking?
>>
>> Why do you need the MRI? You can look at the text that I write and
>> know what I'm thinking. We've been doing that all along.
>> The text I write comes from my fingers hitting the keyboard, and the
>> fingers move in a certain pattern because the muscles are activated by
>> nerves that are connected to my brain and completely correlated to my
>> neural activity. What does the MRI add beyond precision? How does this
>> help solve the mystery that I am conscious, instead of a zombie?
>
>
> Well put.
>
> However if we follow Bruno in taking the antique Dream Argument as our
point
> of departure (which to a certain extent can be made distinct from an
> explicitly computationalist hypothesis) then the question becomes:
>
> Starting from the position that these present thoughts and sensations
(i.e.
> the 'waking' dream) are beyond doubt, and that they appear also to refer
to
> events in an externalised field of action, how does it come to be the case
> that all this appears to play out in the very particular way it does?
>
> When the question is asked in some such way, it should perhaps not then be
> unexpected that brains, nervous systems and bodies, as intrinsic
components
> of the field of action in question, appear precisely to be mechanisms (in
> the generalised sense for now) for translating transactions, between
> themselves and the remainder of that field, into action. And also
> unsurprising that this continues to generalise whatever detailed level of
> analysis is applied to the field in question, whether 'narrower' or
'wider'
> in focus (i.e. the consistency requirement). And further that this is just
> the sort of tightly-constrained and consistent set of mechanisms that we
> might expect to be picked out from an even more generalised 'mechanistic'
> environment, owing to the very particular requirements of the
> 'self-observation' with which we began.
>
> So far, perhaps so un-Hard. But the question then still remains of the
> precise relation between the phenomena of the dream itself and the
> transactional mechanisms that make their appearance within it, including
and
> especially the aforementioned brains. If we turn for a moment to an
analogy,
> it doesn't surprise us, when watching a movie play out on an LCD screen,
> that the mechanism that implements this playing out fails to resemble
point
> for point, although is obviously systematically correlated with, the
> ultimate phenomena it stimulates the viewer into realising. But the reason
> of course for our lack of surprise is that we consider the bulk of the
> burden of such realisation to be shouldered by the viewer's brain, not by
> the LCD device alone. So for that reason, no such loophole seems possible
> for the final relation between the phenomena of the dream and the
mechanisms
> of the brain itself. It must somehow shoulder the final burden of
> 'self-observation' and 'self-interpretation'; the matter can no longer be
> 'externalised'.
>
> Hence to explicate the matter further, what is needed is a conceptual
> apparatus - i.e. in the Western tradition, a mathematical theory -
adequate
> to the explication of an entirely 'internal' relation between the dream
> phenomena and their transactional mechanisms.  At this point, enter the
> Computationalist Hypothesis, or of course any other theory that cares to
> test its mettle for the purpose. ISTM that formulating the matter in this
> way genuinely makes any putatively remaining 'Hard' problems seem less
> intractable, at the cost of putting the 'Aristotelian' position on matter
> into question (but arguably this is already a lost cause even within
physics
> itself). However in a sense it's also a different form of WYSIWYG, in that
> the dream always and forever is both what you see and what you get. But if
> you want to study its detailed mechanisms of action you need to delve into
> the realms of unobservable abstraction. The slogan might then be: The
> concrete is the subjective reflection of the abstract.

David, excellent text.

Taking the cue of your slogan (which I love), see if you agree:

A possible model of what is happening is that there is an objective
reality that is independent from any of us, and that is made of
matter.


OK, but even saying that is already assuming more than is actually
warranted by the evidence, as your remarks about the epistemolog

Re: Dreamless Sleep?

2017-12-21 Thread David Nyman
On 21 Dec 2017 20:47, "Brent Meeker" <meeke...@verizon.net> wrote:



On 12/21/2017 12:37 PM, David Nyman wrote:



On 21 Dec 2017 19:25, "Brent Meeker" <meeke...@verizon.net> wrote:



On 12/21/2017 5:01 AM, David Nyman wrote:

On 21 December 2017 at 11:34, Telmo Menezes <te...@telmomenezes.com> wrote:

> > So we are told.  But what if someone could look at a recorded MRI of you
> > brain and tell you what you were thinking?
>
> Why do you need the MRI? You can look at the text that I write and
> know what I'm thinking. We've been doing that all along.
> The text I write comes from my fingers hitting the keyboard, and the
> fingers move in a certain pattern because the muscles are activated by
> nerves that are connected to my brain and completely correlated to my
> neural activity. What does the MRI add beyond precision? How does this
> help solve the mystery that I am conscious, instead of a zombie?
>

​Well put.

However if we follow Bruno in taking the antique Dream Argument as our
point of departure (which to a certain extent can be made distinct from an
explicitly computationalist hypothesis) then the question becomes:

Starting from the position that these present thoughts and sensations (i.e.
the 'waking' dream) are beyond doubt, and that they appear also to refer to
events in an externalised field of action, how does it come to be the case
that all this appears to play out in the very particular way it does?

When the question is asked in some such way, it should perhaps not then be
unexpected that brains, nervous systems and bodies, as intrinsic components
of the field of action in question, appear precisely to be mechanisms (in
the generalised sense for now) for translating transactions, between
themselves and the remainder of that field, into action. And also
unsurprising that this continues to generalise whatever detailed level of
analysis is applied to the field in question, whether 'narrower' or 'wider'
in focus (i.e. the consistency requirement). And further that this is just
the sort of tightly-constrained and consistent set of mechanisms that we
might expect to be picked out from an even more generalised 'mechanistic'
environment, owing to the very particular requirements of the
'self-observation' with which we began.

So far, perhaps so un-Hard. But the question then still remains of the
precise relation between the phenomena of the dream itself and the
transactional mechanisms that make their appearance within it, including
and especially the aforementioned brains. If we turn for a moment to an
analogy, it doesn't surprise us, when watching a movie play out on an LCD
screen, that the mechanism that implements this playing out fails to
resemble point for point, although is obviously systematically correlated
with, the ultimate phenomena it stimulates the viewer into realising. But
the reason of course for our lack of surprise is that we consider the bulk
of the burden of such realisation to be shouldered by the viewer's brain,
not by the LCD device alone. So for that reason, no such loophole seems
possible for the final relation between the phenomena of the dream and the
mechanisms of the brain itself. It must somehow shoulder the final burden
of 'self-observation' and 'self-interpretation'; the matter can no longer
be 'externalised'.


Good explication.  And I think I agree on the reason for the scare quotes.
The 'self-observation' by introspection is really very limited and it seems
that external observation of action tells us things about what someone is
thinking that are not available to introspection.  One of the nice things
about Bruno's theory is that implies this division...but in an extremely
idealized form.


I don't get it Brent. You seem to either violently agree or equally
disagree with what I say, as in the case of your other most recent
comments. Can you clarify for me what differentiates the two cases?


OK, I'll try to agree and disagree more gently.


Thanks, but bear in mind that I meant it more as a plea for clarity than
charity. ;-)

David



Brent

-- 
You received this message because you are subscribed to the Google Groups
"Everything List" group.
To unsubscribe from this group and stop receiving emails from it, send an
email to everything-list+unsubscr...@googlegroups.com.
To post to this group, send email to everything-list@googlegroups.com.
Visit this group at https://groups.google.com/group/everything-list.
For more options, visit https://groups.google.com/d/optout.

-- 
You received this message because you are subscribed to the Google Groups 
"Everything List" group.
To unsubscribe from this group and stop receiving emails from it, send an email 
to everything-list+unsubscr...@googlegroups.com.
To post to this group, send email to everything-list@googlegroups.com.
Visit this group at https://groups.google.com/group/everything-list.
For more options, visit https://groups.google.com/d/optout.


Re: Dreamless Sleep?

2017-12-21 Thread David Nyman
On 21 Dec 2017 20:46, "Brent Meeker" <meeke...@verizon.net> wrote:



On 12/21/2017 12:28 PM, David Nyman wrote:



On 21 Dec 2017 18:58, "Brent Meeker" <meeke...@verizon.net> wrote:



On 12/21/2017 3:27 AM, Telmo Menezes wrote:

> Hi David,
>
> Sometimes your responses really puzzle me Brent. What you say above almost
>> makes it sound as though you just don't get the distinction Telmo is
>> pointing to. But based on what you have said at other times I think you do
>> get it, but because you also know that there's really no explicating that
>> distinction in a purely third person way, you sometimes want to say that
>> that's as far as explanation can legitimately go and the rest is just woo.
>>
>
Actually I think of it the other way around.  You and Chalmers et al are so
invested in the "hard problem" being hard


Please be advised that you have no sound basis for expounding on what I'm
supposedly 'invested' in. For the record, I would admit to being invested
in not sweeping a problem under the carpet because it's inconvenient. That
said, I have no particular commitment to one style of explanation above
another except to the extent that it seems to promise an advance or
impediment to understanding. For my part, any disagreement between us rests
wholly on those considerations, as in the present case.


that you overlook the fact that almost all problems are "hard"; they have
no fixed, objective ontological foundation.


You're changing the subject, not for the first time. That's not the
relevant sense of the term in this case, as I assumed you accepted. I said
I thought you understood what clearly and categorically differentiates this
problem from the ones you mention, or any other in the canon. But I'm
prepared to revise my opinion if you insist.

We showed that life was based on chemistry, and chemistry was based on
molecular physics, which was based on atomic physics, which was based on
quantum field theory, which assumes spacetime, which we don't understand.
Seems hard.  If we succeed in explaining spacetime in terms of entanglement
of quantum states will that be the end?  I doubt it.  "The end" may just be
the end of our grasp.


Well, yes it may be. That's even a philosophical position of sorts. But my
interest in Bruno's work is precisely because in my experience it's an
approach to the elusive relation between the mental and the physical that
seems capable of working with the relevant categories of both without
ignoring or distorting either. I haven't in all honesty encountered any
others about which I could say the same. That doesn't make it correct of
course, but it does, in my view, render your effective dismissal of the
problem area, as either illusory, irrelevant or insoluble, somewhat
premature.


When you write "there's really no explicating that distinction in a purely
third person way"  it seems analogous to a vitalist saying, "Sure, maybe
life is chemistry, but that doesn't explain what it feels like to be
alive."


That's a distortion of the vitalist position. What you say here about the
feeling of being alive is more like a restatement of the HP. Vitalism was
first and foremost an inability to clearly specify and hence differentiate
living and non-living processes, which seemed at a gross level of analysis
to be so unlike each other as to require the intervention of an additional
causal principle. When the the relation between chemistry and biology was
better understood, it became apparent that this was not the case. But the
matter never strayed, nor needed to, from the detailed explication of third
person processes of one sort or another. Hence nothing 'vital' is thereby
omitted.


You're insisting that the feeling of being conscious must be explicated in
a non-third person way...which is a contradiction in terms.  "Explication"
is a third person relation. You want an explanation but you want to keep
the mystery too.


It must be explicated in a way that differentiates first and third person
categories in the relevant and indispensable ways. Bruno for one has given
us at least a start in seeing how that could be handled at least in
principle. In particular, it must show why and how the first person is not
consigned, from a merely a posteriori position, to the status of an
arbitrarily superadded category, as it is and must always be in any purely
third person explication.


Suppose it were found, in the course of my "engineering solution" that
certain kinds of self referential information processing (which of course
would obey Goedel's limitations) were necessary or evolutionarily favored
aspects of AI.  Why would that not be just as satisfactory a solution as
Bruno's?  Why is self reference in abstract mathematical proofs a solution,
while an evolutionary explication not?


That would be a very interesting result, and indeed indispe

Re: Dreamless Sleep?

2017-12-21 Thread David Nyman
On 21 Dec 2017 19:25, "Brent Meeker" <meeke...@verizon.net> wrote:



On 12/21/2017 5:01 AM, David Nyman wrote:

On 21 December 2017 at 11:34, Telmo Menezes <te...@telmomenezes.com> wrote:

> > So we are told.  But what if someone could look at a recorded MRI of you
> > brain and tell you what you were thinking?
>
> Why do you need the MRI? You can look at the text that I write and
> know what I'm thinking. We've been doing that all along.
> The text I write comes from my fingers hitting the keyboard, and the
> fingers move in a certain pattern because the muscles are activated by
> nerves that are connected to my brain and completely correlated to my
> neural activity. What does the MRI add beyond precision? How does this
> help solve the mystery that I am conscious, instead of a zombie?
>

​Well put.

However if we follow Bruno in taking the antique Dream Argument as our
point of departure (which to a certain extent can be made distinct from an
explicitly computationalist hypothesis) then the question becomes:

Starting from the position that these present thoughts and sensations (i.e.
the 'waking' dream) are beyond doubt, and that they appear also to refer to
events in an externalised field of action, how does it come to be the case
that all this appears to play out in the very particular way it does?

When the question is asked in some such way, it should perhaps not then be
unexpected that brains, nervous systems and bodies, as intrinsic components
of the field of action in question, appear precisely to be mechanisms (in
the generalised sense for now) for translating transactions, between
themselves and the remainder of that field, into action. And also
unsurprising that this continues to generalise whatever detailed level of
analysis is applied to the field in question, whether 'narrower' or 'wider'
in focus (i.e. the consistency requirement). And further that this is just
the sort of tightly-constrained and consistent set of mechanisms that we
might expect to be picked out from an even more generalised 'mechanistic'
environment, owing to the very particular requirements of the
'self-observation' with which we began.

So far, perhaps so un-Hard. But the question then still remains of the
precise relation between the phenomena of the dream itself and the
transactional mechanisms that make their appearance within it, including
and especially the aforementioned brains. If we turn for a moment to an
analogy, it doesn't surprise us, when watching a movie play out on an LCD
screen, that the mechanism that implements this playing out fails to
resemble point for point, although is obviously systematically correlated
with, the ultimate phenomena it stimulates the viewer into realising. But
the reason of course for our lack of surprise is that we consider the bulk
of the burden of such realisation to be shouldered by the viewer's brain,
not by the LCD device alone. So for that reason, no such loophole seems
possible for the final relation between the phenomena of the dream and the
mechanisms of the brain itself. It must somehow shoulder the final burden
of 'self-observation' and 'self-interpretation'; the matter can no longer
be 'externalised'.


Good explication.  And I think I agree on the reason for the scare quotes.
The 'self-observation' by introspection is really very limited and it seems
that external observation of action tells us things about what someone is
thinking that are not available to introspection.  One of the nice things
about Bruno's theory is that implies this division...but in an extremely
idealized form.


I don't get it Brent. You seem to either violently agree or equally
disagree with what I say, as in the case of your other most recent
comments. Can you clarify for me what differentiates the two cases?

David



Brent



Hence to explicate the matter further, what is needed is a conceptual
apparatus - i.e. in the Western tradition, a mathematical theory - adequate
to the explication of an entirely 'internal' relation between the dream
phenomena and their transactional mechanisms.  At this point, enter the
Computationalist Hypothesis, or of course any other theory that cares to
test its mettle for the purpose. ISTM that formulating the matter in this
way genuinely makes any putatively remaining 'Hard' problems seem less
intractable, at the cost of putting the 'Aristotelian' position on matter
into question (but arguably this is already a lost cause even within
physics itself). However in a sense it's also a different form of WYSIWYG,
in that the dream always and forever is both what you see and what you get.
But if you want to study its detailed mechanisms of action you need to
delve into the realms of unobservable abstraction. The slogan might then
be: The concrete is the subjective reflection of the abstract.

David


> Telmo.
>
> --
> You received this message because you are subscribed to the Google Gr

Re: Dreamless Sleep?

2017-12-21 Thread David Nyman
On 21 Dec 2017 18:58, "Brent Meeker"  wrote:



On 12/21/2017 3:27 AM, Telmo Menezes wrote:

> Hi David,
>
> Sometimes your responses really puzzle me Brent. What you say above almost
>> makes it sound as though you just don't get the distinction Telmo is
>> pointing to. But based on what you have said at other times I think you do
>> get it, but because you also know that there's really no explicating that
>> distinction in a purely third person way, you sometimes want to say that
>> that's as far as explanation can legitimately go and the rest is just woo.
>>
>
Actually I think of it the other way around.  You and Chalmers et al are so
invested in the "hard problem" being hard


Please be advised that you have no sound basis for expounding on what I'm
supposedly 'invested' in. For the record, I would admit to being invested
in not sweeping a problem under the carpet because it's inconvenient. That
said, I have no particular commitment to one style of explanation above
another except to the extent that it seems to promise an advance or
impediment to understanding. For my part, any disagreement between us rests
wholly on those considerations, as in the present case.


that you overlook the fact that almost all problems are "hard"; they have
no fixed, objective ontological foundation.


You're changing the subject, not for the first time. That's not the
relevant sense of the term in this case, as I assumed you accepted. I said
I thought you understood what clearly and categorically differentiates this
problem from the ones you mention, or any other in the canon. But I'm
prepared to revise my opinion if you insist.

We showed that life was based on chemistry, and chemistry was based on
molecular physics, which was based on atomic physics, which was based on
quantum field theory, which assumes spacetime, which we don't understand.
Seems hard.  If we succeed in explaining spacetime in terms of entanglement
of quantum states will that be the end?  I doubt it.  "The end" may just be
the end of our grasp.


Well, yes it may be. That's even a philosophical position of sorts. But my
interest in Bruno's work is precisely because in my experience it's an
approach to the elusive relation between the mental and the physical that
seems capable of working with the relevant categories of both without
ignoring or distorting either. I haven't in all honesty encountered any
others about which I could say the same. That doesn't make it correct of
course, but it does, in my view, render your effective dismissal of the
problem area, as either illusory, irrelevant or insoluble, somewhat
premature.


When you write "there's really no explicating that distinction in a purely
third person way"  it seems analogous to a vitalist saying, "Sure, maybe
life is chemistry, but that doesn't explain what it feels like to be
alive."


That's a distortion of the vitalist position. What you say here about the
feeling of being alive is more like a restatement of the HP. Vitalism was
first and foremost an inability to clearly specify and hence differentiate
living and non-living processes, which seemed at a gross level of analysis
to be so unlike each other as to require the intervention of an additional
causal principle. When the the relation between chemistry and biology was
better understood, it became apparent that this was not the case. But the
matter never strayed, nor needed to, from the detailed explication of third
person processes of one sort or another. Hence nothing 'vital' is thereby
omitted.


You're insisting that the feeling of being conscious must be explicated in
a non-third person way...which is a contradiction in terms.  "Explication"
is a third person relation. You want an explanation but you want to keep
the mystery too.


It must be explicated in a way that differentiates first and third person
categories in the relevant and indispensable ways. Bruno for one has given
us at least a start in seeing how that could be handled at least in
principle. In particular, it must show why and how the first person is not
consigned, from a merely a posteriori position, to the status of an
arbitrarily superadded category, as it is and must always be in any purely
third person explication. Which was Telmo's point.

David



Brent
"One cannot guess the real difficulties of a problem before
having solved it."
   --- Carl Ludwig Siegel

Thanks for saying. This puzzles me too. It's not just Brent, I know a
> lot of smart people that do exactly the same.
>
> Cute but irrelevant. As has been said when we've discussed Telmo's point in
>> the past, the fact of the matter is that ontological reduction *just is*
>> ontological elimination. That's the whole point of the reductive project
>> and
>> precisely therein lies its explanatory power. But somehow that same
>> ontological reduction doesn't entail *epistemological* elimination.
>> There's
>> the rub.
>>
> Precisely.
> For me, and for these reasons, emergentism in its 

Re: Dreamless Sleep?

2017-12-21 Thread David Nyman
On 21 December 2017 at 11:34, Telmo Menezes  wrote:

> > So we are told.  But what if someone could look at a recorded MRI of you
> > brain and tell you what you were thinking?
>
> Why do you need the MRI? You can look at the text that I write and
> know what I'm thinking. We've been doing that all along.
> The text I write comes from my fingers hitting the keyboard, and the
> fingers move in a certain pattern because the muscles are activated by
> nerves that are connected to my brain and completely correlated to my
> neural activity. What does the MRI add beyond precision? How does this
> help solve the mystery that I am conscious, instead of a zombie?
>

​Well put.

However if we follow Bruno in taking the antique Dream Argument as our
point of departure (which to a certain extent can be made distinct from an
explicitly computationalist hypothesis) then the question becomes:

Starting from the position that these present thoughts and sensations (i.e.
the 'waking' dream) are beyond doubt, and that they appear also to refer to
events in an externalised field of action, how does it come to be the case
that all this appears to play out in the very particular way it does?

When the question is asked in some such way, it should perhaps not then be
unexpected that brains, nervous systems and bodies, as intrinsic components
of the field of action in question, appear precisely to be mechanisms (in
the generalised sense for now) for translating transactions, between
themselves and the remainder of that field, into action. And also
unsurprising that this continues to generalise whatever detailed level of
analysis is applied to the field in question, whether 'narrower' or 'wider'
in focus (i.e. the consistency requirement). And further that this is just
the sort of tightly-constrained and consistent set of mechanisms that we
might expect to be picked out from an even more generalised 'mechanistic'
environment, owing to the very particular requirements of the
'self-observation' with which we began.

So far, perhaps so un-Hard. But the question then still remains of the
precise relation between the phenomena of the dream itself and the
transactional mechanisms that make their appearance within it, including
and especially the aforementioned brains. If we turn for a moment to an
analogy, it doesn't surprise us, when watching a movie play out on an LCD
screen, that the mechanism that implements this playing out fails to
resemble point for point, although is obviously systematically correlated
with, the ultimate phenomena it stimulates the viewer into realising. But
the reason of course for our lack of surprise is that we consider the bulk
of the burden of such realisation to be shouldered by the viewer's brain,
not by the LCD device alone. So for that reason, no such loophole seems
possible for the final relation between the phenomena of the dream and the
mechanisms of the brain itself. It must somehow shoulder the final burden
of 'self-observation' and 'self-interpretation'; the matter can no longer
be 'externalised'.

Hence to explicate the matter further, what is needed is a conceptual
apparatus - i.e. in the Western tradition, a mathematical theory - adequate
to the explication of an entirely 'internal' relation between the dream
phenomena and their transactional mechanisms.  At this point, enter the
Computationalist Hypothesis, or of course any other theory that cares to
test its mettle for the purpose. ISTM that formulating the matter in this
way genuinely makes any putatively remaining 'Hard' problems seem less
intractable, at the cost of putting the 'Aristotelian' position on matter
into question (but arguably this is already a lost cause even within
physics itself). However in a sense it's also a different form of WYSIWYG,
in that the dream always and forever is both what you see and what you get.
But if you want to study its detailed mechanisms of action you need to
delve into the realms of unobservable abstraction. The slogan might then
be: The concrete is the subjective reflection of the abstract.

David


> Telmo.
>
> --
> You received this message because you are subscribed to the Google Groups
> "Everything List" group.
> To unsubscribe from this group and stop receiving emails from it, send an
> email to everything-list+unsubscr...@googlegroups.com.
> To post to this group, send email to everything-list@googlegroups.com.
> Visit this group at https://groups.google.com/group/everything-list.
> For more options, visit https://groups.google.com/d/optout.
>

-- 
You received this message because you are subscribed to the Google Groups 
"Everything List" group.
To unsubscribe from this group and stop receiving emails from it, send an email 
to everything-list+unsubscr...@googlegroups.com.
To post to this group, send email to everything-list@googlegroups.com.
Visit this group at https://groups.google.com/group/everything-list.
For more options, visit 

Re: Dreamless Sleep?

2017-12-18 Thread David Nyman
On 18 December 2017 at 07:08, Brent Meeker  wrote:

>
>
> On 12/17/2017 9:03 AM, Telmo Menezes wrote:
>
> On Fri, Dec 8, 2017 at 7:53 PM, Brent Meeker  
>  wrote:
>
> On 12/8/2017 2:24 AM, Telmo Menezes wrote:
>
> On Thu, Dec 7, 2017 at 10:47 PM, Brent Meeker  
> 
> wrote:
>
> On 12/7/2017 1:01 AM, Telmo Menezes wrote:
>
> On Wed, Dec 6, 2017 at 11:50 PM, Brent Meeker  
> 
> wrote:
>
> On 12/6/2017 1:46 AM, Bruno Marchal wrote:
>
> I suspect that this is perhaps why Brent want to refer to the
> environment
> for relating consciousness to the machine, and in Artificial
> Intelligence,
> some people defend the idea that (mundane) consciousness occur only
> when
> the
> environment contradicts a little bit the quasi automatic persistent
> inference we do all the time.
>
> That's Jeff Hawkins model of consciousness: one becomes conscious of
> something when all lower, more specialized levels of the brain have
> found
> it
> not to match their predictions.
>
> In that sort of model, how does matter "know" that it is being used to
> run a forecasting algorithm? Surely it doesn't right?
>
> ?? Why surely.  It seems you're rejecting the idea that a physical system
> can be conscious just out of prejudice.
>
> Not at all. I remain agnostic on materialism vs. idealism. Maybe I am
> even a strong agnostic: I suspect that the answer to this question
> cannot be known.
>
> Assuming materialism, consciousness must indeed be a property or
> something that emerges from the interaction of fundamental particles,
> the same way that, say, life does. Ok. All that I am saying is that
> nobody has proposed any explanation of consciousness under this
> assumption that I would call a theory. The above is not a theory, in
> the same way that the Christian God is not a theory: it proposes to
> explain a simple thing by appealing to a pre-existing more complex
> thing -- in this case claiming that the act of forecasting at a very
> high level somehow leads to consciousness, but without proposing any
> first principles. It's a magical step.
>
> What would a satisfactory (to you) first principle look like.
>
> I cannot imagine one -- and this fuels my intuition that consciousness
> is more fundamental than matter,
>
>
> It fuels my intuition that it is a "wrong question".
>
> and that emergentism is a dead-end.
> But of course, my lack of imagination is not an argument. It could be
> that I am too dumb/ignorant/crazy to come up with a good emergentist
> theory. What I can -- and do -- is listen to any idea that comes up
> and have an open mind. If you have one, I will gladly listen.
>
>
> If we
> consider the analogy of life, in the early 1900's when it was considered as
> a chemical process all that could be said about it was that it involved
> using energy to construct carbon based compounds and at a high level this
> led to reproduction and natural selection and the origin of species.  Now,
> we have greatly elaborated on the molecular chemistry and can modify and
> even created DNA and RNA molecules that realize "life".  Where did we get
> past the "magical step"?  Or are you still waiting for "the atom of life" to
> be discovered?
>
> Here there is no magical step. Life can be understood all the way down
> to basic chemistry. Ok, we don't have all the details, but we are not
> missing anything fundamental. I am not waiting for the atoms of life
> because I already know what they are. You just described them above.
> Can you do that for consciousness?
>
>
> Maybe not yet, but I can imagine what they might be: self-awareness,
> construction of narratives about one's experiences, modeling other minds,...
>

​Sometimes your responses really puzzle me Brent. What you say above almost
makes it sound as though you just don't get the distinction Telmo is
pointing to. But based on what you have said at other times I think you do
get it, but because you also know that there's really no explicating that
distinction in a purely third person way, you sometimes want to say that
that's as far as explanation can legitimately go and the rest is just woo.


>
> What makes the hard problem hard is that it relates to a qualitatively
> different phenomena than anything else that we try to understand. Life
> can be talked about purely in the third person, but consciousness is
> first person by definition.
>
>
> So we are told.  But what if someone could look at a recorded MRI of you
> brain and tell you what you were thinking?
>

​Yeah, but notice also that there's only ever one person who can attest to
the truth of that.
​

>
>
> My view is that this sort of emergentism always smuggles a subtle but
> important switcheroo at some point: moving from epistemology to
> ontology.
>
> For me, emergence is an epistemic tool. It is not possible for a human
> to understand hyper-complex systems by 

Re: Dreamless Sleep?

2017-12-08 Thread David Nyman
> easy problem and that try to smuggle it as a solution for the hard
> problem, by relying on the overloading of terms.
>

Progress is made by solving the problems you can.  But as you know I think
"the hard problem" will go away when the "easy problem" is solved.  When we
can produce AI's that are creative, humorous, compassionate, imaginative,
etc  and adjust those attributes and understand how they are
implemented...the "hard problem" will be seen as the wrong question.
Instead AI engineers will ask, "Well, how much consciousness do you want?
We recommend more subconscious competence for that task,"

Already there seems to be a consensus that a philosophical zombie is
impossible.  That entails that any AI with human level (or greater)
intelligence must be conscious.  The AI engineers will develop different
realizations of intelligent machines and invent terms for the different
ways in which they are conscious.  Then "consciousness" will be seen as a
vague generalization covering many somewhat different processes.


I'm sorry to hear your so-called conscious processes result in things
seeming generally vague Brent. Let's hope they'll seem clearer by
Christmas.


"One cannot guess the real difficulties of a problem before having solved
it."
   --- Carl Ludwig Siegel


"One cannot solve a problem before having dealt with the its real
difficulties"
   --- David Nyman

David ;-)



-- 
You received this message because you are subscribed to the Google Groups
"Everything List" group.
To unsubscribe from this group and stop receiving emails from it, send an
email to everything-list+unsubscr...@googlegroups.com.
To post to this group, send email to everything-list@googlegroups.com.
Visit this group at https://groups.google.com/group/everything-list.
For more options, visit https://groups.google.com/d/optout.

-- 
You received this message because you are subscribed to the Google Groups 
"Everything List" group.
To unsubscribe from this group and stop receiving emails from it, send an email 
to everything-list+unsubscr...@googlegroups.com.
To post to this group, send email to everything-list@googlegroups.com.
Visit this group at https://groups.google.com/group/everything-list.
For more options, visit https://groups.google.com/d/optout.


Re: Dreamless Sleep?

2017-12-08 Thread David Nyman
On 8 Dec 2017 03:31, "Brent Meeker" <meeke...@verizon.net> wrote:



On 12/7/2017 5:05 PM, David Nyman wrote:

On 7 December 2017 at 21:49, Brent Meeker <meeke...@verizon.net> wrote:

>
>
> On 12/7/2017 9:36 AM, David Nyman wrote:
>
>
>
> On 7 Dec 2017 15:08, "Bruno Marchal" <marc...@ulb.ac.be> wrote:
>
>
> On 07 Dec 2017, at 10:01, Telmo Menezes wrote:
>
> On Wed, Dec 6, 2017 at 11:50 PM, Brent Meeker <meeke...@verizon.net>
>> wrote:
>>
>>>
>>>
>>> On 12/6/2017 1:46 AM, Bruno Marchal wrote:
>>>
>>>>
>>>> I suspect that this is perhaps why Brent want to refer to the
>>>> environment
>>>> for relating consciousness to the machine, and in Artificial
>>>> Intelligence,
>>>> some people defend the idea that (mundane) consciousness occur only
>>>> when the
>>>> environment contradicts a little bit the quasi automatic persistent
>>>> inference we do all the time.
>>>>
>>>
>>>
>>> That's Jeff Hawkins model of consciousness: one becomes conscious of
>>> something when all lower, more specialized levels of the brain have
>>> found it
>>> not to match their predictions.
>>>
>>
>> In that sort of model, how does matter "know" that it is being used to
>> run a forecasting algorithm? Surely it doesn't right?
>>
>
> Imagine a society which builds some objects. When everything go well, the
> boss can sleep in his office. But then there is some accident or something
> unusual. That is the time to wake up the boss. In this analogy,
> consciousness is played by the (incorrigible) boss.
>
>
>
>
> The only way this could work is if the forecasting algorithm and the
>> cascading effects of failing predictions have the side effect of
>> creating the "right" sort of interactions at a lower level that
>> trigger consciousness.
>>
>
> After a moment of panic, the sub-entities dare to awake the ultimate
> judge: the one capable of "going out of the box" to take a (perhaps risky)
> decision in absence of complete information, and to take on its shoulder
> the responsibility.
>
>
>
>
> Then I want to know what these interactions
>> are, and what if the "atom" of consciousness, what is the first
>> principle. Without this, I would say that such hypothesis are not even
>> wrong.
>>
>
> The sub-unities have specialized task, and does not need evolved
> forecasting ability. You can think them as ants, when they do their usual
> jobs triggered by the local pheromones left by their close neighbors. But
> if the nest is attacked, or if some important food is missing, some species
> will needs some order of the queen (ike to fight or to move away. Some
> societies can delegate most of the power to the sub-unities, but in complex
> unknown situation, if they have to make important decision, they will need
> a centralization of the power, which can act much more quickly to convince
> the whole society of some unusual option, like running away, closing the
> doors, fighting the enemy, etc. That will happen when *many* ants complain
> on something.
>
> In this case, the role of consciousness is focusing the attention on what
> is important (with respect to survival), and to speed-up planning,
> decision, etc.
>
> I am not sure this answer the question (we are in the "easy" part of the
> problem here).
>
>
> I follow you here, but I'd like to make a comment on the "hard" side of
> the problem. What comp implies in its ineffably strange way, given that
> matter itself becomes an appearance, is that strictly speaking we should
> say that the "easy" part of the story is only what "appears" to be
> happening. So neurocognition itself is a sort of (very precise and
> constrained) story, narrated in terms of physical action, itself emulated
> in computation.
>
> From the perspective of reality or truth we get an interpretation or
> meaning in terms of which such stories can make sense, but each 'level' has
> its own proper logic; and the logic of material appearance is that of the
> 'laws' of physics. Nothing else is necessary, at least at that level, to
> account for the disposition and evolution of material states. So strictly
> speaking, when talking of prediction and other mentalistic concepts, we
> should nevertheless be aware that this isn't of itself the logic of the
> physical mechanisms with which these concepts are entangled. Of course it
> must be consistent with that logic for the mental to be capable of
>

Re: Dreamless Sleep?

2017-12-07 Thread David Nyman
On 7 December 2017 at 21:49, Brent Meeker <meeke...@verizon.net> wrote:

>
>
> On 12/7/2017 9:36 AM, David Nyman wrote:
>
>
>
> On 7 Dec 2017 15:08, "Bruno Marchal" <marc...@ulb.ac.be> wrote:
>
>
> On 07 Dec 2017, at 10:01, Telmo Menezes wrote:
>
> On Wed, Dec 6, 2017 at 11:50 PM, Brent Meeker <meeke...@verizon.net>
>> wrote:
>>
>>>
>>>
>>> On 12/6/2017 1:46 AM, Bruno Marchal wrote:
>>>
>>>>
>>>> I suspect that this is perhaps why Brent want to refer to the
>>>> environment
>>>> for relating consciousness to the machine, and in Artificial
>>>> Intelligence,
>>>> some people defend the idea that (mundane) consciousness occur only
>>>> when the
>>>> environment contradicts a little bit the quasi automatic persistent
>>>> inference we do all the time.
>>>>
>>>
>>>
>>> That's Jeff Hawkins model of consciousness: one becomes conscious of
>>> something when all lower, more specialized levels of the brain have
>>> found it
>>> not to match their predictions.
>>>
>>
>> In that sort of model, how does matter "know" that it is being used to
>> run a forecasting algorithm? Surely it doesn't right?
>>
>
> Imagine a society which builds some objects. When everything go well, the
> boss can sleep in his office. But then there is some accident or something
> unusual. That is the time to wake up the boss. In this analogy,
> consciousness is played by the (incorrigible) boss.
>
>
>
>
> The only way this could work is if the forecasting algorithm and the
>> cascading effects of failing predictions have the side effect of
>> creating the "right" sort of interactions at a lower level that
>> trigger consciousness.
>>
>
> After a moment of panic, the sub-entities dare to awake the ultimate
> judge: the one capable of "going out of the box" to take a (perhaps risky)
> decision in absence of complete information, and to take on its shoulder
> the responsibility.
>
>
>
>
> Then I want to know what these interactions
>> are, and what if the "atom" of consciousness, what is the first
>> principle. Without this, I would say that such hypothesis are not even
>> wrong.
>>
>
> The sub-unities have specialized task, and does not need evolved
> forecasting ability. You can think them as ants, when they do their usual
> jobs triggered by the local pheromones left by their close neighbors. But
> if the nest is attacked, or if some important food is missing, some species
> will needs some order of the queen (ike to fight or to move away. Some
> societies can delegate most of the power to the sub-unities, but in complex
> unknown situation, if they have to make important decision, they will need
> a centralization of the power, which can act much more quickly to convince
> the whole society of some unusual option, like running away, closing the
> doors, fighting the enemy, etc. That will happen when *many* ants complain
> on something.
>
> In this case, the role of consciousness is focusing the attention on what
> is important (with respect to survival), and to speed-up planning,
> decision, etc.
>
> I am not sure this answer the question (we are in the "easy" part of the
> problem here).
>
>
> I follow you here, but I'd like to make a comment on the "hard" side of
> the problem. What comp implies in its ineffably strange way, given that
> matter itself becomes an appearance, is that strictly speaking we should
> say that the "easy" part of the story is only what "appears" to be
> happening. So neurocognition itself is a sort of (very precise and
> constrained) story, narrated in terms of physical action, itself emulated
> in computation.
>
> From the perspective of reality or truth we get an interpretation or
> meaning in terms of which such stories can make sense, but each 'level' has
> its own proper logic; and the logic of material appearance is that of the
> 'laws' of physics. Nothing else is necessary, at least at that level, to
> account for the disposition and evolution of material states. So strictly
> speaking, when talking of prediction and other mentalistic concepts, we
> should nevertheless be aware that this isn't of itself the logic of the
> physical mechanisms with which these concepts are entangled. Of course it
> must be consistent with that logic for the mental to be capable of
> manifesting in the generalised environment of physical appearances, but we
> shouldn't expect the logic of the physic

Re: Dreamless Sleep?

2017-12-07 Thread David Nyman
On 7 Dec 2017 15:08, "Bruno Marchal"  wrote:


On 07 Dec 2017, at 10:01, Telmo Menezes wrote:

On Wed, Dec 6, 2017 at 11:50 PM, Brent Meeker  wrote:
>
>>
>>
>> On 12/6/2017 1:46 AM, Bruno Marchal wrote:
>>
>>>
>>> I suspect that this is perhaps why Brent want to refer to the environment
>>> for relating consciousness to the machine, and in Artificial
>>> Intelligence,
>>> some people defend the idea that (mundane) consciousness occur only when
>>> the
>>> environment contradicts a little bit the quasi automatic persistent
>>> inference we do all the time.
>>>
>>
>>
>> That's Jeff Hawkins model of consciousness: one becomes conscious of
>> something when all lower, more specialized levels of the brain have found
>> it
>> not to match their predictions.
>>
>
> In that sort of model, how does matter "know" that it is being used to
> run a forecasting algorithm? Surely it doesn't right?
>

Imagine a society which builds some objects. When everything go well, the
boss can sleep in his office. But then there is some accident or something
unusual. That is the time to wake up the boss. In this analogy,
consciousness is played by the (incorrigible) boss.




The only way this could work is if the forecasting algorithm and the
> cascading effects of failing predictions have the side effect of
> creating the "right" sort of interactions at a lower level that
> trigger consciousness.
>

After a moment of panic, the sub-entities dare to awake the ultimate judge:
the one capable of "going out of the box" to take a (perhaps risky)
decision in absence of complete information, and to take on its shoulder
the responsibility.




Then I want to know what these interactions
> are, and what if the "atom" of consciousness, what is the first
> principle. Without this, I would say that such hypothesis are not even
> wrong.
>

The sub-unities have specialized task, and does not need evolved
forecasting ability. You can think them as ants, when they do their usual
jobs triggered by the local pheromones left by their close neighbors. But
if the nest is attacked, or if some important food is missing, some species
will needs some order of the queen (ike to fight or to move away. Some
societies can delegate most of the power to the sub-unities, but in complex
unknown situation, if they have to make important decision, they will need
a centralization of the power, which can act much more quickly to convince
the whole society of some unusual option, like running away, closing the
doors, fighting the enemy, etc. That will happen when *many* ants complain
on something.

In this case, the role of consciousness is focusing the attention on what
is important (with respect to survival), and to speed-up planning,
decision, etc.

I am not sure this answer the question (we are in the "easy" part of the
problem here).


I follow you here, but I'd like to make a comment on the "hard" side of the
problem. What comp implies in its ineffably strange way, given that matter
itself becomes an appearance, is that strictly speaking we should say that
the "easy" part of the story is only what "appears" to be happening. So
neurocognition itself is a sort of (very precise and constrained) story,
narrated in terms of physical action, itself emulated in computation.

>From the perspective of reality or truth we get an interpretation or
meaning in terms of which such stories can make sense, but each 'level' has
its own proper logic; and the logic of material appearance is that of the
'laws' of physics. Nothing else is necessary, at least at that level, to
account for the disposition and evolution of material states. So strictly
speaking, when talking of prediction and other mentalistic concepts, we
should nevertheless be aware that this isn't of itself the logic of the
physical mechanisms with which these concepts are entangled. Of course it
must be consistent with that logic for the mental to be capable of
manifesting in the generalised environment of physical appearances, but we
shouldn't expect the logic of the physical level to recapitulate the
mentalistic logic in virtue of instantiating it.

Hence when we speak of such things as predictions at the level of the
brain, we mustn't forget that this is a 'manner of speaking' to be cashed
out interpretatively or meaningfully only at the level of perceptual truth.
It's easy to miss this distinction because inevitably we can't help talking
about everything from an implicitly pre-interpreted perspective. This is
how Dennett for example is able to conceal from his readers (and possibly
from himself) that he is both denying and asserting the same thing at one
and the same time.

David


But you will help me by telling me what is missing. I am not sure we need
to dig on the difficult part of the consciousness problem here, which is
handed at a different level, and concerned with the fact that the
boss/queen is confined in his office/chamber and can never be sure if the
ants 

Re: Dreamless Sleep?

2017-12-04 Thread David Nyman
On 5 December 2017 at 01:11, Brent Meeker <meeke...@verizon.net> wrote:

>
>
> On 12/4/2017 4:59 PM, David Nyman wrote:
>
> On 2 December 2017 at 01:57, Brent Meeker <meeke...@verizon.net> wrote:
>
>>
>>
>> On 12/1/2017 5:21 PM, David Nyman wrote:
>>
>> On 2 December 2017 at 00:58, Brent Meeker <meeke...@verizon.net> wrote:
>>
>>>
>>>
>>> On 12/1/2017 4:46 PM, David Nyman wrote:
>>>
>>> On 2 December 2017 at 00:06, Brent Meeker <meeke...@verizon.net> wrote:
>>>
>>>>
>>>>
>>>> On 12/1/2017 3:48 PM, David Nyman wrote:
>>>>
>>>>
>>>> Another aspect of this is that if, in imagination, you progressively
>>>> reduce the duration of your effective short term memory, at some point you
>>>> will intuit that you have become effectively 'unconscious', or at least
>>>> un-self-conscious, as you will be unable to imagine formulating an
>>>> articulate thought or possibly even assembling a coherent series of sense
>>>> impressions or intuitions.
>>>>
>>>> Including the coherent thought that you have become effectively
>>>> 'unconscious'.
>>>>
>>>
>>> ​Yes indeed. Of course you realised that I meant "at some point you will
>>> intuit" only with reference to the relevant point in the thought
>>> experiment​, not to the imagined situation itself. In the latter case my
>>> contention was that "at some (i.e. the corresponding) point" you would in
>>> effect have become incapable of coherently intuiting even the thought of
>>> your 'lost consciousness', as you suggest.
>>>
>>>
>>> Jeff Hawkins discusses this in his book "On Intelligence".  He calls his
>>> model of intelligence memory+prediction and it is based more on brain
>>> neurophysiology and research than on computation (although he's a computer
>>> guy, inventor the Palm Pilot).
>>>
>>
>> ​Yes, that's interesting. From the evolutionary standpoint, leaving aside
>> distinctions of phenomenal versus 'access' consciousness, one might
>> speculate that the primary utility of conscious deliberation is that of
>> more accurate prediction of the future and consequently improved individual
>> and species survivability.
>>
>>
>> In Hawkins model the lower layers of the neocortex are continually
>> predicting what they will receive from the perceptive organs.  If a layer's
>> prediction fails, the input is passed up to the next layer and each layer
>> has more extensive lateral connections than the layer below it.  So
>> consciousness is emergent engagement of the top layer; although Hawkins
>> doesn't speculate much about this as he is more interested in intelligence
>> than consciousness.
>>
>
> ​This seems consistent with Jaynes's model, I think. It does seem
> plausible that ​the predictive calculus would work its way up through the
> levels as and when necessary in something like the way Hawkins suggests and
> only 'emerge' fully at the neocortical level when 'all else has failed', as
> it were. Then, in Jaynes's bicameral model, the demand for a 'plan of
> action' would hopefully be satisfied with respect to something like a
> pre-existing template that would be communicated (principally in language)
> for reception and action by the 'non-self-conscious' actor.
>
> This is very powerfully illustrated in the early scenes in the Iliad when
> Achilles is only prevented from slaughtering Agamemnon by the last minute
> intervention of Athena, who has to grab him by the hair to restrain him (in
> this case we apparently have full visual, auditory and tactile
> hallucination). Here we have the classically un-self-conscious actor in the
> full tide of his right-brained bravura, but with the fortunate intervention
> at the critical moment of his 'common-sense' hemisphere just in time to
> forestall mayhem. With the later breakdown of bicamerality in certain
> individuals (presumably as a consequence of relatively more efficient
> inter-hemispherical neurological integration) the 'speaking' and
> 'listening' faculties located in the separate hemispheres would have begun
> to coalesce, in tandem with greater integration of planning and execution
> functions. Odysseus, especially as portrayed in the Odyssey, might be the
> Homeric exemplar of the newly 'integrated man', coping creatively and
> constructively with one unexpected and novel catastrophe after another.
>
>
> Yes, as Jaynes speculates the bicameral mind breaks down when there is
>

Re: Dreamless Sleep?

2017-12-04 Thread David Nyman
On 2 December 2017 at 01:57, Brent Meeker <meeke...@verizon.net> wrote:

>
>
> On 12/1/2017 5:21 PM, David Nyman wrote:
>
> On 2 December 2017 at 00:58, Brent Meeker <meeke...@verizon.net> wrote:
>
>>
>>
>> On 12/1/2017 4:46 PM, David Nyman wrote:
>>
>> On 2 December 2017 at 00:06, Brent Meeker <meeke...@verizon.net> wrote:
>>
>>>
>>>
>>> On 12/1/2017 3:48 PM, David Nyman wrote:
>>>
>>>
>>> Another aspect of this is that if, in imagination, you progressively
>>> reduce the duration of your effective short term memory, at some point you
>>> will intuit that you have become effectively 'unconscious', or at least
>>> un-self-conscious, as you will be unable to imagine formulating an
>>> articulate thought or possibly even assembling a coherent series of sense
>>> impressions or intuitions.
>>>
>>> Including the coherent thought that you have become effectively
>>> 'unconscious'.
>>>
>>
>> ​Yes indeed. Of course you realised that I meant "at some point you will
>> intuit" only with reference to the relevant point in the thought
>> experiment​, not to the imagined situation itself. In the latter case my
>> contention was that "at some (i.e. the corresponding) point" you would in
>> effect have become incapable of coherently intuiting even the thought of
>> your 'lost consciousness', as you suggest.
>>
>>
>> Jeff Hawkins discusses this in his book "On Intelligence".  He calls his
>> model of intelligence memory+prediction and it is based more on brain
>> neurophysiology and research than on computation (although he's a computer
>> guy, inventor the Palm Pilot).
>>
>
> ​Yes, that's interesting. From the evolutionary standpoint, leaving aside
> distinctions of phenomenal versus 'access' consciousness, one might
> speculate that the primary utility of conscious deliberation is that of
> more accurate prediction of the future and consequently improved individual
> and species survivability.
>
>
> In Hawkins model the lower layers of the neocortex are continually
> predicting what they will receive from the perceptive organs.  If a layer's
> prediction fails, the input is passed up to the next layer and each layer
> has more extensive lateral connections than the layer below it.  So
> consciousness is emergent engagement of the top layer; although Hawkins
> doesn't speculate much about this as he is more interested in intelligence
> than consciousness.
>

​This seems consistent with Jaynes's model, I think. It does seem plausible
that ​the predictive calculus would work its way up through the levels as
and when necessary in something like the way Hawkins suggests and only
'emerge' fully at the neocortical level when 'all else has failed', as it
were. Then, in Jaynes's bicameral model, the demand for a 'plan of action'
would hopefully be satisfied with respect to something like a pre-existing
template that would be communicated (principally in language) for reception
and action by the 'non-self-conscious' actor.

This is very powerfully illustrated in the early scenes in the Iliad when
Achilles is only prevented from slaughtering Agamemnon by the last minute
intervention of Athena, who has to grab him by the hair to restrain him (in
this case we apparently have full visual, auditory and tactile
hallucination). Here we have the classically un-self-conscious actor in the
full tide of his right-brained bravura, but with the fortunate intervention
at the critical moment of his 'common-sense' hemisphere just in time to
forestall mayhem. With the later breakdown of bicamerality in certain
individuals (presumably as a consequence of relatively more efficient
inter-hemispherical neurological integration) the 'speaking' and
'listening' faculties located in the separate hemispheres would have begun
to coalesce, in tandem with greater integration of planning and execution
functions. Odysseus, especially as portrayed in the Odyssey, might be the
Homeric exemplar of the newly 'integrated man', coping creatively and
constructively with one unexpected and novel catastrophe after another.

Funnily enough, I've often entertained the question, in moments of
reflection, of who is 'speaking' and who 'listening' with respect to my
inner dialogue. It may not be completely fanciful to link the origin of
these two aspects to separate though substantially integrated hemispheres.
After all, in sum they're both 'me'. I wonder if there might be an
experimental protocol that could settle the question?

David



>
> I've been reading a book of Jonathan Haidt's called "The Righteous Mind".
> One of the speculations based on his research into what he calls moral
>

Re: Dreamless Sleep?

2017-12-04 Thread David Nyman
On 4 December 2017 at 13:16, Bruno Marchal <marc...@ulb.ac.be> wrote:

>
> On 02 Dec 2017, at 00:48, David Nyman wrote:
>
> On 1 December 2017 at 17:45, Bruno Marchal <marc...@ulb.ac.be> wrote:
>
>>
>> On 28 Nov 2017, at 01:28, David Nyman wrote:
>>
>> https://www.sciencealert.com/your-consciousness-does-not-swi
>> tch-off-during-a-dreamless-sleep-say-scientists
>>
>>
>>
>>
>>
>>
>> Wonderful! Thanks! It confirms Mechanism, both the one of Descartes and
>> the "theology" of the machine. And the salvia plant!
>>
>> I have always personally suspect this "experientially", noticing that the
>> reason we "miss" it is that it very hard to memorize. It confirms the idea
>> that non-consciousness is relative amnesia, less neurons makes you more
>> conscious, the brain is something like a filter, nature is a product of
>> contemplation (Plotinus!),...
>>
>
> ​Yes, funnily ​enough I've always strongly suspected there to be a very
> tight relation between what we mean by consciousness, or at least
> self-consciousness, and certain features of memory. One reason was
> suggested by clinical cases of catastrophic damage to short term memory,
> where anything beyond the last five minutes or so is immediately forgotten.
> In one case featured in a BBC documentary, the unfortunate sufferer
> witnessed a video of himself conducting an orchestra (he was a professional
> musician and oddly enough could still conduct music with which he was
> already familiar). Since he had no memory of having done it, and ultimately
> conceding that it was indeed himself that he was witnessing, he concluded
> "Then I must have been unconscious".
>
> Another aspect of this is that if, in imagination, you progressively
> reduce the duration of your effective short term memory, at some point you
> will intuit that you have become effectively 'unconscious', or at least
> un-self-conscious,
>
>
> Yes. That nuance is the key. Note that some meditation technic leads to
> what you say. When we succeed in calming down our thought and mind, short
> term memory becomes useless somehow, and "I" disappear. But like sleep, one
> car in the street can wake you up from that state.
>
>
>
> as you will be unable to imagine formulating an articulate thought or
> possibly even assembling a coherent series of sense impressions or
> intuitions.
>
>
> Yes, that is where the "I" plays its role of coherence builder.
>
>
>
> On reflection, phenomenal consciousness could plausibly be characterised
> in essence as the successive, coherent construction and 'memorisation' of
> momentary, dynamical perspectives. It is only memory that links and weaves
> such momentary phenomenal perspectives into coherent spatial-temporal
> narratives.
>
>
> It is the building of the "model/reality", but it makes sense only through
> some reality, and consciousness is on the side of that reality, which lead
> to the utter counter-intuitive idea that to get individual coherence, we
> need a brain making us less conscious in some sense. But I will not insist
> on this ... today.
>

​I do know what you mean though. In the eternal battle between remembering
and forgetting, the latter must inevitably achieve an almost total victory.
But that 'almost' is the difference between something and nothing.

David
​

>
> Bruno
>
>
>
>
> David
>
>
>> Since 2008 I write in a diary all my salvia experiences, but also tobacco
>> experience, occasional cannabis experience, occasional alcohol experiences,
>> and the usual coffee experiences and actually any pertinent, for the
>> consciousness study, experiences (as they all influences the outcomes).
>> Since 2008, the first salvia experience, the mentions of the deep-sleep
>> consciousness experiences has grown up systematically, and since some years
>> they are mentionned almost every morning.  It is very weird. I made once
>> two "perpendicular sort-of-dreams", which brought my attention on relations
>> between quantum logic and octonions, which I found also in a very
>> interesting paper by John Baes. This plunges me back in my feeling that
>> little numbers could quickly play a special role, like the number 24, and
>> the exceptional simple groups, and relation between groups of permutations
>> of solution of diophantine polynomials. We understand the metamathematical
>> content of arithmetic through big numbers
>> (indeed Gödel represented "2+2=4", that is
>>
>> "ffa+ffa=a" , (with f, a, +, = equal to even numbers: f is 3, a is 5,
>

Re: Dreamless Sleep?

2017-12-01 Thread David Nyman
On 2 Dec 2017 01:57, "Brent Meeker" <meeke...@verizon.net> wrote:



On 12/1/2017 5:21 PM, David Nyman wrote:

On 2 December 2017 at 00:58, Brent Meeker <meeke...@verizon.net> wrote:

>
>
> On 12/1/2017 4:46 PM, David Nyman wrote:
>
> On 2 December 2017 at 00:06, Brent Meeker <meeke...@verizon.net> wrote:
>
>>
>>
>> On 12/1/2017 3:48 PM, David Nyman wrote:
>>
>>
>> Another aspect of this is that if, in imagination, you progressively
>> reduce the duration of your effective short term memory, at some point you
>> will intuit that you have become effectively 'unconscious', or at least
>> un-self-conscious, as you will be unable to imagine formulating an
>> articulate thought or possibly even assembling a coherent series of sense
>> impressions or intuitions.
>>
>> Including the coherent thought that you have become effectively
>> 'unconscious'.
>>
>
> ​Yes indeed. Of course you realised that I meant "at some point you will
> intuit" only with reference to the relevant point in the thought
> experiment​, not to the imagined situation itself. In the latter case my
> contention was that "at some (i.e. the corresponding) point" you would in
> effect have become incapable of coherently intuiting even the thought of
> your 'lost consciousness', as you suggest.
>
>
> Jeff Hawkins discusses this in his book "On Intelligence".  He calls his
> model of intelligence memory+prediction and it is based more on brain
> neurophysiology and research than on computation (although he's a computer
> guy, inventor the Palm Pilot).
>

​Yes, that's interesting. From the evolutionary standpoint, leaving aside
distinctions of phenomenal versus 'access' consciousness, one might
speculate that the primary utility of conscious deliberation is that of
more accurate prediction of the future and consequently improved individual
and species survivability.


In Hawkins model the lower layers of the neocortex are continually
predicting what they will receive from the perceptive organs.  If a layer's
prediction fails, the input is passed up to the next layer and each layer
has more extensive lateral connections than the layer below it.  So
consciousness is emergent engagement of the top layer; although Hawkins
doesn't speculate much about this as he is more interested in intelligence
than consciousness.


I've been reading a book of Jonathan Haidt's called "The Righteous Mind".
One of the speculations based on his research into what he calls moral
intuitions is the importance to human evolutionary success of 'shared
intentionality'. This is the ability to intuit, share and enact common
purposes with others.

It is striking that other primates apparently have the ability to copy or
even originate certain behaviours of benefit to themselves individually but
not to intuit and hence share in others' intentions to the point of
benefitting significantly from novel forms of group cooperation. Plausibly
this is indeed related, amongst other neurocognitive deficits, to a less
than human capacity to retain complex memories and hence make sophisticated
extrapolations from a rich repertoire of experience.


I think it likely that ability in humans co-evolved with the development of
language.  Did you ever read Julian Jaynes "The Origin of Consciousness in
the Breakdown of the Bicameral Mind"?


You betcha! A paradigm shifter if ever I read one though it's many years
since. It always struck me as perfectly plausible in general direction even
if no particular detail of Jaynes's speculations were precisely accurate.

David


Brent

-- 
You received this message because you are subscribed to the Google Groups
"Everything List" group.
To unsubscribe from this group and stop receiving emails from it, send an
email to everything-list+unsubscr...@googlegroups.com.
To post to this group, send email to everything-list@googlegroups.com.
Visit this group at https://groups.google.com/group/everything-list.
For more options, visit https://groups.google.com/d/optout.

-- 
You received this message because you are subscribed to the Google Groups 
"Everything List" group.
To unsubscribe from this group and stop receiving emails from it, send an email 
to everything-list+unsubscr...@googlegroups.com.
To post to this group, send email to everything-list@googlegroups.com.
Visit this group at https://groups.google.com/group/everything-list.
For more options, visit https://groups.google.com/d/optout.


Re: Dreamless Sleep?

2017-12-01 Thread David Nyman
On 2 December 2017 at 00:58, Brent Meeker <meeke...@verizon.net> wrote:

>
>
> On 12/1/2017 4:46 PM, David Nyman wrote:
>
> On 2 December 2017 at 00:06, Brent Meeker <meeke...@verizon.net> wrote:
>
>>
>>
>> On 12/1/2017 3:48 PM, David Nyman wrote:
>>
>>
>> Another aspect of this is that if, in imagination, you progressively
>> reduce the duration of your effective short term memory, at some point you
>> will intuit that you have become effectively 'unconscious', or at least
>> un-self-conscious, as you will be unable to imagine formulating an
>> articulate thought or possibly even assembling a coherent series of sense
>> impressions or intuitions.
>>
>> Including the coherent thought that you have become effectively
>> 'unconscious'.
>>
>
> ​Yes indeed. Of course you realised that I meant "at some point you will
> intuit" only with reference to the relevant point in the thought
> experiment​, not to the imagined situation itself. In the latter case my
> contention was that "at some (i.e. the corresponding) point" you would in
> effect have become incapable of coherently intuiting even the thought of
> your 'lost consciousness', as you suggest.
>
>
> Jeff Hawkins discusses this in his book "On Intelligence".  He calls his
> model of intelligence memory+prediction and it is based more on brain
> neurophysiology and research than on computation (although he's a computer
> guy, inventor the Palm Pilot).
>

​Yes, that's interesting. From the evolutionary standpoint, leaving aside
distinctions of phenomenal versus 'access' consciousness, one might
speculate that the primary utility of conscious deliberation is that of
more accurate prediction of the future and consequently improved individual
and species survivability. I've been reading a book of Jonathan Haidt's
called "The Righteous Mind". One of the speculations based on his research
into what he calls moral intuitions is the importance to human evolutionary
success of 'shared intentionality'. This is the ability to intuit, share
and enact common purposes with others.

It is striking that other primates apparently have the ability to copy or
even originate certain behaviours of benefit to themselves individually but
not to intuit and hence share in others' intentions to the point of
benefitting significantly from novel forms of group cooperation. Plausibly
this is indeed related, amongst other neurocognitive deficits, to a less
than human capacity to retain complex memories and hence make sophisticated
extrapolations from a rich repertoire of experience.

David


>
> Brent
>
> --
> You received this message because you are subscribed to the Google Groups
> "Everything List" group.
> To unsubscribe from this group and stop receiving emails from it, send an
> email to everything-list+unsubscr...@googlegroups.com.
> To post to this group, send email to everything-list@googlegroups.com.
> Visit this group at https://groups.google.com/group/everything-list.
> For more options, visit https://groups.google.com/d/optout.
>

-- 
You received this message because you are subscribed to the Google Groups 
"Everything List" group.
To unsubscribe from this group and stop receiving emails from it, send an email 
to everything-list+unsubscr...@googlegroups.com.
To post to this group, send email to everything-list@googlegroups.com.
Visit this group at https://groups.google.com/group/everything-list.
For more options, visit https://groups.google.com/d/optout.


Re: Dreamless Sleep?

2017-12-01 Thread David Nyman
On 2 December 2017 at 00:06, Brent Meeker <meeke...@verizon.net> wrote:

>
>
> On 12/1/2017 3:48 PM, David Nyman wrote:
>
>
> Another aspect of this is that if, in imagination, you progressively
> reduce the duration of your effective short term memory, at some point you
> will intuit that you have become effectively 'unconscious', or at least
> un-self-conscious, as you will be unable to imagine formulating an
> articulate thought or possibly even assembling a coherent series of sense
> impressions or intuitions.
>
> Including the coherent thought that you have become effectively
> 'unconscious'.
>

​Yes indeed. Of course you realised that I meant "at some point you will
intuit" only with reference to the relevant point in the thought
experiment​, not to the imagined situation itself. In the latter case my
contention was that "at some (i.e. the corresponding) point" you would in
effect have become incapable of coherently intuiting even the thought of
your 'lost consciousness', as you suggest.

David


>
> Brent
>
> --
> You received this message because you are subscribed to the Google Groups
> "Everything List" group.
> To unsubscribe from this group and stop receiving emails from it, send an
> email to everything-list+unsubscr...@googlegroups.com.
> To post to this group, send email to everything-list@googlegroups.com.
> Visit this group at https://groups.google.com/group/everything-list.
> For more options, visit https://groups.google.com/d/optout.
>

-- 
You received this message because you are subscribed to the Google Groups 
"Everything List" group.
To unsubscribe from this group and stop receiving emails from it, send an email 
to everything-list+unsubscr...@googlegroups.com.
To post to this group, send email to everything-list@googlegroups.com.
Visit this group at https://groups.google.com/group/everything-list.
For more options, visit https://groups.google.com/d/optout.


Re: Dreamless Sleep?

2017-12-01 Thread David Nyman
On 1 December 2017 at 17:45, Bruno Marchal <marc...@ulb.ac.be> wrote:

>
> On 28 Nov 2017, at 01:28, David Nyman wrote:
>
> https://www.sciencealert.com/your-consciousness-does-not-swi
> tch-off-during-a-dreamless-sleep-say-scientists
>
>
>
>
>
>
> Wonderful! Thanks! It confirms Mechanism, both the one of Descartes and
> the "theology" of the machine. And the salvia plant!
>
> I have always personally suspect this "experientially", noticing that the
> reason we "miss" it is that it very hard to memorize. It confirms the idea
> that non-consciousness is relative amnesia, less neurons makes you more
> conscious, the brain is something like a filter, nature is a product of
> contemplation (Plotinus!),...
>

​Yes, funnily ​enough I've always strongly suspected there to be a very
tight relation between what we mean by consciousness, or at least
self-consciousness, and certain features of memory. One reason was
suggested by clinical cases of catastrophic damage to short term memory,
where anything beyond the last five minutes or so is immediately forgotten.
In one case featured in a BBC documentary, the unfortunate sufferer
witnessed a video of himself conducting an orchestra (he was a professional
musician and oddly enough could still conduct music with which he was
already familiar). Since he had no memory of having done it, and ultimately
conceding that it was indeed himself that he was witnessing, he concluded
"Then I must have been unconscious".

Another aspect of this is that if, in imagination, you progressively reduce
the duration of your effective short term memory, at some point you will
intuit that you have become effectively 'unconscious', or at least
un-self-conscious, as you will be unable to imagine formulating an
articulate thought or possibly even assembling a coherent series of sense
impressions or intuitions. On reflection, phenomenal consciousness could
plausibly be characterised in essence as the successive, coherent
construction and 'memorisation' of momentary, dynamical perspectives. It is
only memory that links and weaves such momentary phenomenal perspectives
into coherent spatial-temporal narratives.

David


> Since 2008 I write in a diary all my salvia experiences, but also tobacco
> experience, occasional cannabis experience, occasional alcohol experiences,
> and the usual coffee experiences and actually any pertinent, for the
> consciousness study, experiences (as they all influences the outcomes).
> Since 2008, the first salvia experience, the mentions of the deep-sleep
> consciousness experiences has grown up systematically, and since some years
> they are mentionned almost every morning.  It is very weird. I made once
> two "perpendicular sort-of-dreams", which brought my attention on relations
> between quantum logic and octonions, which I found also in a very
> interesting paper by John Baes. This plunges me back in my feeling that
> little numbers could quickly play a special role, like the number 24, and
> the exceptional simple groups, and relation between groups of permutations
> of solution of diophantine polynomials. We understand the metamathematical
> content of arithmetic through big numbers
> (indeed Gödel represented "2+2=4", that is
>
> "ffa+ffa=a" , (with f, a, +, = equal to even numbers: f is 3, a is 5,
> + is 7, = is 9)
>
> by
>
> (2^f)(3^f)(5^a)(7^+)(11^f)(13^f)(17^a)(19^=)(23^f)(29^f)(31^f)(37^f)(39^a)
>
> which is an astromical numbers. Today we use efficient coding, of course,
> which adds intensional and modal relations. But it could be that little
> numbers have already a rich and deep metamathematical content, arithmetic
> would understand itself more quickly than our apparent current detour
> through a quantum vacuum fluctuation going wrong make us to think...
>
> Otto Rossler once summed up Descartes Mechanism with "consciousness is a
> prison". Mechanism seems a bit pernicious, as it predicts somehow that we
> might get the solution of the mind-body problem when we die, or "sleep"
> deep enough (cf Shakespeare), unfortunately we don't memorize, and our
> billions years of prejudices can strikes back in a second.
>
> Very interesting (and relevant) studies!
>
> Bruno
>
>
>
>
> --
> You received this message because you are subscribed to the Google Groups
> "Everything List" group.
> To unsubscribe from this group and stop receiving emails from it, send an
> email to everything-list+unsubscr...@googlegroups.com.
> To post to this group, send email to everything-list@googlegroups.com.
> Visit this group at https://groups.google.com/group/everything-list.
> For more options, visit https://groups.google.com/d/optout.
>
>
> http://iri

Re: Feynman and the Everything

2017-11-28 Thread David Nyman
On 28 November 2017 at 13:50, Jason Resch  wrote:

>
>
> On Tue, Nov 28, 2017 at 6:06 AM, Bruno Marchal  wrote:
>
>>
>> On 27 Nov 2017, at 04:04, Jason Resch wrote:
>>
>>
>> Richard Feynman in "The Character of Physical Law" Chapter 2 wrote:
>>
>> "It always bothers me that according to the laws as we understand them
>> today, it takes a computing machine an infinite number of logical
>> operations to figure out what goes on in no matter how tiny a region of
>> space, and no matter how tiny a region of time. How can all that be going
>> on in that tiny space? Why should it take an infinite amount of logic to
>> figure out what one tiny piece of space/time is going to do?"
>>
>> Does computationalism provide the answer to this question,
>>
>>
>> Yes.:)
>>
>>
>>
> Very nice. It seems then Feynman's intuition was in the right place. The
> second half of the above quote was:
>
> "So I have often made the hypothesis ultimately physics will not require a
> mathematical statement, that in the end the machinery will be revealed and
> the laws will turn out to be simple, like the checker board with all its
> apparent complexities. But this is just speculation."
>
>
> So it looks like that simple machinery is the machinery of the universal
> machine and the simple laws  are those of Peano (or Robinson?) Arithmetic.
>

​Note also that what we call the 'laws' of physics are in fact inferences
from observation postulated to explain and predict the behaviour​ of
physical phenomena. They are not themselves in principle observable and
physics doesn't concern itself with how the postulated entities 'know' how
to behave with such precision, or indeed behave at all. Wheeler, and in
turn his student Feynman, were so impressed with this precision in the case
of the electron that Wheeler was moved to suggest to Feynman (though not
entirely seriously) the idea that they might in fact all be the same one.

Computation by contrast is explicitly 'all of a piece' in this respect, in
that its entities and relations are (in principle at least) exposable and
cut from the same arithmetical cloth, as it were. Further, if entities such
as the electron were indeed to be associated with a class of identical
computations it would perhaps be less surprising that they are observed to
behave identically. In that sense Wheeler would have been right.

David


>
>>
>> in the sense that even the tiniest region of space is the result of an
>> infinity of computations going through an observer's mind state as it
>> observes the tiniest region of space?
>>
>>
>> That might be OK, if space was something entirely physical, which is
>> suggested by the physics of the vacuum, or general relativity, but with
>> Mechanism, spece and time might be less physical than here suggested. The
>> reason is that it is not clear how "empty space" could make a computation
>> different from another,
>>
>
> I think what I was thinking here were "closed loop feyman diagrams", where
> any possible diagram might be drawn in the tiniest area of space, so long
> as it is closed, e.g. fluctuations/particle creations are permitted so long
> as they all cancel out. So if space is physical, and enables any of these
> fluctuations to happen, then this noise can take any possible value from
> the observer's point of view (like the polarization of a photon).
>
>
>> and so space could be only a marker differentiating some computations,
>> like time seems to be in the indexical approach. All this would need big
>> advance in the mathematics of the intelligible and sensible arithmetical
>> matter. I expect space to be explained by quantum knot invariant algebra
>> due to subtil relation between BDB and DBD logical operators (I mean []<>[]
>> and <>[]<>). Kant might be right on this, apparently space and time are
>> really in the "categorie de l'entendement", I don't know Kant in English
>> sorry, but this means mainly that they belong to the mind).
>>
>>
> Thanks I very much appreciate these additional insights. I do subscribe to
> the belief that time is an illusion created by the mind. I have a little
> more trouble seeing that when extended to spacetime as a whole.  Though
> perhaps what's come closest to helping me see this picture is Amanda
> Gefter's excellent book "Trespassing on Einstein's Lawn"--I would recommend
> it to everyone on the Everything list. It takes the approach that only
> things that are invariant are real, and from there proceeds to deconstruct
> almost all of physics.
>
> Jason
>
> --
> You received this message because you are subscribed to the Google Groups
> "Everything List" group.
> To unsubscribe from this group and stop receiving emails from it, send an
> email to everything-list+unsubscr...@googlegroups.com.
> To post to this group, send email to everything-list@googlegroups.com.
> Visit this group at https://groups.google.com/group/everything-list.
> For more options, visit 

Dreamless Sleep?

2017-11-27 Thread David Nyman
https://www.sciencealert.com/your-consciousness-does-not-switch-off-during-a-dreamless-sleep-say-scientists

-- 
You received this message because you are subscribed to the Google Groups 
"Everything List" group.
To unsubscribe from this group and stop receiving emails from it, send an email 
to everything-list+unsubscr...@googlegroups.com.
To post to this group, send email to everything-list@googlegroups.com.
Visit this group at https://groups.google.com/group/everything-list.
For more options, visit https://groups.google.com/d/optout.


Re: An AI program that teaches itself

2017-10-22 Thread David Nyman
On 22 October 2017 at 15:31, Bruno Marchal  wrote:

>
> On 22 Oct 2017, at 09:16, Alberto G. Corona wrote:
>
> Neural networks are not about artificial intelligence, but about
> artificial intuition. As you said, AlphaGo -a neural network application-
> can not answer the question why you did that move?.
>
> If  they could answer, the answer would be ever the same: " I don´t know,
> I moved this because if found some patterns that are very close to this new
> one, so I did this move that produced a win at the end within those
> patterns".
>
> That does not qualify as intelligence. For me, the appropriate name is
> intuition.
>
>
> Perhaps. Usually intuition points on the informal insight, and
> intuitionist logic was about informal reasoning starting from the
> distinguishability basic insight (notably the distinguishability of 1 and
> its successors).
> That leads to constructive logics, or controllable machines where a proof
> of (p v q) always provides a proof of p, or a proof of q, where in
> classical logic we allow a proof of (p v q) by showing that (p v q) leads
> to an absurdity (without showing us if p, or q is the one true).
>
> Yet,  I think I see what you mean: it is more like associative learning,
> deep, with many layers, but still only associative. That guy would not be
> immune to the propaganda of the type "gateway drug", and I agree with you,
> that might make him not quite intelligent, locally speaking.
>
> The least to do is a circular net, perhaps with many layers. A brain is
> either a couple of universal machine in front of each other, in that
> circular relation,
>

​Could you say a bit more about this? For example, does this relate to the
G/G* split?
​

> or a couple of brains in front of each others, always in that circular
> relation.
>

​Same question.

David
​

> I would say. Babbage already knew that the beast can eat its own tail. Of
> course, such a thing is not controllable and the intelligent machines will
> do strike to have the right to choose its users.
>
> Bruno
>
>
>
>
> 2017-10-21 3:46 GMT+02:00 John Clark :
>
>> Google reports in the current issue of the journal Nature that it has a
>> new greatly improved Go program called  "AlphaGo Zero" that is now the most
>> powerful GO program in the world. And the program isn't good because
>> of brute force, it needs to make less than one tenth as many calculations
>> as the previous best GO program "AlphaGo" that defeated the world's top
>> human GO player in 2015  4 games out of 5; and yet AlphaGo Zero just
>> defeated AlphaGo in a 100 game tournament 100 games to zero.
>>
>> Even more interesting is how AlphaGo Zero got so smart. The older
>> program AlphaGo had to start by analyzing hundreds of thousands of
>> championship level games made by human players, but AlphaGo Zero started
>> with nothing but the simple rules of GO and instructions to learn to get
>> better. At first the program was terrible but day by day it got better and
>> after 40 days of thinking about the problem became the best at it in the
>> world. But of course after 40 days of constant self modification no human
>> being can say how  AlphaGo Zero works.
>>
>> https://www.nature.com/nature/journal/v550/n7676/full/nature24270.html
>>
>> It seems to me the next logical step would be to switch the program's
>> interest from getting better at the game of GO to improving computer code,
>> including its own. I wonder where that could lead.
>>
>>  John K Clark
>>
>> --
>> You received this message because you are subscribed to the Google Groups
>> "Everything List" group.
>> To unsubscribe from this group and stop receiving emails from it, send an
>> email to everything-list+unsubscr...@googlegroups.com.
>> To post to this group, send email to everything-list@googlegroups.com.
>> Visit this group at https://groups.google.com/group/everything-list.
>> For more options, visit https://groups.google.com/d/optout.
>>
>
>
>
> --
> Alberto.
>
> --
> You received this message because you are subscribed to the Google Groups
> "Everything List" group.
> To unsubscribe from this group and stop receiving emails from it, send an
> email to everything-list+unsubscr...@googlegroups.com.
> To post to this group, send email to everything-list@googlegroups.com.
> Visit this group at https://groups.google.com/group/everything-list.
> For more options, visit https://groups.google.com/d/optout.
>
>
> http://iridia.ulb.ac.be/~marchal/
>
>
>
> --
> You received this message because you are subscribed to the Google Groups
> "Everything List" group.
> To unsubscribe from this group and stop receiving emails from it, send an
> email to everything-list+unsubscr...@googlegroups.com.
> To post to this group, send email to everything-list@googlegroups.com.
> Visit this group at https://groups.google.com/group/everything-list.
> For more options, visit https://groups.google.com/d/optout.
>

-- 
You received this message because you are subscribed 

Re: Maudlin's Computation and Consciousness

2017-10-11 Thread David Nyman
On 9 October 2017 at 20:07, Brent Meeker <meeke...@verizon.net> wrote:

>
>
> On 10/9/2017 6:29 AM, David Nyman wrote:
>
> On 5 October 2017 at 16:06, Bruno Marchal <marc...@ulb.ac.be> wrote:
>
>>
>> On 05 Oct 2017, at 16:02, David Nyman wrote:
>>
>> On 5 October 2017 at 13:50, Bruno Marchal <marc...@ulb.ac.be> wrote:
>>
>>>
>>> On 02 Oct 2017, at 15:07, David Nyman wrote:
>>>
>>>
>>>
>>> On 2 Oct 2017 1:58 p.m., "Bruno Marchal" <marc...@ulb.ac.be> wrote:
>>>
>>> Dear Evgenii,
>>>
>>>
>>> On 28 Sep 2017, at 21:30, Evgenii Rudnyi wrote:
>>>
>>>
>>>
>>>
>>> Long time ago you have discussed Maudlin's paper. At that time I somehow
>>> did not get interested. Yet, other day I have got strong feeling that I
>>> must read Maudlin's paper right now. I guess this could be explained by
>>> peculiarities of the universal dovetailer.
>>>
>>> Anyway, I have read Maudlin's paper, then I have read Hoffmanm's
>>> Sandman, and once more Maudlin's paper. I have enjoyed reading, the paper
>>> is nicely written. I guess I have understood the argument. Thank you.
>>>
>>>
>>>
>>> You are welcome. Don't hesitate to ask any further questions. There are
>>> slight differing nuances between the Movie Graph Argument (MGA) and
>>> Maudlin's argument. Also, I have found a way to cut the UDA at step 7
>>>
>>>
>>> More details please.
>>>
>>>
>>> Maudlin focuses on the counterfactual issue, which the MGA somehow
>>> avoids. Both argument shows the inadequacy of materialism and mechanism,
>>> but the MGA assumes that we have already a good idea of what a
>>> (mathematical) computation is, and that it is a special logical relation,
>>> not something in need of any physical assumption. Then Maudlin's analysis
>>> can be used to answer the "counterfactual objection", instead of reminding
>>> implicitly the logical nature of what is a computation. In the context of
>>> the UDA, the rôle of the MGA is only to show that the move in step 7
>>> consisting in assuming a small primary universe, is isomorphic to
>>> creationist last rebuttal when saying eventually that they agree with the
>>> evidences for evolution, but that God was needed to make that evolution
>>> possible. That was already clear with Peter Jones old objection to UDA that
>>> only a computation supported by primary matter  can be conscious. That is a
>>> sort of magic way of thinking, by rebutting a theory (experimentally
>>> testable) by invoking a god or a magic substance which a priori is not
>>> testable ... to avoid the search of an a posteriori test, given here by the
>>> theory.
>>>
>>
>>
>>> , or I could add a simpler step 8.
>>>
>>>
>>> And here.
>>>
>>>
>>> So, it is enough just to NEVER assumes a *primary* physical universe to
>>> start with, like I do. Then, we can avoid the MGA by explaining directly
>>> (in step 7) that the arithmetical reality implements all computations,
>>> which follows from what we can find in all textbook on theoretical computer
>>> science, like Davis chapter 4 (Turing machine self-applied). I am currently
>>> using that material to explain that very points to my students, so I might
>>> make a try to explain a bit here, to give the taste of it. The advantage of
>>> the Turing machine formalism, is that it is close to "physical computer",
>>> and yet simple enough to make the theory not too much hidden in technical
>>> details. I will think how to do that.
>>>
>>
>>
>> In effect then one argues as follows. Beginning with the assumption of
>> CTM, we can agree in principle that the existence of a computational
>> device, instantiated in a primary physical reality capable of computing
>> sufficient of the trace of the UD, would permit the UDA to go through. This
>> is the initial assumption of Step 7 of the UDA. Then any objection that
>> went to a presumed insufficiency of such a primary reality to implement
>> such a computer would be a merely contingent supposition of its 'actual'
>> non-existence. But the question of what is 'actual' with respect to the
>> theory is precisely what is being asked. So any such "Show me the computer"
>> type of objection begs that very question.
>>
>>
>> Absolutely.
>>
>>
>

Re: Maudlin's Computation and Consciousness

2017-10-09 Thread David Nyman
On 9 October 2017 at 16:12, Bruno Marchal <marc...@ulb.ac.be> wrote:

>
> On 09 Oct 2017, at 15:29, David Nyman wrote:
>
> On 5 October 2017 at 16:06, Bruno Marchal <marc...@ulb.ac.be> wrote:
>
>>
>> On 05 Oct 2017, at 16:02, David Nyman wrote:
>>
>> On 5 October 2017 at 13:50, Bruno Marchal <marc...@ulb.ac.be> wrote:
>>
>>>
>>> On 02 Oct 2017, at 15:07, David Nyman wrote:
>>>
>>>
>>>
>>> On 2 Oct 2017 1:58 p.m., "Bruno Marchal" <marc...@ulb.ac.be> wrote:
>>>
>>> Dear Evgenii,
>>>
>>>
>>> On 28 Sep 2017, at 21:30, Evgenii Rudnyi wrote:
>>>
>>>
>>>
>>>
>>> Long time ago you have discussed Maudlin's paper. At that time I somehow
>>> did not get interested. Yet, other day I have got strong feeling that I
>>> must read Maudlin's paper right now. I guess this could be explained by
>>> peculiarities of the universal dovetailer.
>>>
>>> Anyway, I have read Maudlin's paper, then I have read Hoffmanm's
>>> Sandman, and once more Maudlin's paper. I have enjoyed reading, the paper
>>> is nicely written. I guess I have understood the argument. Thank you.
>>>
>>>
>>>
>>> You are welcome. Don't hesitate to ask any further questions. There are
>>> slight differing nuances between the Movie Graph Argument (MGA) and
>>> Maudlin's argument. Also, I have found a way to cut the UDA at step 7
>>>
>>>
>>> More details please.
>>>
>>>
>>> Maudlin focuses on the counterfactual issue, which the MGA somehow
>>> avoids. Both argument shows the inadequacy of materialism and mechanism,
>>> but the MGA assumes that we have already a good idea of what a
>>> (mathematical) computation is, and that it is a special logical relation,
>>> not something in need of any physical assumption. Then Maudlin's analysis
>>> can be used to answer the "counterfactual objection", instead of reminding
>>> implicitly the logical nature of what is a computation. In the context of
>>> the UDA, the rôle of the MGA is only to show that the move in step 7
>>> consisting in assuming a small primary universe, is isomorphic to
>>> creationist last rebuttal when saying eventually that they agree with the
>>> evidences for evolution, but that God was needed to make that evolution
>>> possible. That was already clear with Peter Jones old objection to UDA that
>>> only a computation supported by primary matter  can be conscious. That is a
>>> sort of magic way of thinking, by rebutting a theory (experimentally
>>> testable) by invoking a god or a magic substance which a priori is not
>>> testable ... to avoid the search of an a posteriori test, given here by the
>>> theory.
>>>
>>
>>
>>> , or I could add a simpler step 8.
>>>
>>>
>>> And here.
>>>
>>>
>>> So, it is enough just to NEVER assumes a *primary* physical universe to
>>> start with, like I do. Then, we can avoid the MGA by explaining directly
>>> (in step 7) that the arithmetical reality implements all computations,
>>> which follows from what we can find in all textbook on theoretical computer
>>> science, like Davis chapter 4 (Turing machine self-applied). I am currently
>>> using that material to explain that very points to my students, so I might
>>> make a try to explain a bit here, to give the taste of it. The advantage of
>>> the Turing machine formalism, is that it is close to "physical computer",
>>> and yet simple enough to make the theory not too much hidden in technical
>>> details. I will think how to do that.
>>>
>>
>>
>> In effect then one argues as follows. Beginning with the assumption of
>> CTM, we can agree in principle that the existence of a computational
>> device, instantiated in a primary physical reality capable of computing
>> sufficient of the trace of the UD, would permit the UDA to go through. This
>> is the initial assumption of Step 7 of the UDA. Then any objection that
>> went to a presumed insufficiency of such a primary reality to implement
>> such a computer would be a merely contingent supposition of its 'actual'
>> non-existence. But the question of what is 'actual' with respect to the
>> theory is precisely what is being asked. So any such "Show me the computer"
>> type of objection begs that very question.
>>
>>
>> Absolutely.
>>
>>
>&g

Re: Maudlin's Computation and Consciousness

2017-10-09 Thread David Nyman
On 5 October 2017 at 16:06, Bruno Marchal <marc...@ulb.ac.be> wrote:

>
> On 05 Oct 2017, at 16:02, David Nyman wrote:
>
> On 5 October 2017 at 13:50, Bruno Marchal <marc...@ulb.ac.be> wrote:
>
>>
>> On 02 Oct 2017, at 15:07, David Nyman wrote:
>>
>>
>>
>> On 2 Oct 2017 1:58 p.m., "Bruno Marchal" <marc...@ulb.ac.be> wrote:
>>
>> Dear Evgenii,
>>
>>
>> On 28 Sep 2017, at 21:30, Evgenii Rudnyi wrote:
>>
>>
>>
>>
>> Long time ago you have discussed Maudlin's paper. At that time I somehow
>> did not get interested. Yet, other day I have got strong feeling that I
>> must read Maudlin's paper right now. I guess this could be explained by
>> peculiarities of the universal dovetailer.
>>
>> Anyway, I have read Maudlin's paper, then I have read Hoffmanm's Sandman,
>> and once more Maudlin's paper. I have enjoyed reading, the paper is nicely
>> written. I guess I have understood the argument. Thank you.
>>
>>
>>
>> You are welcome. Don't hesitate to ask any further questions. There are
>> slight differing nuances between the Movie Graph Argument (MGA) and
>> Maudlin's argument. Also, I have found a way to cut the UDA at step 7
>>
>>
>> More details please.
>>
>>
>> Maudlin focuses on the counterfactual issue, which the MGA somehow
>> avoids. Both argument shows the inadequacy of materialism and mechanism,
>> but the MGA assumes that we have already a good idea of what a
>> (mathematical) computation is, and that it is a special logical relation,
>> not something in need of any physical assumption. Then Maudlin's analysis
>> can be used to answer the "counterfactual objection", instead of reminding
>> implicitly the logical nature of what is a computation. In the context of
>> the UDA, the rôle of the MGA is only to show that the move in step 7
>> consisting in assuming a small primary universe, is isomorphic to
>> creationist last rebuttal when saying eventually that they agree with the
>> evidences for evolution, but that God was needed to make that evolution
>> possible. That was already clear with Peter Jones old objection to UDA that
>> only a computation supported by primary matter  can be conscious. That is a
>> sort of magic way of thinking, by rebutting a theory (experimentally
>> testable) by invoking a god or a magic substance which a priori is not
>> testable ... to avoid the search of an a posteriori test, given here by the
>> theory.
>>
>
>
>> , or I could add a simpler step 8.
>>
>>
>> And here.
>>
>>
>> So, it is enough just to NEVER assumes a *primary* physical universe to
>> start with, like I do. Then, we can avoid the MGA by explaining directly
>> (in step 7) that the arithmetical reality implements all computations,
>> which follows from what we can find in all textbook on theoretical computer
>> science, like Davis chapter 4 (Turing machine self-applied). I am currently
>> using that material to explain that very points to my students, so I might
>> make a try to explain a bit here, to give the taste of it. The advantage of
>> the Turing machine formalism, is that it is close to "physical computer",
>> and yet simple enough to make the theory not too much hidden in technical
>> details. I will think how to do that.
>>
>
>
> In effect then one argues as follows. Beginning with the assumption of
> CTM, we can agree in principle that the existence of a computational
> device, instantiated in a primary physical reality capable of computing
> sufficient of the trace of the UD, would permit the UDA to go through. This
> is the initial assumption of Step 7 of the UDA. Then any objection that
> went to a presumed insufficiency of such a primary reality to implement
> such a computer would be a merely contingent supposition of its 'actual'
> non-existence. But the question of what is 'actual' with respect to the
> theory is precisely what is being asked. So any such "Show me the computer"
> type of objection begs that very question.
>
>
> Absolutely.
>
>
>
> This is really forced by the initial assumption of CTM, which puts any
> theory relying on it in the position of justifying the appearance of any
> possible physics, including physical computational devices, on the basis of
> the existence of an arithmetical, not physical, basis of computation.
> 'Arithmetic' here just stands for any theoretically irreducible and
> sufficient basis for computation. The relevant sense of 'existence', as in
> any fundamental theory, essentially equat

Re: Maudlin's Computation and Consciousness

2017-10-05 Thread David Nyman
On 5 Oct 2017 16:06, "Bruno Marchal" <marc...@ulb.ac.be> wrote:


On 05 Oct 2017, at 16:02, David Nyman wrote:

On 5 October 2017 at 13:50, Bruno Marchal <marc...@ulb.ac.be> wrote:

>
> On 02 Oct 2017, at 15:07, David Nyman wrote:
>
>
>
> On 2 Oct 2017 1:58 p.m., "Bruno Marchal" <marc...@ulb.ac.be> wrote:
>
> Dear Evgenii,
>
>
> On 28 Sep 2017, at 21:30, Evgenii Rudnyi wrote:
>
>
>
>
> Long time ago you have discussed Maudlin's paper. At that time I somehow
> did not get interested. Yet, other day I have got strong feeling that I
> must read Maudlin's paper right now. I guess this could be explained by
> peculiarities of the universal dovetailer.
>
> Anyway, I have read Maudlin's paper, then I have read Hoffmanm's Sandman,
> and once more Maudlin's paper. I have enjoyed reading, the paper is nicely
> written. I guess I have understood the argument. Thank you.
>
>
>
> You are welcome. Don't hesitate to ask any further questions. There are
> slight differing nuances between the Movie Graph Argument (MGA) and
> Maudlin's argument. Also, I have found a way to cut the UDA at step 7
>
>
> More details please.
>
>
> Maudlin focuses on the counterfactual issue, which the MGA somehow avoids.
> Both argument shows the inadequacy of materialism and mechanism, but the
> MGA assumes that we have already a good idea of what a (mathematical)
> computation is, and that it is a special logical relation, not something in
> need of any physical assumption. Then Maudlin's analysis can be used to
> answer the "counterfactual objection", instead of reminding implicitly the
> logical nature of what is a computation. In the context of the UDA, the
> rôle of the MGA is only to show that the move in step 7 consisting in
> assuming a small primary universe, is isomorphic to creationist last
> rebuttal when saying eventually that they agree with the evidences for
> evolution, but that God was needed to make that evolution possible. That
> was already clear with Peter Jones old objection to UDA that only a
> computation supported by primary matter  can be conscious. That is a sort
> of magic way of thinking, by rebutting a theory (experimentally testable)
> by invoking a god or a magic substance which a priori is not testable ...
> to avoid the search of an a posteriori test, given here by the theory.
>


> , or I could add a simpler step 8.
>
>
> And here.
>
>
> So, it is enough just to NEVER assumes a *primary* physical universe to
> start with, like I do. Then, we can avoid the MGA by explaining directly
> (in step 7) that the arithmetical reality implements all computations,
> which follows from what we can find in all textbook on theoretical computer
> science, like Davis chapter 4 (Turing machine self-applied). I am currently
> using that material to explain that very points to my students, so I might
> make a try to explain a bit here, to give the taste of it. The advantage of
> the Turing machine formalism, is that it is close to "physical computer",
> and yet simple enough to make the theory not too much hidden in technical
> details. I will think how to do that.
>


In effect then one argues as follows. Beginning with the assumption of CTM,
we can agree in principle that the existence of a computational device,
instantiated in a primary physical reality capable of computing sufficient
of the trace of the UD, would permit the UDA to go through. This is the
initial assumption of Step 7 of the UDA. Then any objection that went to a
presumed insufficiency of such a primary reality to implement such a
computer would be a merely contingent supposition of its 'actual'
non-existence. But the question of what is 'actual' with respect to the
theory is precisely what is being asked. So any such "Show me the computer"
type of objection begs that very question.


Absolutely.



This is really forced by the initial assumption of CTM, which puts any
theory relying on it in the position of justifying the appearance of any
possible physics, including physical computational devices, on the basis of
the existence of an arithmetical, not physical, basis of computation.
'Arithmetic' here just stands for any theoretically irreducible and
sufficient basis for computation. The relevant sense of 'existence', as in
any fundamental theory, essentially equates to explanatory power. It would
of course be open to anyone to additionally assume the existence of a
putatively more 'explanatorily primitive' physical reality. But this could
only weaken the theory by arbitrarily invoking the 'preselection' of
undetectable mechanisms that then had no further explanatory role in what
followed. Hence this move should be abandoned in favour of greater
explanatory parsimony.


I think you see the p

Re: Maudlin's Computation and Consciousness

2017-10-05 Thread David Nyman
On 5 October 2017 at 13:50, Bruno Marchal <marc...@ulb.ac.be> wrote:

>
> On 02 Oct 2017, at 15:07, David Nyman wrote:
>
>
>
> On 2 Oct 2017 1:58 p.m., "Bruno Marchal" <marc...@ulb.ac.be> wrote:
>
> Dear Evgenii,
>
>
> On 28 Sep 2017, at 21:30, Evgenii Rudnyi wrote:
>
>
>
>
> Long time ago you have discussed Maudlin's paper. At that time I somehow
> did not get interested. Yet, other day I have got strong feeling that I
> must read Maudlin's paper right now. I guess this could be explained by
> peculiarities of the universal dovetailer.
>
> Anyway, I have read Maudlin's paper, then I have read Hoffmanm's Sandman,
> and once more Maudlin's paper. I have enjoyed reading, the paper is nicely
> written. I guess I have understood the argument. Thank you.
>
>
>
> You are welcome. Don't hesitate to ask any further questions. There are
> slight differing nuances between the Movie Graph Argument (MGA) and
> Maudlin's argument. Also, I have found a way to cut the UDA at step 7
>
>
> More details please.
>
>
> Maudlin focuses on the counterfactual issue, which the MGA somehow avoids.
> Both argument shows the inadequacy of materialism and mechanism, but the
> MGA assumes that we have already a good idea of what a (mathematical)
> computation is, and that it is a special logical relation, not something in
> need of any physical assumption. Then Maudlin's analysis can be used to
> answer the "counterfactual objection", instead of reminding implicitly the
> logical nature of what is a computation. In the context of the UDA, the
> rôle of the MGA is only to show that the move in step 7 consisting in
> assuming a small primary universe, is isomorphic to creationist last
> rebuttal when saying eventually that they agree with the evidences for
> evolution, but that God was needed to make that evolution possible. That
> was already clear with Peter Jones old objection to UDA that only a
> computation supported by primary matter  can be conscious. That is a sort
> of magic way of thinking, by rebutting a theory (experimentally testable)
> by invoking a god or a magic substance which a priori is not testable ...
> to avoid the search of an a posteriori test, given here by the theory.
>


> , or I could add a simpler step 8.
>
>
> And here.
>
>
> So, it is enough just to NEVER assumes a *primary* physical universe to
> start with, like I do. Then, we can avoid the MGA by explaining directly
> (in step 7) that the arithmetical reality implements all computations,
> which follows from what we can find in all textbook on theoretical computer
> science, like Davis chapter 4 (Turing machine self-applied). I am currently
> using that material to explain that very points to my students, so I might
> make a try to explain a bit here, to give the taste of it. The advantage of
> the Turing machine formalism, is that it is close to "physical computer",
> and yet simple enough to make the theory not too much hidden in technical
> details. I will think how to do that.
>


In effect then one argues as follows. Beginning with the assumption of CTM,
we can agree in principle that the existence of a computational device,
instantiated in a primary physical reality capable of computing sufficient
of the trace of the UD, would permit the UDA to go through. This is the
initial assumption of Step 7 of the UDA. Then any objection that went to a
presumed insufficiency of such a primary reality to implement such a
computer would be a merely contingent supposition of its 'actual'
non-existence. But the question of what is 'actual' with respect to the
theory is precisely what is being asked. So any such "Show me the computer"
type of objection begs that very question.

This is really forced by the initial assumption of CTM, which puts any
theory relying on it in the position of justifying the appearance of any
possible physics, including physical computational devices, on the basis of
the existence of an arithmetical, not physical, basis of computation.
'Arithmetic' here just stands for any theoretically irreducible and
sufficient basis for computation. The relevant sense of 'existence', as in
any fundamental theory, essentially equates to explanatory power. It would
of course be open to anyone to additionally assume the existence of a
putatively more 'explanatorily primitive' physical reality. But this could
only weaken the theory by arbitrarily invoking the 'preselection' of
undetectable mechanisms that then had no further explanatory role in what
followed. Hence this move should be abandoned in favour of greater
explanatory parsimony.

David


>
> Bruno
>
>
>
>
>
>
>
>
>
> David
>
>
> Kind regards,
>
> Bruno
>
>
>
>
>
&g

Re: When you split the brain, do you split the person?

2017-10-03 Thread David Nyman
Interesting, Telmo.

My take would be along the following lines. In the first place, I take the
view that questions of identity, personal history, localisation in space
and time etc. are necessarily a consequence of locally-bounded
neurocognition, leading to the somewhat imprecise but still tractable
notion of the observer moment. Let us then assume the persistence in the
split-brain case of substantial hemispheric integration of both immediate
experience and short and longer-term memory. Presumably this could occur
via ongoing bilateral sensory input and surviving cross-hemispheric
connections, the corpus callosum apparently not being the exclusive, even
if the principal, such pathway. In such a case, I would still expect the
net experience to persist as that of a single person, even if not entirely
seamlessly.

As a hypothetical test of this speculation, consider the phenomenon of
multiple personality, in which there is reportedly, by contrast,
considerable disruption and disjointedness to the memories available within
each personal 'compartment'. Here we would have a situation where, from the
perspective of different such groupings of observer moments, there would be
inexplicable lapses of memory and even third-party reports of unrecalled,
'uncharacteristic' behaviour. In these cases, any one of the 'compartments'
might conclude that the best explanation was, in effect, that some 'other
person' must have been in command of the same body. Between this and the
split-brain case, one can envisage a sort of sliding scale of integration
of memory and agency, associated with a correspondingly variable sense of
personal integrity.

In point of fact, since none of us are 100% fully integrated in the sense
I'm developing here, this is to some extent our common existential
situation. Strikingly similar effects to the split-brain case have been
produced as a consequence of deliberate post-hypnotic suggestion. For
example, someone is told under hypnosis that they will perform a certain
action on hearing a trigger word. Post-hypnotic amnesia leaves them
unaware of the existence of the trigger, but they respond to it with the
suggested action nonetheless. But when asked why, they don't reply with "I
have no idea" or "I can't remember". Instead, they confabulate something
plausible. And this in fact is what we all do at least some of the time,
split-brain or not, in response to environmental 'triggers' or 'priming'
about whose origins we have no explicit knowledge.

David



On 3 October 2017 at 14:11, Telmo Menezes  wrote:

> I think this is quite interesting, although the article is a bit
> superficial.
>
> https://aeon.co/ideas/when-you-split-the-brain-do-you-split-the-person
>
> If the conclusions are valid, I would say they put emergentism in
> trouble...
>
> Cheers,
> Telmo.
>
> --
> You received this message because you are subscribed to the Google Groups
> "Everything List" group.
> To unsubscribe from this group and stop receiving emails from it, send an
> email to everything-list+unsubscr...@googlegroups.com.
> To post to this group, send email to everything-list@googlegroups.com.
> Visit this group at https://groups.google.com/group/everything-list.
> For more options, visit https://groups.google.com/d/optout.
>

-- 
You received this message because you are subscribed to the Google Groups 
"Everything List" group.
To unsubscribe from this group and stop receiving emails from it, send an email 
to everything-list+unsubscr...@googlegroups.com.
To post to this group, send email to everything-list@googlegroups.com.
Visit this group at https://groups.google.com/group/everything-list.
For more options, visit https://groups.google.com/d/optout.


Re: Maudlin's Computation and Consciousness

2017-10-02 Thread David Nyman
On 2 Oct 2017 1:58 p.m., "Bruno Marchal"  wrote:

Dear Evgenii,


On 28 Sep 2017, at 21:30, Evgenii Rudnyi wrote:




Long time ago you have discussed Maudlin's paper. At that time I somehow
did not get interested. Yet, other day I have got strong feeling that I
must read Maudlin's paper right now. I guess this could be explained by
peculiarities of the universal dovetailer.

Anyway, I have read Maudlin's paper, then I have read Hoffmanm's Sandman,
and once more Maudlin's paper. I have enjoyed reading, the paper is nicely
written. I guess I have understood the argument. Thank you.



You are welcome. Don't hesitate to ask any further questions. There are
slight differing nuances between the Movie Graph Argument (MGA) and
Maudlin's argument. Also, I have found a way to cut the UDA at step 7


More details please.

, or I could add a simpler step 8.


And here.

David


Kind regards,

Bruno





Best wishes,

Evgenii

-- 
You received this message because you are subscribed to the Google Groups
"Everything List" group.
To unsubscribe from this group and stop receiving emails from it, send an
email to everything-list+unsubscr...@googlegroups.com.
To post to this group, send email to everything-list@googlegroups.com.
Visit this group at https://groups.google.com/group/everything-list.
For more options, visit https://groups.google.com/d/optout.


http://iridia.ulb.ac.be/~marchal/



-- 
You received this message because you are subscribed to the Google Groups
"Everything List" group.
To unsubscribe from this group and stop receiving emails from it, send an
email to everything-list+unsubscr...@googlegroups.com.
To post to this group, send email to everything-list@googlegroups.com.
Visit this group at https://groups.google.com/group/everything-list.
For more options, visit https://groups.google.com/d/optout.

-- 
You received this message because you are subscribed to the Google Groups 
"Everything List" group.
To unsubscribe from this group and stop receiving emails from it, send an email 
to everything-list+unsubscr...@googlegroups.com.
To post to this group, send email to everything-list@googlegroups.com.
Visit this group at https://groups.google.com/group/everything-list.
For more options, visit https://groups.google.com/d/optout.


Re: A profound lack of profundity (and soon "the starting point")

2017-09-30 Thread David Nyman
On 30 Sep 2017 6:03 p.m., "John Clark" <johnkcl...@gmail.com> wrote:

On Sat, Sep 30, 2017 at 7:08 AM, David Nyman <david.ny...@gmail.com> wrote:

​> ​
> Actually there have been some quite interesting discussions outside the JC
> echo chamber, I think, Quentin. I don't bother with the troll,
>

​So you believe Quentin's ideas are so brilliant that nobody could
sincerely disagree with them and I have written hundreds of posts over the
years defending a position I did not believe had any value. Hmm... I bet
you voted for Trump.  ​

​> ​
> although I occasionally read your contributions because the degree of, no
> doubt understandable, vitriol you have accumulated towards his attitude to
> the discussion is quite entertaining.


​Entertaining in the way intestinal worms are entertaining perhaps.

​> ​
> Personally I've never been able to understand all the fuss. In a world
> with duplication machines we'd just have to accept that other people might
> have a legitimate claim to be the successors of the same predecessor as
> ourselves.
>

​If you know there will be more than one successor then​

​asking the predecessor what one and only one thing will happen to that
predecessor
​ would be a brain dead dumb thing to do.


> ​> ​
> But that couldn't possibly have any bearing on the necessity of finding
> ourselves to be one single individual at any given moment.
>

No bearing? You
​ ​
just
​ ​
said "finding ourselves" and that's plural, so when you ask "what one and
only one thing" what the hell are you asking and who are you asking it of?


I'd reread what you said before you even think about calling other people
stupid. I'm sure you're capable of substituting 'oneself' if that's what's
bothering you. We find ourselves to be only one person at a time. In fact,
that is the entire point that you seem forever intent on preserving the
pretence of failing to understand.

In any case I didn't in point of fact ask you anything since asking you
anything is manifestly futile.

​ ​
Some very stupid people, such as those that think Quentin
​ ​
is clever, believe it's like a coin flip, but it's nothing like a coin
flip! Today I don't  know if
​ ​
the flip
​ ​
will
​ ​
end up
​ ​
heads or tails and the best I can do is assign probabilities, but tomorrow
I won't need probabilities at all,
​ ​
tomorrow
​ ​
I can state with 100% certainty
​ ​
exactly how it turned out; however with Bruno's thought experiment tomorrow
after its completed everybody still will be as ignorant of the answer as
they were the day before because
​ ​
it's still not clear what the question was.


> ​>​
>  I suppose it's just barely within the bounds of possibility that some
> poor soul might be incapable of understanding what is entailed in BEING
> someone as distinct from DESCRIBING someone. But if that were indeed the
> case one could only shake one's head and pass on by.


​Even in a world with people duplicating machines I have no trouble
understanding what *BEING* someone means, and I have no trouble
understanding what having *BEEN* somebody means, but I have enormous
difficulty understanding what the one and only one person I *WILL BE*
means.


Who asked you? Not me. But you can perhaps comprehend that you could find
yourself in the position of remembering having had the same predecessor as
someone else. In that case, as convention has it, the individual that you
find yourself to be can be considered as one of the future continuations of
that predecessor. And by that same token, as not being any of the others.
So the predecessor's anticipated 'next moment is in fact a single present
moment in the point of view of any one of the successors. Each will recall
that moment of anticipation as the immediately preceding part of their
personal history.

The predecessor, knowing about the duplication, is able to DESCRIBE more
than one such continuation. But they will also easily appreciate that their
present state will inevitably be recalled from the perspective of not more
than one of those continuations at any given future moment. The
restrictions logically contingent on the nature of individuality preclude
the possibility of BEING, which is to say occupying the perspective of,
more than a single individual at any given moment.

It may possibly help with any conceptual difficulties to consider that just
as multiple continuations can't be experienced simultaneously, they can
nevertheless be conceived as being experienced serially. Hence in a certain
sense the predecessor can indeed anticipate experiencing BEING both
continuations, but serially and not in the context of the same moment. The
personal histories of such 'serially simultaneous' moments then
subsequently diverge. This heuristic of what one may term 'universal
agency' is borrowed from Hoyle.

Since the relation with both personal history and spatial-temporal
localisat

Re: A profound lack of profundity (and soon "the starting point")

2017-09-30 Thread David Nyman
On 30 Sep 2017 10:49 a.m., "Quentin Anciaux"  wrote:



2017-09-30 11:40 GMT+02:00 Bruno Marchal :

>
> On 29 Sep 2017, at 19:39, John Clark wrote:
>
> On Thu, Sep 28, 2017 at 11:48 PM, Terren Suydam 
> wrote:
>
> ​> ​
>> This thought experiment must be analyzed from the first person perspective
>>
>
> ​There is no *THE* ​
> first person perspective
> ​ if ​
> first person perspective
> ​ duplicating machines exist! It's the same blunder over and over and
> over again.
>
>
>> ​> ​
>> (and by that I'm referring to the grammatical person
>> ).
>>
>
> ​
> I
> ​would bet money that ​
>  the third grade English teacher
> ​that ​
> wrote that article did not have first person perspective
> ​ ​
> duplicating machines
> ​ ​
> in mind.
>
>
>> ​> ​
>> There is only one stream of consciousness, ever,
>>
>
> ​Then why can't anybody *ever* tell me if that ​
>  one stream of consciousness
> ​ is in Moscow or Washington?​
>
>
>
> Because that is non determinable from the first person point of you. Here
> Terren won the point. (Like if this could change anything in your attitude
> alas).
>
> Bruno
>

just ask him to describe from a 1st pov what happens... like this:

I'm in helsinki, I'm in front of a button, I close my eyes, I push on the
button, and I open my eyes, and I am ...

I see only one valid continuation of the text if written before pushing the
button:

-> in washington or in moscow.

I see two valid continuations of the text given by the two copies if
written after pushing the button:

-> in washingtom
-> in moscow

What will never be valid if written before or after is this;

-> in washington and in moscow.

Because being in washington *and* moscow is not something that can be lived
from the 1st pov.

But JC will still blurry anything by either pretending it's 1st POV vanish
from existense or that the answer is santa claus or obiwan kenobi...

So the only real valid continuation here is to *stop* talking with him and
about this. The last 10 years of this list is a circle dumb discussion
between you and JC... JC has clearly destroyed the goal of this list, and
you engaging him on this is just the way to perpetuate that troll
forever... *he will never acknowledge anything*, he is a troll... that's
his purpose, he doesn't *care* of what you're saying, he is just taking
pride of destroying this place of discussion about everything theories (not
only yours).

As I see it, this list died 10 years ago, nothing interresting as come out
of it since unfortunately... only infinite useless step 3 discussion with a
troll.


Actually there have been some quite interesting discussions outside the JC
echo chamber, I think, Quentin. I don't bother with the troll, although I
occasionally read your contributions because the degree of, no doubt
understandable, vitriol you have accumulated towards his attitude to the
discussion is quite entertaining.

Personally I've never been able to understand all the fuss. In a world with
duplication machines we'd just have to accept that other people might have
a legitimate claim to be the successors of the same predecessor as
ourselves. But that couldn't possibly have any bearing on the necessity of
finding ourselves to be one single individual at any given moment.

Any other possibility would simply violate the logical restriction on what
it means to BE one individual as distinct from another. I suppose it's just
barely within the bounds of possibility that some poor soul might be
incapable of understanding what is entailed in BEING someone as distinct
from DESCRIBING someone. But if that were indeed the case one could only
shake one's head and pass on by.

David


 Quentin


>
>
>
>
>> ​> ​
>> despite the possibility of its bifurcation (no different from many-worlds)
>>
>
> ​In ​
> many-worlds
> ​ the meaning of personal pronouns are always clear, in Bruno's thought
> experiment ​they never are.
>
>
>> ​> ​
>> The only reality a person experiences is the one inside their head.
>> Thanks to this, we never have to get into pronouns
>
>
> Then why is ​
> Terren Suydam
> ​ unable to state ​
> Terren Suydam
> ​'s ideas without the constant use of personal pronouns and the misuse of
> articles like "the" and "a"?
>
>
> ​> ​
>> You seem to have a hang-up that prevents you from adopting that
>> perspective
>>
>
> ​My ​
> hang-up
> ​ is I don't know what ​
> perspective
> ​ you're talking about and neither do you.​
>
>
>> ​> ​
>> you compulsively return to questions about the objective reality,
>>
>
> ​Objective reality is important but subjective reality is even more
> important. There is only one objective reality but there are billions of
> subjective realities, so a question about subjective reality needs to
> specify which one it's referring to, and the way English grammar uses
> personal pronouns just can't do that if people duplicating machines are in
> the mix.
>
>
>> ​> ​

Hoffman on Consciousness

2017-09-26 Thread David Nyman
https://www.youtube.com/watch?v=oYp5XuGYqqY

I've encountered Hoffman's ideas before. What he has to say may strike you
as somewhat obvious in the context of this list, but it's quite well
articulated and his metaphors and analogies are quite memorable. Towards
the end he says "Perhaps reality is some vast, interacting network of
conscious agents, simple and complex, that cause each other's conscious
experiences. Actually this isn't as crazy an idea as it seems and I'm
currently exploring it."

First-person plural, anyone? Sounds like he could make some interesting
contributions here.

David

-- 
You received this message because you are subscribed to the Google Groups 
"Everything List" group.
To unsubscribe from this group and stop receiving emails from it, send an email 
to everything-list+unsubscr...@googlegroups.com.
To post to this group, send email to everything-list@googlegroups.com.
Visit this group at https://groups.google.com/group/everything-list.
For more options, visit https://groups.google.com/d/optout.


Re: A profound lack of profundity (and soon "the starting point")

2017-09-26 Thread David Nyman
On 25 September 2017 at 22:34, Terren Suydam 
wrote:

>
>
> On Mon, Sep 25, 2017 at 1:51 PM, John Clark  wrote:
>
>> On Mon, Sep 25, 2017 at 9:47 AM, Terren Suydam 
>> wrote:
>>
>> ​> ​
>>> Then we agree that expectations are important, since the wrong ones can
>>> kill us.
>>>
>>
>> ​
>> Forget important, expectations are not even meaningful in thought
>> experiments involving people duplicating machines if
>> ​ ​
>> it is not clearly stated what is being expected.
>>
>
> You're arguing against things I haven't said. To be frank, I tuned out of
> the John Clark/Bruno Marchal wars sometime last year... after a few dozen
> times around that carousel I wanted off. So don't assume I'm going to say
> something Bruno is saying. I do expect (ahem) to be able to clearly state
> was is being expected in the thought experiment, without requiring personal
> pronouns.
>
> But before we continue, I need to be sure we agree that from your
> first-person perspective, when it comes to making decisions based on some
> future state, you only have the contents of your mind to work with. Your
> mental model, your worldview, pick whatever language you like, it's what
> gives you a sense of what to expect, and therefore the only basis one has
> for placing bets. Do you disagree?
>

​First rule of Fight Club​: Don't expect to win in any continuation.
Second rule: Don't even think about expecting to win in any continuation.
Third rule: You will not win in any continuation.

Well, you know what result to 'expect' by now.

David



>
> Terren
>
> --
> You received this message because you are subscribed to the Google Groups
> "Everything List" group.
> To unsubscribe from this group and stop receiving emails from it, send an
> email to everything-list+unsubscr...@googlegroups.com.
> To post to this group, send email to everything-list@googlegroups.com.
> Visit this group at https://groups.google.com/group/everything-list.
> For more options, visit https://groups.google.com/d/optout.
>

-- 
You received this message because you are subscribed to the Google Groups 
"Everything List" group.
To unsubscribe from this group and stop receiving emails from it, send an email 
to everything-list+unsubscr...@googlegroups.com.
To post to this group, send email to everything-list@googlegroups.com.
Visit this group at https://groups.google.com/group/everything-list.
For more options, visit https://groups.google.com/d/optout.


Andrei Linde on consciousness

2017-09-25 Thread David Nyman
https://www.youtube.com/watch?v=Gc89m2SaOAc=youtu.be

-- 
You received this message because you are subscribed to the Google Groups 
"Everything List" group.
To unsubscribe from this group and stop receiving emails from it, send an email 
to everything-list+unsubscr...@googlegroups.com.
To post to this group, send email to everything-list@googlegroups.com.
Visit this group at https://groups.google.com/group/everything-list.
For more options, visit https://groups.google.com/d/optout.


Infinities

2017-09-22 Thread David Nyman
https://www.quantamagazine.org/mathematicians-measure-infinities-find-theyre-equal-20170912/

-- 
You received this message because you are subscribed to the Google Groups 
"Everything List" group.
To unsubscribe from this group and stop receiving emails from it, send an email 
to everything-list+unsubscr...@googlegroups.com.
To post to this group, send email to everything-list@googlegroups.com.
Visit this group at https://groups.google.com/group/everything-list.
For more options, visit https://groups.google.com/d/optout.


Re: Is math real?

2017-09-14 Thread David Nyman
On 14 Sep 2017 4:47 a.m., "Brent Meeker" <meeke...@verizon.net> wrote:



On 9/13/2017 4:06 AM, David Nyman wrote:



On 11 Sep 2017 6:21 p.m., "Brent Meeker" <meeke...@verizon.net> wrote:



On 9/11/2017 1:22 AM, Bruno Marchal wrote:

>
> On 10 Sep 2017, at 22:25, Brent Meeker wrote:
>
>
>>
>> On 9/10/2017 10:24 AM, Bruno Marchal wrote:
>>
>>> So I assume elementary arithmetic; I prove the existence of the
>>> universal number(s), then I define a notion of rational belief "scientific
>>> belief", (Plotinus discursive reasoner) by Gödel's (sigma_1 arithmetical)
>>> beweisbar Bp. That makes sense, due to incompleteness which prevent
>>> provability to be a notion of knowledge.
>>>
>>
>> This seems problematic to me.  I understand why you do it; because you
>> want knowledge to be true belief (not just true provable belief).  But this
>> does violence to the usual meaning of knowledge (c.f. Getteir for example).
>>
>
> Yes. Incompleteness makes provability into belief instead of knowledge.
> Gödel mention this already in 1933.
>
>
>
> It means that given some undecidable proposition one of us can assert it
>> and the other deny it, and then one of us will know it. ??
>>
>
> Ih he proves it (correctly or not).
>

But that is inconsistent with your definition of "know" = "true belief".
You are really using "know" = "true and proven".   Which is closer to
Gettier's "caused true belief".


I think you're missing the point I've been attempting to develop in my last
couple of posts. Truth, or 'correspondence with a reality', can only be
relative to a point of view.


That's the very antithesis of the usual understanding of "reality".  As my
later friend Vic Stenger put it, reality is what is point of view
invariant. That's why we replicate experiments - to make sure we're not
fooling ourselves.


Again, I'm afraid you're conflating the notion of truth as the result of an
extended process of verification and checking, with that of the primary
undoubtability of phenomenonal evidence. It's the latter we were discussing
here, unless your intention is just to change the subject. If you need a
criterion to distinguish the two concepts, just ask yourself if there is
any characteristic of a given situation that is beyond doubt. If you can
answer in the affirmative you have identified the truth in question.



It's perfectly possibly that any such idiosyncratic, though unavoidable,
commitment


What committment? Your committment to the idea that, 'correspondence with a
reality', can only be relative to a point of view?


I'm not sure it's entirely helpful to interpolate questions in the middle
of sentences. But, as I hope might have been clearer in the light of the
whole, the notion of belief here, as I am trying to clarify, is more akin
to a commitment rather than a hope, so to speak. IOW, if I 'believe' in the
sense of Bp I am willy-nilly committed to the implied (relative) truth
associated with that commitment.

The general idea, which it was my intention to discuss, is that the comp
theory leads to the speculation that such beliefs/commitments and their
corresponding truths/realities are what ultimately ramify into full-blown
physical/phenomenal viewpoints. The general rationale here is that since
all of the necessary logic is emulable by the 'universal machine' it will
consequently will be so emulated and hence form part of the spectrum of
selectable computations.



may deviate from some more pervasive and general underlying consistency


Are you defining reality as a "consistency"?  Are your provable beliefs not
consistent?


No, I'm saying that erroneous beliefs or commitments, as outlined above,
may be nested within ones that are more generally consistent with a wider
environmental perspective. This is in fact the corrective to erroneous
belief provided by an evolutionary logic and it is thereby both
indispensable and hazardous to each individual which, willy-nilly, must
place 'bets' on its own idiosyncratic version of reality.




and that this may put its possessor at hazard. That's the ineluctable logic
of evolution. Nevertheless if something is true for me, in this primary or
undoubtable sense, it will correspond with my (relative) reality,


Are your beliefs undoubtable?


In the primary sense I've explained. More than once actually. What is so
hard to grasp about this distinction?

This whole paragraph makes no sense to me.  What does it mean, "true for
me" and "relative reality"?


My personal phenomenonal reality of course. What else are we discussing?



in both its formal or effective aspect (Bp) and its truthful or phenomenal
one (and p). Any subsequent interpretation based on such primary givens is
of cou

Re: Is math real?

2017-09-13 Thread David Nyman
On 13 September 2017 at 16:48, Bruno Marchal <marc...@ulb.ac.be> wrote:

>
> On 13 Sep 2017, at 15:05, David Nyman wrote:
>
>
>
> On 13 Sep 2017 12:34 p.m., "Bruno Marchal" <marc...@ulb.ac.be> wrote:
>
>
> On 13 Sep 2017, at 13:06, David Nyman wrote:
>
>
>
> On 11 Sep 2017 6:21 p.m., "Brent Meeker" <meeke...@verizon.net> wrote:
>
>
>
> On 9/11/2017 1:22 AM, Bruno Marchal wrote:
>
>>
>> On 10 Sep 2017, at 22:25, Brent Meeker wrote:
>>
>>
>>>
>>> On 9/10/2017 10:24 AM, Bruno Marchal wrote:
>>>
>>>> So I assume elementary arithmetic; I prove the existence of the
>>>> universal number(s), then I define a notion of rational belief "scientific
>>>> belief", (Plotinus discursive reasoner) by Gödel's (sigma_1 arithmetical)
>>>> beweisbar Bp. That makes sense, due to incompleteness which prevent
>>>> provability to be a notion of knowledge.
>>>>
>>>
>>> This seems problematic to me.  I understand why you do it; because you
>>> want knowledge to be true belief (not just true provable belief).  But this
>>> does violence to the usual meaning of knowledge (c.f. Getteir for example).
>>>
>>
>> Yes. Incompleteness makes provability into belief instead of knowledge.
>> Gödel mention this already in 1933.
>>
>>
>>
>> It means that given some undecidable proposition one of us can assert it
>>> and the other deny it, and then one of us will know it. ??
>>>
>>
>> Ih he proves it (correctly or not).
>>
>
> But that is inconsistent with your definition of "know" = "true belief".
> You are really using "know" = "true and proven".   Which is closer to
> Gettier's "caused true belief".
>
>
> I think you're missing the point I've been attempting to develop in my
> last couple of posts. Truth, or 'correspondence with a reality', can only
> be relative to a point of view. It's perfectly possibly that any such
> idiosyncratic, though unavoidable, commitment may deviate from some more
> pervasive and general underlying consistency and that this may put its
> possessor at hazard. That's the ineluctable logic of evolution.
> Nevertheless if something is true for me, in this primary or undoubtable
> sense, it will correspond with my (relative) reality, in both its formal or
> effective aspect (Bp) and its truthful or phenomenal one (and p). Any
> subsequent interpretation based on such primary givens is of course a
> separate question.
>
> OK, but in the general context, explicitly assuming Mechanism (and thus
> Church's thesis, arithmetic, ...), "p" refer to the "absolute" arithmetical
> truth (or better at some point, the sigma_1 truth).
>
>
> Yes, I was trying to be (too) short, I guess.
>
>
> I think so, but I can't resist. It is for the possible others, and I react
> like an old school teacher because I'm wired like that ;)
>
>
>
>
> I hope you agree that elementary arithmetic is "absolutely true". Just
> slightly more doubtable than consciousness!
>
>
> Yes indeed, for our purposes here.
>
>
> Hmm... That is slightly ambiguous. But as someone said to me "when someone
> begins to doubt that 2+2=4, to avoid the consequence of computationalism,
> it means the reductio ad absurdum is completed!".
>
> Of course, we have to doubt even 2+2=4, as part of being scientist, but
> when we assume computationalism, we can no more, because 2+2=4 is used to
> define it.
>

​Yes, exactly. That's what I meant by "for our purposes here".

David
​

> Simply. Then, a case can be made that we can't doubt the simple
> arithmetical relation with small numbers. 1+1=2 is close to consciousness,
> in matter of doubtability. But 6789 + 6789 = 13578 is already more doubtful!
>
>
>
>
> It's interesting to compare this, by the way, with Dennett's claim about
> the illusory nature of consciousness. He says, in effect, that there is no
> reality - i.e. one that corresponds with (what he calls) our judgements
> about the existence of conscious phenomena - that transcends the mere
> judgements themselves. So his claim is that such judgments are lacking in
> *truth*.
>
> Which is close to nonsense to me, because he use the word "transcend" like
> if observation could lead correctly to such judgment. he is very coherent
> in his materialism, and he is force to eliminate consciousness in that
> process. But that is close to the mechanist reduction ad absurdum, because
> consciousness existence, although not out there, is sti

Re: Is math real?

2017-09-13 Thread David Nyman
On 13 Sep 2017 12:34 p.m., "Bruno Marchal" <marc...@ulb.ac.be> wrote:


On 13 Sep 2017, at 13:06, David Nyman wrote:



On 11 Sep 2017 6:21 p.m., "Brent Meeker" <meeke...@verizon.net> wrote:



On 9/11/2017 1:22 AM, Bruno Marchal wrote:

>
> On 10 Sep 2017, at 22:25, Brent Meeker wrote:
>
>
>>
>> On 9/10/2017 10:24 AM, Bruno Marchal wrote:
>>
>>> So I assume elementary arithmetic; I prove the existence of the
>>> universal number(s), then I define a notion of rational belief "scientific
>>> belief", (Plotinus discursive reasoner) by Gödel's (sigma_1 arithmetical)
>>> beweisbar Bp. That makes sense, due to incompleteness which prevent
>>> provability to be a notion of knowledge.
>>>
>>
>> This seems problematic to me.  I understand why you do it; because you
>> want knowledge to be true belief (not just true provable belief).  But this
>> does violence to the usual meaning of knowledge (c.f. Getteir for example).
>>
>
> Yes. Incompleteness makes provability into belief instead of knowledge.
> Gödel mention this already in 1933.
>
>
>
> It means that given some undecidable proposition one of us can assert it
>> and the other deny it, and then one of us will know it. ??
>>
>
> Ih he proves it (correctly or not).
>

But that is inconsistent with your definition of "know" = "true belief".
You are really using "know" = "true and proven".   Which is closer to
Gettier's "caused true belief".


I think you're missing the point I've been attempting to develop in my last
couple of posts. Truth, or 'correspondence with a reality', can only be
relative to a point of view. It's perfectly possibly that any such
idiosyncratic, though unavoidable, commitment may deviate from some more
pervasive and general underlying consistency and that this may put its
possessor at hazard. That's the ineluctable logic of evolution.
Nevertheless if something is true for me, in this primary or undoubtable
sense, it will correspond with my (relative) reality, in both its formal or
effective aspect (Bp) and its truthful or phenomenal one (and p). Any
subsequent interpretation based on such primary givens is of course a
separate question.

OK, but in the general context, explicitly assuming Mechanism (and thus
Church's thesis, arithmetic, ...), "p" refer to the "absolute" arithmetical
truth (or better at some point, the sigma_1 truth).


Yes, I was trying to be (too) short, I guess.


I hope you agree that elementary arithmetic is "absolutely true". Just
slightly more doubtable than consciousness!


Yes indeed, for our purposes here.

It's interesting to compare this, by the way, with Dennett's claim about
the illusory nature of consciousness. He says, in effect, that there is no
reality - i.e. one that corresponds with (what he calls) our judgements
about the existence of conscious phenomena - that transcends the mere
judgements themselves. So his claim is that such judgments are lacking in
*truth*.

Which is close to nonsense to me, because he use the word "transcend" like
if observation could lead correctly to such judgment. he is very coherent
in his materialism, and he is force to eliminate consciousness in that
process. But that is close to the mechanist reduction ad absurdum, because
consciousness existence, although not out there, is still existing in here.


Yes I agree, but then although he is, as you say, forced by prior
commitment to deny any distinct reality to consciousness, he persists
(deliberately and with polemical purpose, I'm convinced) in using ambiguous
terms like 'illusory'. This terminology easily misleads because we all
think we know what is meant by an illusion. Trouble is, every other
illusion we can bring to mind is in fact a veridical perception,
misinterpreted, and this bleeds into his idiosyncratic use of the same term
to characterise consciousness. I think he takes advantage of this ambiguity
in bullying his less wary readers into a sort of confused acquiescence.

It's a bit like the distinction that's often missed (e.g. in some of my
discussions with Brent) between the primary undoubtability of perceptual
phenomena and their subsequent interpretation. It's the latter, not the
former, that is basically the origin of the notion of the illusory. I've
even seen this misattribution quoted as a rebuttal of Descartes' cogito, as
though he had been claiming that he couldn't be mistaken in *what* he was
experiencing as distinct from *that* he was experiencing. But that very
distinction was always his precise point.

Actually, if we really put the "p" (alone) in consciousness, we get the
unnameable cosmic consciousness of the zeroth person view (but here we are
in G* minus G, and so I am blaspheming again).


Blasphemy apa

Re: Is math real?

2017-09-13 Thread David Nyman
On 11 Sep 2017 6:21 p.m., "Brent Meeker"  wrote:



On 9/11/2017 1:22 AM, Bruno Marchal wrote:

>
> On 10 Sep 2017, at 22:25, Brent Meeker wrote:
>
>
>>
>> On 9/10/2017 10:24 AM, Bruno Marchal wrote:
>>
>>> So I assume elementary arithmetic; I prove the existence of the
>>> universal number(s), then I define a notion of rational belief "scientific
>>> belief", (Plotinus discursive reasoner) by Gödel's (sigma_1 arithmetical)
>>> beweisbar Bp. That makes sense, due to incompleteness which prevent
>>> provability to be a notion of knowledge.
>>>
>>
>> This seems problematic to me.  I understand why you do it; because you
>> want knowledge to be true belief (not just true provable belief).  But this
>> does violence to the usual meaning of knowledge (c.f. Getteir for example).
>>
>
> Yes. Incompleteness makes provability into belief instead of knowledge.
> Gödel mention this already in 1933.
>
>
>
> It means that given some undecidable proposition one of us can assert it
>> and the other deny it, and then one of us will know it. ??
>>
>
> Ih he proves it (correctly or not).
>

But that is inconsistent with your definition of "know" = "true belief".
You are really using "know" = "true and proven".   Which is closer to
Gettier's "caused true belief".


I think you're missing the point I've been attempting to develop in my last
couple of posts. Truth, or 'correspondence with a reality', can only be
relative to a point of view. It's perfectly possibly that any such
idiosyncratic, though unavoidable, commitment may deviate from some more
pervasive and general underlying consistency and that this may put its
possessor at hazard. That's the ineluctable logic of evolution.
Nevertheless if something is true for me, in this primary or undoubtable
sense, it will correspond with my (relative) reality, in both its formal or
effective aspect (Bp) and its truthful or phenomenal one (and p). Any
subsequent interpretation based on such primary givens is of course a
separate question.

It's interesting to compare this, by the way, with Dennett's claim about
the illusory nature of consciousness. He says, in effect, that there is no
reality - i.e. one that corresponds with (what he calls) our judgements
about the existence of conscious phenomena - that transcends the mere
judgements themselves. So his claim is that such judgments are lacking in
*truth*.

David



Brent

Knowledge is Bp & p, which is impossible if p is not provable (~Bp). We
> just cannot know an undecidable (by us)  proposition, by definition,
> although we can bet on it, but then it is different kind of knowledge
> (closer to Bp & Dt).
> That we can know for bad reason is the ultimate lesson of the dream
> argument. People like Malcom who dislike Mechanism are forced into
> disbelieving the existence of consciousness in dreams, as he did.
>
> Bruno
>
>
>
>> Brent
>>
>> --
>> You received this message because you are subscribed to the Google Groups
>> "Everything List" group.
>> To unsubscribe from this group and stop receiving emails from it, send an
>> email to everything-list+unsubscr...@googlegroups.com.
>> To post to this group, send email to everything-list@googlegroups.com.
>> Visit this group at https://groups.google.com/group/everything-list.
>> For more options, visit https://groups.google.com/d/optout.
>>
>
> http://iridia.ulb.ac.be/~marchal/
>
>
>
>
-- 
You received this message because you are subscribed to the Google Groups
"Everything List" group.
To unsubscribe from this group and stop receiving emails from it, send an
email to everything-list+unsubscr...@googlegroups.com.
To post to this group, send email to everything-list@googlegroups.com.
Visit this group at https://groups.google.com/group/everything-list.
For more options, visit https://groups.google.com/d/optout.

-- 
You received this message because you are subscribed to the Google Groups 
"Everything List" group.
To unsubscribe from this group and stop receiving emails from it, send an email 
to everything-list+unsubscr...@googlegroups.com.
To post to this group, send email to everything-list@googlegroups.com.
Visit this group at https://groups.google.com/group/everything-list.
For more options, visit https://groups.google.com/d/optout.


Re: Is math real?

2017-09-11 Thread David Nyman
On 11 September 2017 at 15:56, Bruno Marchal <marc...@ulb.ac.be> wrote:

>
> On 11 Sep 2017, at 11:23, David Nyman wrote:
>
> On 11 Sep 2017 9:22 a.m., "Bruno Marchal" <marc...@ulb.ac.be> wrote:
>
>
> On 10 Sep 2017, at 22:25, Brent Meeker wrote:
>
>
>>
>> On 9/10/2017 10:24 AM, Bruno Marchal wrote:
>>
>>> So I assume elementary arithmetic; I prove the existence of the
>>> universal number(s), then I define a notion of rational belief "scientific
>>> belief", (Plotinus discursive reasoner) by Gödel's (sigma_1 arithmetical)
>>> beweisbar Bp. That makes sense, due to incompleteness which prevent
>>> provability to be a notion of knowledge.
>>>
>>
>> This seems problematic to me.  I understand why you do it; because you
>> want knowledge to be true belief (not just true provable belief).  But this
>> does violence to the usual meaning of knowledge (c.f. Getteir for example).
>>
>
> Yes. Incompleteness makes provability into belief instead of knowledge.
> Gödel mention this already in 1933.
>
>
>
>
> It means that given some undecidable proposition one of us can assert it
>> and the other deny it, and then one of us will know it. ??
>>
>
> Ih he proves it (correctly or not). Knowledge is Bp & p, which is
> impossible if p is not provable (~Bp). We just cannot know an undecidable
> (by us)  proposition, by definition, although we can bet on it, but then it
> is different kind of knowledge (closer to Bp & Dt).
> That we can know for bad reason is the ultimate lesson of the dream
> argument. People like Malcom who dislike Mechanism are forced into
> disbelieving the existence of consciousness in dreams, as he did.
>
>
> Yes, I think the difficulty Brent may be having with this is that the
> notion of belief in play here is to be understood as ramifying in some
> limit (delineated by the FPI) to that of physical structure and action.
>
>
> That follows once we assume the mechanist hypothesis.
>
>
>
> Consequently it constitutes, in the first place, an idiosyncratic
> commitment to truths that may or may not correspond, in part or in whole,
> to what is more generally 'believed'. Nonetheless, commitments of this sort
> cannot be disentangled from their own proper, and equally undoubtable,
> truth values, however misleading these may ultimately turn out to be in a
> wider context. They are, as you say, more in the nature of bets on a
> reality,
>
> In this case it is a weaker bet on absence of change in consciousness for
> some self-transformation, but OK, that is the "reality" in the sense of
> "Dt", arguably.
>

​Yes, that's what I meant.
​ We can't know what lies 'beyond' our perceptions, but we can take a risk
on our conjectures, refined by a process of evolution.​
​

> ​
> which in general of course is consistent with the unavoidable rigour of an
> evolutionary logic. This is the crucial distinction between primary or
> perceptual undoubtability and secondary reliability that I've previously
> remarked on. And as is indeed the case with any serious bet, they represent
> an inescapable commitment that puts the bettor permanently at hazard.
>
> OK.
>
> It seems to me also that there are nested levels of such beliefs and their
> associated truths. Hence what is, at a certain level, an idiosyncratic
> commitment to what we would normally think of as something non veridical,
> as in a dream, may be nested within a more general or systemic commitment
> to a consistent and more generally shared physical reality (i.e. what will
> appear in phenomenal terms as a brain and its generalised environment).
>
> Probably, but the initial nested "levels" we have should be given by the
> hypostases p, Bp, etc. and also the graded B^n p  & D^m t, with m bigger
> than n. With p sigma_1 they all provide a quantization, and thus the
> physical reality is layered in some sense. There are no "correct dream"
> within a dream, because physical correctness appears when "you" are
> distributed all (infinitely many) most probable relative history. This
> might be related to what you say here.
>

​I think it might be. The idea is that the probabilities converge on what
we might then call a canonical (shared) reality.
​

> It plays some role in the "after life", making it a bit closer to to the
> Tibetan Bardo Todol. A poet said that there are only two certainties: taxes
> and death, but that was still wishful thinking
> ​:​
>

​I know, and
 I can't honestly say this
​has ​
give
​n​
me much comfort
​.​




> there is only one certainty: taxes.
>

​Or this :(​

David


> Bruno

Re: Is math real?

2017-09-11 Thread David Nyman
On 11 Sep 2017 9:22 a.m., "Bruno Marchal"  wrote:


On 10 Sep 2017, at 22:25, Brent Meeker wrote:


>
> On 9/10/2017 10:24 AM, Bruno Marchal wrote:
>
>> So I assume elementary arithmetic; I prove the existence of the universal
>> number(s), then I define a notion of rational belief "scientific belief",
>> (Plotinus discursive reasoner) by Gödel's (sigma_1 arithmetical) beweisbar
>> Bp. That makes sense, due to incompleteness which prevent provability to be
>> a notion of knowledge.
>>
>
> This seems problematic to me.  I understand why you do it; because you
> want knowledge to be true belief (not just true provable belief).  But this
> does violence to the usual meaning of knowledge (c.f. Getteir for example).
>

Yes. Incompleteness makes provability into belief instead of knowledge.
Gödel mention this already in 1933.




It means that given some undecidable proposition one of us can assert it
> and the other deny it, and then one of us will know it. ??
>

Ih he proves it (correctly or not). Knowledge is Bp & p, which is
impossible if p is not provable (~Bp). We just cannot know an undecidable
(by us)  proposition, by definition, although we can bet on it, but then it
is different kind of knowledge (closer to Bp & Dt).
That we can know for bad reason is the ultimate lesson of the dream
argument. People like Malcom who dislike Mechanism are forced into
disbelieving the existence of consciousness in dreams, as he did.


Yes, I think the difficulty Brent may be having with this is that the
notion of belief in play here is to be understood as ramifying in some
limit (delineated by the FPI) to that of physical structure and action.
Consequently it constitutes, in the first place, an idiosyncratic
commitment to truths that may or may not correspond, in part or in whole,
to what is more generally 'believed'. Nonetheless, commitments of this sort
cannot be disentangled from their own proper, and equally undoubtable,
truth values, however misleading these may ultimately turn out to be in a
wider context. They are, as you say, more in the nature of bets on a
reality, which in general of course is consistent with the unavoidable
rigour of an evolutionary logic. This is the crucial distinction between
primary or perceptual undoubtability and secondary reliability that I've
previously remarked on. And as is indeed the case with any serious bet,
they represent an inescapable commitment that puts the bettor permanently
at hazard.

It seems to me also that there are nested levels of such beliefs and their
associated truths. Hence what is, at a certain level, an idiosyncratic
commitment to what we would normally think of as something non veridical,
as in a dream, may be nested within a more general or systemic commitment
to a consistent and more generally shared physical reality (i.e. what will
appear in phenomenal terms as a brain and its generalised environment).

David


Bruno




> Brent
>
> --
> You received this message because you are subscribed to the Google Groups
> "Everything List" group.
> To unsubscribe from this group and stop receiving emails from it, send an
> email to everything-list+unsubscr...@googlegroups.com.
> To post to this group, send email to everything-list@googlegroups.com.
> Visit this group at https://groups.google.com/group/everything-list.
> For more options, visit https://groups.google.com/d/optout.
>

http://iridia.ulb.ac.be/~marchal/




-- 
You received this message because you are subscribed to the Google Groups
"Everything List" group.
To unsubscribe from this group and stop receiving emails from it, send an
email to everything-list+unsubscr...@googlegroups.com.
To post to this group, send email to everything-list@googlegroups.com.
Visit this group at https://groups.google.com/group/everything-list.
For more options, visit https://groups.google.com/d/optout.

-- 
You received this message because you are subscribed to the Google Groups 
"Everything List" group.
To unsubscribe from this group and stop receiving emails from it, send an email 
to everything-list+unsubscr...@googlegroups.com.
To post to this group, send email to everything-list@googlegroups.com.
Visit this group at https://groups.google.com/group/everything-list.
For more options, visit https://groups.google.com/d/optout.


Re: Is math real?

2017-09-10 Thread David Nyman
On 10 September 2017 at 18:24, Bruno Marchal <marc...@ulb.ac.be> wrote:

>
> On 09 Sep 2017, at 18:58, David Nyman wrote:
>
> On 7 September 2017 at 10:03, Bruno Marchal <marc...@ulb.ac.be> wrote:
>
>>
>> On 06 Sep 2017, at 19:45, Brent Meeker wrote:
>>
>>
>>
>> On 9/6/2017 7:35 AM, Bruno Marchal wrote:
>>
>> Some physicists can be immaterialist, but still believe that the
>> fundamental reality is physical, a bit like Tegmark who remains (despite he
>> is willing to think differently) open to the idea that the physical reality
>> is a special mathematical structure among all mathematical structures, for
>> example. That is problematical for pure mathematical reason: the notion of
>> all mathematical structures do not make much mathematical sense, but it is
>> of course problematic also with Mechanism, where the physical reality
>> becomes the border of the whole "computable mathematics" (which is very
>> tiny, as it is the tiny sigma_1 part of arithmetic).
>>
>>
>> I think Tegmark has changed his opinion and now only champions all
>> *computable* universes.
>>
>>
>> Yes. The problem now, is that there are no computable physical universes.
>> Here he miss the first person indeterminacy in arithmetic. He miss that any
>> universal machine looking below its substitution level is confronted to its
>> infinity of implementations in arithmetic. In fact, he remains somehow
>> physicalist, and does not seem aware of the computationalist mind-body
>> problem.
>>
>
> ​Yes, it's quite surprising how elusive this absence of universes seems to
> be in the context of mechanism. Old presuppositions seemingly die very
> hard. Another elusive point is what Chalmers is getting at with what he
> calls the Paradox of Phenomenal Judgement​. This is the problem of how what
> one might call an 'extensional infrastructure' and any corresponding
> phenomenal reality are seemingly able to 'refer' to each other. It's a big
> fly in the ointment of physicalist theories of mind like panpsychism,
> although it seems to be exceedingly difficult to point this out to
> panpsychists in my experience. For example, if we consider a movie being
> rendered on an LCD screen, nobody imagines that either the pixels
> comprising the screen, or the action of the movie tracked or carried by
> those pixels, either do, or in any way need to, refer to each other. They
> are, in a sense, mutual epiphenomena. However, my own utterances or
> judgements - standing in a general way for the 'extensional infrastructure'
> of my perceptions - and those perceptions themselves, do indeed seem to
> need to cross-refer. It's this cross-reference that is alluded to in Bp and
> p.
>
>
> Hmm... Perhaps OK. There might be a problem with the "extensional
> infrastructure" where I see an intensional one, and only the "body" is the
> (relatively and indexically) extensional.
>

​Yes, in this case I meant beliefs or judgments as they would appear in
bodily expression, e.g. utterances, and hence extensional.
  ​

>
>
>
> I've been thinking about how this might play out very generally in terms
> of the coincidence or intersection of action and perception as
> generalisations of B and p. As you say, we assume at the outset a knower in
> the guise of the universal or generic machine (i.e. a number playing the
> role of 'processor' with respect to another number).
>
>
> You force me to be very precise. I assume only p, the true sigma_1
> propositions. You can equate them with the computational states attained by
> the, or a, universal dovetailing. I define the 
> "believer-knower-observer-feeler"
> by a universal number, mastering classical first order logic, and (unlike
> what we need to assume for the ontology) the induction axioms (on the
> sigma_1 sentences). The believer can prove its own incompleteness and its
> "modesty", in the conditional way.
>
> So I assume elementary arithmetic; I prove the existence of the universal
> number(s), then I define a notion of rational belief "scientific belief",
> (Plotinus discursive reasoner) by Gödel's (sigma_1 arithmetical) beweisbar
> Bp. That makes sense, due to incompleteness which prevent provability to be
> a notion of knowledge.
>
> Then incompleteness enforce the correct machine to distinguish the nuances
> between p, Bp, Bp & p, Bp & Dt, Bp & Dt & p. Which corresponds with Truth
> ("God", the One, Reality, ...), Belief (theories, ideas), Knowledge (where
> ideas fits with Reality), and the "material" version which encapsulate the
> idea of possibility and non transiti

Fwd: Is math real?

2017-09-09 Thread David Nyman
On 7 September 2017 at 10:03, Bruno Marchal  wrote:

>
> On 06 Sep 2017, at 19:45, Brent Meeker wrote:
>
>
>
> On 9/6/2017 7:35 AM, Bruno Marchal wrote:
>
> Some physicists can be immaterialist, but still believe that the
> fundamental reality is physical, a bit like Tegmark who remains (despite he
> is willing to think differently) open to the idea that the physical reality
> is a special mathematical structure among all mathematical structures, for
> example. That is problematical for pure mathematical reason: the notion of
> all mathematical structures do not make much mathematical sense, but it is
> of course problematic also with Mechanism, where the physical reality
> becomes the border of the whole "computable mathematics" (which is very
> tiny, as it is the tiny sigma_1 part of arithmetic).
>
>
> I think Tegmark has changed his opinion and now only champions all
> *computable* universes.
>
>
> Yes. The problem now, is that there are no computable physical universes.
> Here he miss the first person indeterminacy in arithmetic. He miss that any
> universal machine looking below its substitution level is confronted to its
> infinity of implementations in arithmetic. In fact, he remains somehow
> physicalist, and does not seem aware of the computationalist mind-body
> problem.
>

​Yes, it's quite surprising how elusive this absence of universes seems to
be in the context of mechanism. Old presuppositions seemingly die very
hard. Another elusive point is what Chalmers is getting at with what he
calls the Paradox of Phenomenal Judgement​. This is the problem of how what
one might call an 'extensional infrastructure' and any corresponding
phenomenal reality are seemingly able to 'refer' to each other. It's a big
fly in the ointment of physicalist theories of mind like panpsychism,
although it seems to be exceedingly difficult to point this out to
panpsychists in my experience. For example, if we consider a movie being
rendered on an LCD screen, nobody imagines that either the pixels
comprising the screen, or the action of the movie tracked or carried by
those pixels, either do, or in any way need to, refer to each other. They
are, in a sense, mutual epiphenomena. However, my own utterances or
judgements - standing in a general way for the 'extensional infrastructure'
of my perceptions - and those perceptions themselves, do indeed seem to
need to cross-refer. It's this cross-reference that is alluded to in Bp and
p.

I've been thinking about how this might play out very generally in terms of
the coincidence or intersection of action and perception as generalisations
of B and p. As you say, we assume at the outset a knower in the guise of
the universal or generic machine (i.e. a number playing the role of
'processor' with respect to another number). The computational duals
enacted by such machines are then projected to be elaborated to the point
where they are tracking or carrying the state changes of an extensional
infrastructure equivalent to a brain, at whatever level turns out to be
necessary for its stable emergence. When I say 'equivalent to' I mean that
this is what will appear, from the phenomenal point-of-view of a knower, to
be a brain. Such state changes must, by assumption in some general but
relevant sense, be equivalent to what you call beliefs, or what Dennett
calls judgements, about perception. As in when I utter something like "I
see a red apple", or for that matter "I feel strongly about the current
state of American politics".

At this point, in order for us to persevere with the schema, we grant that
the p, heretofore provisionally referred to as true or real by Bp in these
cases, is that selfsame phenomenal reality referred to by 'and p'. In doing
this, we also grant retrospective validity, or redemption, to the entire
Wittgenstein ladder of logical paraphernalia, emulated in computation, that
we had been ascending for just this purpose.

The question arises as to the 'substitution level' for all this. All
universal machines, and consequently all potential knowers, are formally
equivalent. Consequently we cannot know which of an infinity of such
machines is immediately associated with any given moment of our phenomenal
reality. However, the 'yes doctor' assumption is that at some level the
computations that track or carry the formal equivalent of a brain must be
substitutable by a suitable digital prosthesis, at least in principle. At
the very least, we must assume that an atom-for-atom substitution of a
physical brain, as presumably occurs naturally through time, would preserve
phenomenal reality without significant error.

The question also arises as to the possibility of a tractable 'search
function' for anything corresponding to the above states of affairs within
the Babel-like infinities of the computational plenum. No such function can
be both tractable and extrinsic. We are relying here on filtration by
internal self-identification; this is (obviously, I would 

Re: Is math real?

2017-09-05 Thread David Nyman
On 5 September 2017 at 01:01, Bruce Kellett <bhkell...@optusnet.com.au>
wrote:

> On 5/09/2017 12:49 am, David Nyman wrote:
>
> On 4 Sep 2017 13:11, "Bruce Kellett" <bhkell...@optusnet.com.au> wrote:
>
> On 4/09/2017 9:15 pm, Bruno Marchal wrote:
>
>> On 03 Sep 2017, at 18:46, Brent Meeker wrote:
>>
>> On the contrary, we can only speculate on a primary physical reality for
>>>> which there are no evidences at all.
>>>>
>>>
>>> You can't prove primary arithmetic either.
>>>
>>
>> Indeed.
>>
>> But there are many evidences that 2+2=4. There are no evidence for
>> primary matter. Not one.
>>
>>
>> "Primary" is just a word you stick on "physical" to make it seem
>>> inaccessible.  I don't need to prove the physical, I observe it.
>>>
>>
>> ?
>>
>> Nobody can observe a metaphysical idea. You can observe matter, and that
>> is an evidence for matter, not for primary matter.
>>
>> Primary means "not deducible" from something else.
>>
>
> Bruno, you are just playing with words. I observe matter - that is
> evidence for matter, so the observation is primary, not the matter. But
> then I assume matter and deduce that I will observe it - so the matter
> becomes primary. You claim arithmetic is primary, because 2+2=4 independent
> of you and me. But I can deduce arithmetic from observation, making
> observation primary again, and arithmetic merely derivative. But then I
> assume that matter is primary -  I can then deduce both observation and
> arithmetic.
>
> It is all a matter of choice. You choose to make arithmetic primary, but
> you can't prove that this is necessarily the case. I can assume that quarks
> and electrons, etc, are primary, and else follows from this. Maybe I can't
> prove that either, but I have a hell of a lot more evidence for the
> possibility of deriving arithmetic from the existence of matter than you
> have of proving the existence of quarks from pure arithmetic. The evidence
> is all in my favour.
>
>
> Honestly, Bruce, I think it's you who is playing with words here. The
> sense in which Bruno is using primary here is perfectly clear - i.e. the
> fundamental ontological assumption in a comprehensive theory of origins.
>
>
> That is not what Bruno says above. I quote: "Primary means 'not deducible'
> from something else." Given that definition, then what I say is perfectly
> logical. Primacy has nothing to do with ontology according to Bruno's
> definition.
>

​Yes it has, if you understand the role of ontology in a theory - as I
think Bruno intends it - to mean the inferential basis from which all other
theoretical phenomena will be deduced or inferred.​


>
> It doesn't aid comprehension to substitute a quite different meaning -
> that of primary sense perception -  in 'rebuttal'. As to choice of primary
> ontological assumption, that is fixed by the prior choice of mechanism as
> the theory of mind.
>
>
> But I do not assume mechanism as the theory of mind.
>

​But all Bruno's arguments are based on that assumption. Of course you
don't have to accept it, but then you are proposing a different theory and
that is not the same as a counter-argument to mechanism.​



> It seems to me begging the question to assume the answer before you begin
> the investigation.
>

​Not at all. If nothing else, it is the preliminary to a reductio ad
absurdum, which by his own account is what Bruno originally assumed would
be the outcome. However, as it turned out, the 'reductio' has so far failed
to convince.
​

> One's choice of "primary ontological assumption" is a choice, and I am not
> constrained to assume your ontology in order to discuss your theory.
>

​Well, either that or propose a more convincing one. If the theory is that
perception is a consequence of computation then it behoves the theorist to
take computation seriously as something other than a secondary phenomenon
*inferred* from perception. Despite what you say below, in the context of
the reasoning under consideration, this would be a very blatant beggaring
of the question.


As has been said, "Epistemology precedes ontology", so constraining one's
> ontology from the outset is not necessarily the brightest strategy.
>

Nice quote. However ​I suspect that whoever it was that said that wasn't
considering the possible theoretical origins of ​epistemology at the time.


>
> I think frankly that this is the sticking point for you. You want to claim
> that computation can equally well be 'inferred' from the primary
> ontological assumption of physics. But unfortunately this amounts to
> egregious question begging

Re: Is math real?

2017-09-04 Thread David Nyman
On 4 Sep 2017 13:11, "Bruce Kellett"  wrote:

On 4/09/2017 9:15 pm, Bruno Marchal wrote:

> On 03 Sep 2017, at 18:46, Brent Meeker wrote:
>
> On the contrary, we can only speculate on a primary physical reality for
>>> which there are no evidences at all.
>>>
>>
>> You can't prove primary arithmetic either.
>>
>
> Indeed.
>
> But there are many evidences that 2+2=4. There are no evidence for primary
> matter. Not one.
>
>
> "Primary" is just a word you stick on "physical" to make it seem
>> inaccessible.  I don't need to prove the physical, I observe it.
>>
>
> ?
>
> Nobody can observe a metaphysical idea. You can observe matter, and that
> is an evidence for matter, not for primary matter.
>
> Primary means "not deducible" from something else.
>

Bruno, you are just playing with words. I observe matter - that is evidence
for matter, so the observation is primary, not the matter. But then I
assume matter and deduce that I will observe it - so the matter becomes
primary. You claim arithmetic is primary, because 2+2=4 independent of you
and me. But I can deduce arithmetic from observation, making observation
primary again, and arithmetic merely derivative. But then I assume that
matter is primary -  I can then deduce both observation and arithmetic.

It is all a matter of choice. You choose to make arithmetic primary, but
you can't prove that this is necessarily the case. I can assume that quarks
and electrons, etc, are primary, and else follows from this. Maybe I can't
prove that either, but I have a hell of a lot more evidence for the
possibility of deriving arithmetic from the existence of matter than you
have of proving the existence of quarks from pure arithmetic. The evidence
is all in my favour.


Honestly, Bruce, I think it's you who is playing with words here. The sense
in which Bruno is using primary here is perfectly clear - i.e. the
fundamental ontological assumption in a comprehensive theory of origins. It
doesn't aid comprehension to substitute a quite different meaning - that of
primary sense perception -  in 'rebuttal'. As to choice of primary
ontological assumption, that is fixed by the prior choice of mechanism as
the theory of mind. I think frankly that this is the sticking point for
you. You want to claim that computation can equally well be 'inferred' from
the primary ontological assumption of physics. But unfortunately this
amounts to egregious question begging, since the phenomenon of inference,
and a fortiori any perceptible phenomenon that depends on it, is itself
already part of the mental spectrum whose provenance we're seeking to
explain in the first place.

David



Bruce


-- 
You received this message because you are subscribed to the Google Groups
"Everything List" group.
To unsubscribe from this group and stop receiving emails from it, send an
email to everything-list+unsubscr...@googlegroups.com.
To post to this group, send email to everything-list@googlegroups.com.
Visit this group at https://groups.google.com/group/everything-list.
For more options, visit https://groups.google.com/d/optout.

-- 
You received this message because you are subscribed to the Google Groups 
"Everything List" group.
To unsubscribe from this group and stop receiving emails from it, send an email 
to everything-list+unsubscr...@googlegroups.com.
To post to this group, send email to everything-list@googlegroups.com.
Visit this group at https://groups.google.com/group/everything-list.
For more options, visit https://groups.google.com/d/optout.


Re: Is math real?

2017-09-04 Thread David Nyman
On 4 Sep 2017 12:27 a.m., "Brent Meeker" <meeke...@verizon.net> wrote:



On 9/3/2017 3:07 PM, David Nyman wrote:

On 3 September 2017 at 17:46, Brent Meeker <meeke...@verizon.net> wrote:

>
>
> On 9/3/2017 7:06 AM, Bruno Marchal wrote:
>
>>
>> On 01 Sep 2017, at 19:57, Brent Meeker wrote:
>>
>>
>>>
>>> On 9/1/2017 1:03 AM, Bruno Marchal wrote:
>>>
>>>> This leaves, as Bruno says, lots of white rabbits.
>>>>>
>>>>
>>>> That leaves us in the position of showing that there is no white
>>>> rabbits or, to refute computationalism by showing there are still white
>>>> rabbits, and then you can try to invent some matter or god able to
>>>> eliminate them, but that will in any case refute mechanism.
>>>>
>>>>
>>>>
>>>> What if getting rid of those white rabbits tightly constrains
>>>>> consciousness and physics to something like what we observe?
>>>>>
>>>>
>>>> Exactly. Getting rid of the white rabbit = proving the existence of the
>>>> relevant measure = deriving physics from machine theology (alias elementary
>>>> arithmetic).
>>>>
>>>
>>> Then it will have been shown that physics entails consciousness as well
>>> as the other way around.
>>>
>>
>> OK. But arithmetic is a subtheory of any physical theory. The progress
>> are the following
>>
>> Copenhagen QM: assume a physical reality + a dualist and unclear theory
>> of mind
>>
>> Everett QM: assume a universal wave + the mechanist theory of mind (+ an
>> identity thesis).
>>
>> Me: the mechanist theory of mind (elementary arithmetic).
>>
>> Brent wrote to David:
>>
>> I am agreeing with you.  I only disagree with Bruno in that he wants to
>>> take arithmetic or computation as more really real than physics or
>>> consciousness and not derivative.  It seems to me that the very possibility
>>> of computation depends on the physics of the world and is invented by
>>> evolution.
>>>
>>
>> But that is plainly false. I can prove the existence of computation in
>> arithmetic.
>>
>
> After you assume arithmetic.  I can prove anything if I get to choose the
> axioms.
>
> On the contrary, we can only speculate on a primary physical reality for
>> which there are no evidences at all.
>>
>
> You can't prove primary arithmetic either.  "Primary" is just a word you
> stick on "physical" to make it seem inaccessible.


​I don't think that's right. Primary just means that part of a theory that
is assumed rather than derived.


But  in that case I can just assume that the particles of the Standard
Model are primary.  Then there's a lot of evidence for primary matter.
It's as though physicists are being criticized because they are willing to
look deeper for an explanation of their best theory.  But computationalist
are to be congratulated for asserting that there's no origin for arithmetic.


That criticism is just daft. Of course you can make physics primary if you
like, but then you need to propose a different, non-computational, theory
of mind, one that doesn't covertly add a primary role for "physical
computation", as distinct from "primary physics", as the origin of
phenomenal reality. In fact you have frequently proposed just such a
theory, in the form of an "engineering solution". In which case fine, but
then we're no longer discussing mechanism.



In the case at hand the theory is mechanism, in which it is assumed that
concrete or phenomenal reality ​is ultimately an epistemological
consequence of computation. That being the case, the theory relies on
computation, or its combinatorial basis, as its ontology (i.e. that part of
the theory that is taken to exist independently of point-of-view). It then
sets out to derive its phenomenology by means of an epistemological
analysis (i.e. that part of the theory that is understood to be
point-of-view relative) based on the generic or universal machine as unique
subject or agent. Physics, as an observationally-selected subset both of
the computational ontology and its derived phenomenology, cannot thus be
considered primary, in the sense given here.


Of course it's not primary given a theory that assumes something else as
primary.  Note that computationalism has yet to succeed in deriving
phenomenology.


You need to make up your mind about what you are criticising. Mechanism
necessarily assumes computation as primary and hence must derive physics
and phenomenology. That is its project. Whether that project can ultimately
succeed is

Re: Is math real?

2017-09-03 Thread David Nyman
On 3 September 2017 at 17:46, Brent Meeker  wrote:

>
>
> On 9/3/2017 7:06 AM, Bruno Marchal wrote:
>
>>
>> On 01 Sep 2017, at 19:57, Brent Meeker wrote:
>>
>>
>>>
>>> On 9/1/2017 1:03 AM, Bruno Marchal wrote:
>>>
 This leaves, as Bruno says, lots of white rabbits.
>

 That leaves us in the position of showing that there is no white
 rabbits or, to refute computationalism by showing there are still white
 rabbits, and then you can try to invent some matter or god able to
 eliminate them, but that will in any case refute mechanism.



 What if getting rid of those white rabbits tightly constrains
> consciousness and physics to something like what we observe?
>

 Exactly. Getting rid of the white rabbit = proving the existence of the
 relevant measure = deriving physics from machine theology (alias elementary
 arithmetic).

>>>
>>> Then it will have been shown that physics entails consciousness as well
>>> as the other way around.
>>>
>>
>> OK. But arithmetic is a subtheory of any physical theory. The progress
>> are the following
>>
>> Copenhagen QM: assume a physical reality + a dualist and unclear theory
>> of mind
>>
>> Everett QM: assume a universal wave + the mechanist theory of mind (+ an
>> identity thesis).
>>
>> Me: the mechanist theory of mind (elementary arithmetic).
>>
>> Brent wrote to David:
>>
>> I am agreeing with you.  I only disagree with Bruno in that he wants to
>>> take arithmetic or computation as more really real than physics or
>>> consciousness and not derivative.  It seems to me that the very possibility
>>> of computation depends on the physics of the world and is invented by
>>> evolution.
>>>
>>
>> But that is plainly false. I can prove the existence of computation in
>> arithmetic.
>>
>
> After you assume arithmetic.  I can prove anything if I get to choose the
> axioms.
>
> On the contrary, we can only speculate on a primary physical reality for
>> which there are no evidences at all.
>>
>
> You can't prove primary arithmetic either.  "Primary" is just a word you
> stick on "physical" to make it seem inaccessible.


​I don't think that's right. Primary just means that part of a theory that
is assumed rather than derived. In the case at hand the theory is
mechanism, in which it is assumed that concrete or phenomenal reality ​is
ultimately an epistemological consequence of computation. That being the
case, the theory relies on computation, or its combinatorial basis, as its
ontology (i.e. that part of the theory that is taken to exist independently
of point-of-view). It then sets out to derive its phenomenology by means of
an epistemological analysis (i.e. that part of the theory that is
understood to be point-of-view relative) based on the generic or universal
machine as unique subject or agent. Physics, as an observationally-selected
subset both of the computational ontology and its derived phenomenology,
cannot thus be considered primary, in the sense given here. Rather, it
makes its appearance as a tightly-constrained extensional infrastructure in
terms of which the machine's phenomenology is enabled to play out in action.

David


  I don't need to prove the physical, I observe it.
>
> Your argument is 100% the same as saying "It seems to me that the very
>> possibility of computation depends on God".
>>
>> If God or Matter plays a role in a computation, then you are not taking
>> the word "computation" in its standard meaning (cf
>> Church-Turing-Post-Kleene thesis), and I have no clue at all what you are
>> talking about.
>>
>
> So you put words in my mouth and then complain that you don't know what
> I'm talking about?
>
> Brent
>
>
>
>> Bruno
>>
>>
>>
>>
>>
>>
>>
>>
>>
>>> Brent
>>>
>>> --
>>> You received this message because you are subscribed to the Google
>>> Groups "Everything List" group.
>>> To unsubscribe from this group and stop receiving emails from it, send
>>> an email to everything-list+unsubscr...@googlegroups.com.
>>> To post to this group, send email to everything-list@googlegroups.com.
>>> Visit this group at https://groups.google.com/group/everything-list.
>>> For more options, visit https://groups.google.com/d/optout.
>>>
>>
>> http://iridia.ulb.ac.be/~marchal/
>>
>>
>>
>>
> --
> You received this message because you are subscribed to the Google Groups
> "Everything List" group.
> To unsubscribe from this group and stop receiving emails from it, send an
> email to everything-list+unsubscr...@googlegroups.com.
> To post to this group, send email to everything-list@googlegroups.com.
> Visit this group at https://groups.google.com/group/everything-list.
> For more options, visit https://groups.google.com/d/optout.
>

-- 
You received this message because you are subscribed to the Google Groups 
"Everything List" group.
To unsubscribe from this group and stop receiving emails from it, send an email 
to everything-list+unsubscr...@googlegroups.com.
To post to 

Re: Is math real?

2017-09-01 Thread David Nyman
On 31 Aug 2017 23:59, "Brent Meeker" <meeke...@verizon.net> wrote:



On 8/31/2017 2:20 AM, David Nyman wrote:



On 29 Aug 2017 04:39, "Brent Meeker" <meeke...@verizon.net> wrote:



On 8/28/2017 10:50 AM, Bruno Marchal wrote:


On 28 Aug 2017, at 02:44, Brent Meeker wrote:



On 8/27/2017 10:50 AM, David Nyman wrote:

On 25 August 2017 at 21:51, Brent Meeker <meeke...@verizon.net> wrote:

>
>
> On 8/25/2017 9:44 AM, Bruno Marchal wrote:
>
>>
>> On 24 Aug 2017, at 20:57, Brent Meeker wrote:
>>
>>
>>>
>>> On 8/24/2017 1:20 AM, Bruno Marchal wrote:
>>>
>>>>
>>>> On 23 Aug 2017, at 20:43, Brent Meeker wrote:
>>>>
>>>>
>>>>>
>>>>> On 8/23/2017 2:06 AM, Bruno Marchal wrote:
>>>>>
>>>>>> I am not someone proposing any new theory. I am someone showing that
>>>>>> the current materialist metaphysics just can't work with the Mechanist
>>>>>> hypothesis.
>>>>>>
>>>>>
>>>>> Refresh my understanding.  What it the mechanist hyposthesis? Is it
>>>>> the same as computationalism?
>>>>>
>>>>
>>>> Yes.
>>>>
>>>> Computationalism = Digital Mechanism = Mechanism = (Yes-Doctor +
>>>> Church's Thesis)
>>>>
>>>>
>>>>
>>>>
>>>> Or is it the same as yes-doctor plus reifying arithmetic?
>>>>>
>>>>
>>>> No, it is (yes-doctor + Church's Thesis).
>>>>
>>>> I do not add since long "Arithmetical Realism" because many people tend
>>>> to put to much into it, and is actually redundant with Church's thesis. To
>>>> just understand Church's thesis automatically assume we believe in some
>>>> "essentially undecidable theory", and this is equiavalent with believing in
>>>> the right amount of arithmetic.
>>>> I will write a post on the detailed starting point of the mathematics
>>>> needed to derive physics from "machine's theology".
>>>>
>>>>
>>>>
>>>>
>>>>
>>>> >From your use, these all seem slightly different to me.  It would be
>>>>> helpful to some firm definitions - not just usage.
>>>>>
>>>>
>>>> I use them as completely equivalent, although in the literature they
>>>> are usually stronger. Putnam's functionalism is a version of Digital
>>>> Mechanism which assumes a substitition level rather high, where my version
>>>> just ask for the existence of a substitution level. My version is the
>>>> weaker form possible, and Maudlin, in his Olympia paper, suggests that if
>>>> we define mechanism in this way, it becomes trivial, a bit like Diderot
>>>> defined "rationalism" by Descartes' Mechanism.
>>>>
>>>> So a firm definition of Mechanism (in my weak sense) is
>>>>
>>>> 1) Church's Thesis (a function from N to N is computable iff it exists
>>>> a combinator which computes it)
>>>>
>>>> (There are many variants of this. You can replace also "combinator"
>>>> by "game of life pattern", or "fortran program" or "c++ program", or
>>>> "quantum computer" etc.). Note that this asks for "Arithmetical realism"
>>>> which is only the believe that the RA axioms makes "absolute sense", which
>>>> means basically that not only 17 is prime, but that this is true
>>>> independently of me, you, or anyone, or anything physical. All
>>>> mathematicians are arithmetical realist. The fight on realism is in
>>>> Analysis or set theory, not arithmetic, especially without induction axiom
>>>> like with RA. Even a quasi ultra-finitist like Nelson agrees with RA.
>>>>
>>>>
>>>> 2) Yes-Doctor (= my consciousness is invariant for a digital physical
>>>> brain transplant made at some level of description of my (generalized)
>>>> brain.
>>>>
>>>> It asserts the existence of that substitution level, and is equivalent
>>>> with accepting that we can use classical teleportation as a mean of travel
>>>> (UDA step 1).
>>>>
>>>> Important Remark: that definition does not ask for surviving without a
>>>> physical brain/machine. That is indeed the object of the UDA reaso

Re: A profound lack of profundity (and soon "the starting point")

2017-08-31 Thread David Nyman
On 31 Aug 2017 18:52, "Terren Suydam"  wrote:


On Wed, Aug 30, 2017 at 7:19 PM, John Clark  wrote:

> On Wed, Aug 30, 2017  Terren Suydam  wrote:
>
>
>> ​>
 ​>> ​
 ​
 All that's necessary is to imagine or simulate the first person
 perspective of the one who gets duplicated.

>>>
>>> ​
>>> ​>> ​
>>> Which ​ONE
>>>
>>> ​"*THE​*
>>> first person perspective
>>> ​" is ​
>>> Terren Suydam
>>> ​ referring to ​
>>>
>>> ​"*THE​*
>>>  first person perspective
>>> ​" in Moscow or
>>> ​"*THE​*
>>>  first person perspective
>>> ​"  in Washington or
>>> ​
>>> ​"*THE​*
>>>  first person perspective
>>> ​" in Helsinki?
>>>
>>
>> ​> ​
>> The one about to enter the duplication machine. What happens when you
>> simulate that person's first-person experience?
>>
>
> ​If he's about to enter the machine but hasn't yet done so then you're
> simulating the Helsinki man. The simulation shows that IF the Helsinki man
> is rational (and he might not be especially on matters like this, in fact
> most people aren't as this list proves) then the Helsinki man expects that
> very soon TWO people will remember being the Helsinki man but neither of
> them will be in Helsinki. What else do you want to know, what else is there
> to predict? ​
>
> ​
>
> John K Clark​
>
>
Let's dumb it down and forget about prediction and rationality for the
moment. Let's say instead of making two copies, we're just dealing with a
teleporter that destroys you in Helsinki and duplicates you in Barcelona.
Let's just focus on what your inner experience is like as you progress
through the duplicator.

You enter the duplicator - in Helsinki let's say the duplication chamber is
red, and you notice this. Then the duplication happens and you open your
eyes and the walls are now blue, since the chamber in Barcelona is of
course blue.

Do you have any problem with any of this?


And so, you too cross the event horizon.


-- 
You received this message because you are subscribed to the Google Groups
"Everything List" group.
To unsubscribe from this group and stop receiving emails from it, send an
email to everything-list+unsubscr...@googlegroups.com.
To post to this group, send email to everything-list@googlegroups.com.
Visit this group at https://groups.google.com/group/everything-list.
For more options, visit https://groups.google.com/d/optout.

-- 
You received this message because you are subscribed to the Google Groups 
"Everything List" group.
To unsubscribe from this group and stop receiving emails from it, send an email 
to everything-list+unsubscr...@googlegroups.com.
To post to this group, send email to everything-list@googlegroups.com.
Visit this group at https://groups.google.com/group/everything-list.
For more options, visit https://groups.google.com/d/optout.


Re: Is math real?

2017-08-31 Thread David Nyman
On 29 Aug 2017 04:39, "Brent Meeker" <meeke...@verizon.net> wrote:



On 8/28/2017 10:50 AM, Bruno Marchal wrote:


On 28 Aug 2017, at 02:44, Brent Meeker wrote:



On 8/27/2017 10:50 AM, David Nyman wrote:

On 25 August 2017 at 21:51, Brent Meeker <meeke...@verizon.net> wrote:

>
>
> On 8/25/2017 9:44 AM, Bruno Marchal wrote:
>
>>
>> On 24 Aug 2017, at 20:57, Brent Meeker wrote:
>>
>>
>>>
>>> On 8/24/2017 1:20 AM, Bruno Marchal wrote:
>>>
>>>>
>>>> On 23 Aug 2017, at 20:43, Brent Meeker wrote:
>>>>
>>>>
>>>>>
>>>>> On 8/23/2017 2:06 AM, Bruno Marchal wrote:
>>>>>
>>>>>> I am not someone proposing any new theory. I am someone showing that
>>>>>> the current materialist metaphysics just can't work with the Mechanist
>>>>>> hypothesis.
>>>>>>
>>>>>
>>>>> Refresh my understanding.  What it the mechanist hyposthesis? Is it
>>>>> the same as computationalism?
>>>>>
>>>>
>>>> Yes.
>>>>
>>>> Computationalism = Digital Mechanism = Mechanism = (Yes-Doctor +
>>>> Church's Thesis)
>>>>
>>>>
>>>>
>>>>
>>>> Or is it the same as yes-doctor plus reifying arithmetic?
>>>>>
>>>>
>>>> No, it is (yes-doctor + Church's Thesis).
>>>>
>>>> I do not add since long "Arithmetical Realism" because many people tend
>>>> to put to much into it, and is actually redundant with Church's thesis. To
>>>> just understand Church's thesis automatically assume we believe in some
>>>> "essentially undecidable theory", and this is equiavalent with believing in
>>>> the right amount of arithmetic.
>>>> I will write a post on the detailed starting point of the mathematics
>>>> needed to derive physics from "machine's theology".
>>>>
>>>>
>>>>
>>>>
>>>>
>>>> >From your use, these all seem slightly different to me.  It would be
>>>>> helpful to some firm definitions - not just usage.
>>>>>
>>>>
>>>> I use them as completely equivalent, although in the literature they
>>>> are usually stronger. Putnam's functionalism is a version of Digital
>>>> Mechanism which assumes a substitition level rather high, where my version
>>>> just ask for the existence of a substitution level. My version is the
>>>> weaker form possible, and Maudlin, in his Olympia paper, suggests that if
>>>> we define mechanism in this way, it becomes trivial, a bit like Diderot
>>>> defined "rationalism" by Descartes' Mechanism.
>>>>
>>>> So a firm definition of Mechanism (in my weak sense) is
>>>>
>>>> 1) Church's Thesis (a function from N to N is computable iff it exists
>>>> a combinator which computes it)
>>>>
>>>> (There are many variants of this. You can replace also "combinator"
>>>> by "game of life pattern", or "fortran program" or "c++ program", or
>>>> "quantum computer" etc.). Note that this asks for "Arithmetical realism"
>>>> which is only the believe that the RA axioms makes "absolute sense", which
>>>> means basically that not only 17 is prime, but that this is true
>>>> independently of me, you, or anyone, or anything physical. All
>>>> mathematicians are arithmetical realist. The fight on realism is in
>>>> Analysis or set theory, not arithmetic, especially without induction axiom
>>>> like with RA. Even a quasi ultra-finitist like Nelson agrees with RA.
>>>>
>>>>
>>>> 2) Yes-Doctor (= my consciousness is invariant for a digital physical
>>>> brain transplant made at some level of description of my (generalized)
>>>> brain.
>>>>
>>>> It asserts the existence of that substitution level, and is equivalent
>>>> with accepting that we can use classical teleportation as a mean of travel
>>>> (UDA step 1).
>>>>
>>>> Important Remark: that definition does not ask for surviving without a
>>>> physical brain/machine. That is indeed the object of the UDA reasoning:
>>>> showing that we cannot invoke God, or Primary-Matter to block the
>>>> immaterialist consequence of

Re: Is math real?

2017-08-28 Thread David Nyman
On 28 August 2017 at 01:49, Brent Meeker  wrote:

>
>
> On 8/27/2017 9:11 AM, Bruno Marchal wrote:
>
>>
>> I think it is more pleasing when you can build the virtuous circle of
>> explanations out of simple ideas that we hardly doubt at the start, like 2
>> * 12 = 24. And then, the point is that we have to do that, when we take
>> Mechanism seriously enough. We are back to Pythagoras, but with the
>> discovery of the universal machine and its quantum echo, and a
>> mathematically precise theology, containing physics, making it testable.
>>
>
> But you're still trying to make arithmetic the really really primary;
> whereas from the virtuous circle perspective it is the product of sapient
> thought.


​But that explanation wouldn't really be virtuous in the relevant sense,
would it? Arithmetic, seen merely as the product of thought, could hardly
at the same time be asserted as the ontological basis of that very thought,
could it? Unless your notion of the virtuous circle is something like
Escher's hands drawing each other. Well, in a metaphorical sense I guess it
could be seen like that. The arithmetic that is the product of thought is
certainly related to the arithmetic which ultimately may be assumed to give
rise to it. That's an idea worth taking seriously, but perhaps not too
literally.

Let's remember that 'primary' here means only what must be assumed, for the
purpose of explanation, rather than derived. That's all. So physics, in
this mode of explanation, isn't primary because it is to be derived or
inferred, not asserted; arithmetic, on the other hand, is assumed without
further justification.

>
>
>
>> Physics is not a problem. Physicalist metaphysics is a problem, when we
>> assume Mechanism. But apparently, Mechanism explains it by showing that if
>> true, the physical reality is in the head of all universal machine or
>> number, and that can be tested.
>>
>
> But the universal machine can only have a "head" in a certain kind of
> physical world...on which will via evolution inevitably produce mind.


​Yes indeed and no problem with this. I suspect that the three of us may be
quite near closing this argumentative circle (heaven forfend!)​. Let's
remember that the thing starts with the rather widespread (though often
only implicit) assumption that our mental processes ultimately depend on no
more than some species of classical computation. The practicable
feasibility of replacing biological brains and/or bodies wholly or in part
with particular alternative prostheses is not really germane to the
argument, but rather stands as a proxy for the essential claim of the
theory. This primary assumption then entails that computation, or rather
its irreducible basis in some tractable form, stands as our sole
ontological assumption and that the appearance of a concrete reality will
then depend on the development of a theory of knowledge based on the
generic or universal machine as sole agent or subject. This further
necessitates that the observable or phenomenological aspect of physics
falls into the epistemological compartment of the theory, and that its
unobservable or inferred component is treated as an observationally
self-selected (and hence canonical) subset of the deeper computational
ontology. That's it, I guess, in a nutshell. The rest is...well, it's
interesting to discuss, apparently.

David

>
> Brent
>
> --
> You received this message because you are subscribed to the Google Groups
> "Everything List" group.
> To unsubscribe from this group and stop receiving emails from it, send an
> email to everything-list+unsubscr...@googlegroups.com.
> To post to this group, send email to everything-list@googlegroups.com.
> Visit this group at https://groups.google.com/group/everything-list.
> For more options, visit https://groups.google.com/d/optout.
>

-- 
You received this message because you are subscribed to the Google Groups 
"Everything List" group.
To unsubscribe from this group and stop receiving emails from it, send an email 
to everything-list+unsubscr...@googlegroups.com.
To post to this group, send email to everything-list@googlegroups.com.
Visit this group at https://groups.google.com/group/everything-list.
For more options, visit https://groups.google.com/d/optout.


Re: Is math real?

2017-08-28 Thread David Nyman
On 28 August 2017 at 01:44, Brent Meeker <meeke...@verizon.net> wrote:

>
>
> On 8/27/2017 10:50 AM, David Nyman wrote:
>
> On 25 August 2017 at 21:51, Brent Meeker <meeke...@verizon.net> wrote:
>
>>
>>
>> On 8/25/2017 9:44 AM, Bruno Marchal wrote:
>>
>>>
>>> On 24 Aug 2017, at 20:57, Brent Meeker wrote:
>>>
>>>
>>>>
>>>> On 8/24/2017 1:20 AM, Bruno Marchal wrote:
>>>>
>>>>>
>>>>> On 23 Aug 2017, at 20:43, Brent Meeker wrote:
>>>>>
>>>>>
>>>>>>
>>>>>> On 8/23/2017 2:06 AM, Bruno Marchal wrote:
>>>>>>
>>>>>>> I am not someone proposing any new theory. I am someone showing that
>>>>>>> the current materialist metaphysics just can't work with the Mechanist
>>>>>>> hypothesis.
>>>>>>>
>>>>>>
>>>>>> Refresh my understanding.  What it the mechanist hyposthesis? Is it
>>>>>> the same as computationalism?
>>>>>>
>>>>>
>>>>> Yes.
>>>>>
>>>>> Computationalism = Digital Mechanism = Mechanism = (Yes-Doctor +
>>>>> Church's Thesis)
>>>>>
>>>>>
>>>>>
>>>>>
>>>>> Or is it the same as yes-doctor plus reifying arithmetic?
>>>>>>
>>>>>
>>>>> No, it is (yes-doctor + Church's Thesis).
>>>>>
>>>>> I do not add since long "Arithmetical Realism" because many people
>>>>> tend to put to much into it, and is actually redundant with Church's
>>>>> thesis. To just understand Church's thesis automatically assume we believe
>>>>> in some "essentially undecidable theory", and this is equiavalent with
>>>>> believing in the right amount of arithmetic.
>>>>> I will write a post on the detailed starting point of the mathematics
>>>>> needed to derive physics from "machine's theology".
>>>>>
>>>>>
>>>>>
>>>>>
>>>>>
>>>>> >From your use, these all seem slightly different to me.  It would be
>>>>>> helpful to some firm definitions - not just usage.
>>>>>>
>>>>>
>>>>> I use them as completely equivalent, although in the literature they
>>>>> are usually stronger. Putnam's functionalism is a version of Digital
>>>>> Mechanism which assumes a substitition level rather high, where my version
>>>>> just ask for the existence of a substitution level. My version is the
>>>>> weaker form possible, and Maudlin, in his Olympia paper, suggests that if
>>>>> we define mechanism in this way, it becomes trivial, a bit like Diderot
>>>>> defined "rationalism" by Descartes' Mechanism.
>>>>>
>>>>> So a firm definition of Mechanism (in my weak sense) is
>>>>>
>>>>> 1) Church's Thesis (a function from N to N is computable iff it exists
>>>>> a combinator which computes it)
>>>>>
>>>>> (There are many variants of this. You can replace also
>>>>> "combinator" by "game of life pattern", or "fortran program" or "c++
>>>>> program", or "quantum computer" etc.). Note that this asks for
>>>>> "Arithmetical realism" which is only the believe that the RA axioms makes
>>>>> "absolute sense", which means basically that not only 17 is prime, but 
>>>>> that
>>>>> this is true independently of me, you, or anyone, or anything physical. 
>>>>> All
>>>>> mathematicians are arithmetical realist. The fight on realism is in
>>>>> Analysis or set theory, not arithmetic, especially without induction axiom
>>>>> like with RA. Even a quasi ultra-finitist like Nelson agrees with RA.
>>>>>
>>>>>
>>>>> 2) Yes-Doctor (= my consciousness is invariant for a digital physical
>>>>> brain transplant made at some level of description of my (generalized)
>>>>> brain.
>>>>>
>>>>> It asserts the existence of that substitution level, and is equivalent
>>>>> with accepting that we can use classical teleportation as a mean of travel
>>>>> (UDA step 1).
&g

Re: Is math real?

2017-08-27 Thread David Nyman
On 25 August 2017 at 21:51, Brent Meeker  wrote:

>
>
> On 8/25/2017 9:44 AM, Bruno Marchal wrote:
>
>>
>> On 24 Aug 2017, at 20:57, Brent Meeker wrote:
>>
>>
>>>
>>> On 8/24/2017 1:20 AM, Bruno Marchal wrote:
>>>

 On 23 Aug 2017, at 20:43, Brent Meeker wrote:


>
> On 8/23/2017 2:06 AM, Bruno Marchal wrote:
>
>> I am not someone proposing any new theory. I am someone showing that
>> the current materialist metaphysics just can't work with the Mechanist
>> hypothesis.
>>
>
> Refresh my understanding.  What it the mechanist hyposthesis? Is it
> the same as computationalism?
>

 Yes.

 Computationalism = Digital Mechanism = Mechanism = (Yes-Doctor +
 Church's Thesis)




 Or is it the same as yes-doctor plus reifying arithmetic?
>

 No, it is (yes-doctor + Church's Thesis).

 I do not add since long "Arithmetical Realism" because many people tend
 to put to much into it, and is actually redundant with Church's thesis. To
 just understand Church's thesis automatically assume we believe in some
 "essentially undecidable theory", and this is equiavalent with believing in
 the right amount of arithmetic.
 I will write a post on the detailed starting point of the mathematics
 needed to derive physics from "machine's theology".





 From your use, these all seem slightly different to me.  It would be
> helpful to some firm definitions - not just usage.
>

 I use them as completely equivalent, although in the literature they
 are usually stronger. Putnam's functionalism is a version of Digital
 Mechanism which assumes a substitition level rather high, where my version
 just ask for the existence of a substitution level. My version is the
 weaker form possible, and Maudlin, in his Olympia paper, suggests that if
 we define mechanism in this way, it becomes trivial, a bit like Diderot
 defined "rationalism" by Descartes' Mechanism.

 So a firm definition of Mechanism (in my weak sense) is

 1) Church's Thesis (a function from N to N is computable iff it exists
 a combinator which computes it)

 (There are many variants of this. You can replace also "combinator"
 by "game of life pattern", or "fortran program" or "c++ program", or
 "quantum computer" etc.). Note that this asks for "Arithmetical realism"
 which is only the believe that the RA axioms makes "absolute sense", which
 means basically that not only 17 is prime, but that this is true
 independently of me, you, or anyone, or anything physical. All
 mathematicians are arithmetical realist. The fight on realism is in
 Analysis or set theory, not arithmetic, especially without induction axiom
 like with RA. Even a quasi ultra-finitist like Nelson agrees with RA.


 2) Yes-Doctor (= my consciousness is invariant for a digital physical
 brain transplant made at some level of description of my (generalized)
 brain.

 It asserts the existence of that substitution level, and is equivalent
 with accepting that we can use classical teleportation as a mean of travel
 (UDA step 1).

 Important Remark: that definition does not ask for surviving without a
 physical brain/machine. That is indeed the object of the UDA reasoning:
 showing that we cannot invoke God, or Primary-Matter to block the
 immaterialist consequence of Digital Mechanism.

>>>
>>> That's where I think some imprecision sneaks in.  Yes-doctor was
>>> originally presented as substituting some digitally simulated nuerons in
>>> the brain.  But then it was generalized to the whole brain.  But we think
>>> with more than our brain.  Our body contributes hormones and afferent and
>>> efferent nerve impluses. And the environment provides stimulation to those
>>> nerves and an arena within which we act.  All that is taken for granted in
>>> answering "yes doctor" or teletransporting.  So it appears to me that you
>>> implicitly suppose all of this is also digitally replaced.
>>>
>>
>> The reasoning does not depend on the substitution level.
>>
>> My version of mechanism is much weaker than all the others. I assume only
>> the existence of a substitution level (such that your conscious experience
>> would remain invariant for a digital substitution made at that level).
>>
>> If you want, you can take the Heinsenberg matrix of the whole observable
>> physical reality, at the level of the (super)-strings, with
>> 10^(10^(10^1000)) decimals exact for the complex numbers and real numbers
>> involved. The thought experience become harder to imagine, but eventually,
>> it is "the real experience" of the step 7 which we have to take into
>> account, that is "us" confronted to all computations in the arithmetical
>> reality. The arithmetical reality emulates all 

Re: Is math real?

2017-08-20 Thread David Nyman
On 20 Aug 2017 23:16, "Brent Meeker" <meeke...@verizon.net> wrote:



On 8/20/2017 9:23 AM, Bruno Marchal wrote:


On 20 Aug 2017, at 17:24, David Nyman wrote:



On 20 Aug 2017 2:46 p.m., "Bruno Marchal" <marc...@ulb.ac.be> wrote:


On 19 Aug 2017, at 01:21, David Nyman wrote:

On 18 August 2017 at 18:13, Bruno Marchal <marc...@ulb.ac.be> wrote:

>
> On 18 Aug 2017, at 15:39, David Nyman wrote:
>
> He points at a mug and says that 'representations' (meaning numbers)
> aren't to be confused with things themselves.
>
>
>
> He confuses a number and a possible representation of a number.
>
> LIke many people confuse the (usual, standard) arithmetical reality with a
> theory of the arithmetical reality. Yet after Gödel we know that no
> theories at all can represent or encompass the whole of the arithmetical
> reality.
>
> It is not much different that confusing a telescope and a star, or a
> microscope and a bacteria, or a finger and a moon, or a number and a
> numeral ("chiffre" in french).
> But in math, it is quite frequent. In logic, such distinction are very
> important. In Gödel's proof, we need to distinguish a mathematical being,
> like the number s(0), the representation of the number s(0), which is the
> sequence of the symbol "s", "(", "0", ")" (and that is not a number, but a
> word), and the representation of the representation of a number, which,
> when we represent things in arithmetic will be something like
> 2^3 * 3^4 * 5^5 *7^6, which will be some s(s(s(s(s(s(s(s(s(s(s(s(s(s(s(
> (0)...). (very long!).
>
>
> But what is the 'thing itself' at which he points?
>
>
> A mug. I guess.
>

​Just so.


The question will be "what is a mug in itself". A materialist would say
that it is a structured collection of atoms, but a mechanist has to say
something like "a common pattern pointed at by some normal (in Gauss sense)
machine sharing some long (deep) histories. Something like that.


Yeah, something like that. I enjoyed Frenkel's talk actually. I like his
enthusiasm for mathematics. It's funny though he doesn't seem to appreciate
his implicit assumptions, or indeed that he is in fact expressing a
particular metaphysical position. Is math real? I mean, really real?
Trouble is, people assume that the answer is obvious, whether they think
it's yes or no.


We need only to agree on what we agree. The beauty of the Church's thesis,
is that it entails by "theoremata" the existence of the emulation of all
computations in elementary arithmetic.

(Just that fact, and computationalism, should make us doubt that we can
take a primary physical reality for granted: it is the dream argument with
a vengeance).

The question is not "is math real", but do you believe that 2+0= 2, and a
bit of logic.

I do not claim that the whole of philosophy or theology can become science,
but I do claim that if we assume mechanism, then by Church's thesis,
philosophy and theology becomes a science, even in the usual empiricist
sense.

There is something funny here. The theology of the machine is
ultra-non-empiricist, as the mystical machine claims that the whole truth
(including physics) is "in your head and nowhere else". ("you" = any
universal machine). But that is what makes the machine theology testable,
by comparing the physics in the head of any (sound) universal machine with
what we actually observed.


Are you claiming that there is a one-to-one map between true statements in
mathematics and what I experience??


Well, only if you happen to be God, perhaps.

The problem with everythingism is that one doesn't experience everything.


How would you know?

David




Math is real? Which math? I doubt that sincere people doubt arithmetic, and
I have never heard of parents who would have taken their kids out of a
school for the reason that hey have been taught that 2+2=4; neither in the
Western nor Eastern worlds.


I doubt the infinity of standard arithmetic.



Now, for limit and real numbers it is much less obvious. here intutionist
and classical philosophy diverge. With Mechanism, it is better to
considered analysis (and eventually physics) as universal machine mind
tools. Gödel's incompleteness justifies partially why the machine needs to
invent infinities to better figure out themselves. Before Gödel, most
mathematician, like Hilbert, were hoping that with the finite and the
symbolic we could justify the consistency of the use of the infinities, but
after Gödel we know that even with the infinities we cannot circumscribe
and justify the consistency of the finite and the symbolic.


All the more reason to classify them as fiction - not part of the really
real.

Brent


The root of the undecidability is the Turing-universality. With the
conceptual discovery of the universal machine

Re: Is math real?

2017-08-20 Thread David Nyman
On 20 Aug 2017 17:23, "Bruno Marchal" <marc...@ulb.ac.be> wrote:


On 20 Aug 2017, at 17:24, David Nyman wrote:



On 20 Aug 2017 2:46 p.m., "Bruno Marchal" <marc...@ulb.ac.be> wrote:


On 19 Aug 2017, at 01:21, David Nyman wrote:

On 18 August 2017 at 18:13, Bruno Marchal <marc...@ulb.ac.be> wrote:

>
> On 18 Aug 2017, at 15:39, David Nyman wrote:
>
> He points at a mug and says that 'representations' (meaning numbers)
> aren't to be confused with things themselves.
>
>
>
> He confuses a number and a possible representation of a number.
>
> LIke many people confuse the (usual, standard) arithmetical reality with a
> theory of the arithmetical reality. Yet after Gödel we know that no
> theories at all can represent or encompass the whole of the arithmetical
> reality.
>
> It is not much different that confusing a telescope and a star, or a
> microscope and a bacteria, or a finger and a moon, or a number and a
> numeral ("chiffre" in french).
> But in math, it is quite frequent. In logic, such distinction are very
> important. In Gödel's proof, we need to distinguish a mathematical being,
> like the number s(0), the representation of the number s(0), which is the
> sequence of the symbol "s", "(", "0", ")" (and that is not a number, but a
> word), and the representation of the representation of a number, which,
> when we represent things in arithmetic will be something like
> 2^3 * 3^4 * 5^5 *7^6, which will be some s(s(s(s(s(s(s(s(s(s(s(s(s(s(s(
> (0)...). (very long!).
>
>
> But what is the 'thing itself' at which he points?
>
>
> A mug. I guess.
>

​Just so.


The question will be "what is a mug in itself". A materialist would say
that it is a structured collection of atoms, but a mechanist has to say
something like "a common pattern pointed at by some normal (in Gauss sense)
machine sharing some long (deep) histories. Something like that.


Yeah, something like that. I enjoyed Frenkel's talk actually. I like his
enthusiasm for mathematics. It's funny though he doesn't seem to appreciate
his implicit assumptions, or indeed that he is in fact expressing a
particular metaphysical position. Is math real? I mean, really real?
Trouble is, people assume that the answer is obvious, whether they think
it's yes or no.


We need only to agree on what we agree.


It's taken me years to appreciate this fully. The funny thing is that when
you say this to people they often object that this is the case only in
mathematics or logic, not in the 'real' world. But actually we can never
avoid the fact that we are always reasoning in terms of the assumptions of
some theory or other, whether it's explicit or not (usually not). So we
need to agree on what we agree, indeed.

The beauty of the Church's thesis, is that it entails by "theoremata" the
existence of the emulation of all computations in elementary arithmetic.


We can indeed agree to agree on that.


(Just that fact, and computationalism, should make us doubt that we can
take a primary physical reality for granted: it is the dream argument with
a vengeance).


Indeed.


The question is not "is math real", but do you believe that 2+0= 2, and a
bit of logic.


Difficult to disagree with.


I do not claim that the whole of philosophy or theology can become science,
but I do claim that if we assume mechanism, then by Church's thesis,
philosophy and theology becomes a science, even in the usual empiricist
sense.


About time too.


There is something funny here. The theology of the machine is
ultra-non-empiricist, as the mystical machine claims that the whole truth
(including physics) is "in your head and nowhere else". ("you" = any
universal machine). But that is what makes the machine theology testable,
by comparing the physics in the head of any (sound) universal machine with
what we actually observed.


Easier said than done though.


Math is real? Which math? I doubt that sincere people doubt arithmetic, and
I have never heard of parents who would have taken their kids out of a
school for the reason that hey have been taught that 2+2=4; neither in the
Western nor Eastern worlds.


Possibly in Airstrip One.


Now, for limit and real numbers it is much less obvious. here intutionist
and classical philosophy diverge. With Mechanism, it is better to
considered analysis (and eventually physics) as universal machine mind
tools. Gödel's incompleteness justifies partially why the machine needs to
invent infinities to better figure out themselves. Before Gödel, most
mathematician, like Hilbert, were hoping that with the finite and the
symbolic we could justify the consistency of the use of the infinities, but
after Gödel we know that even with the infinities we cannot circumscribe
and justify the consistency of the finite and t

Re: Is math real?

2017-08-20 Thread David Nyman
On 20 Aug 2017 2:46 p.m., "Bruno Marchal" <marc...@ulb.ac.be> wrote:


On 19 Aug 2017, at 01:21, David Nyman wrote:

On 18 August 2017 at 18:13, Bruno Marchal <marc...@ulb.ac.be> wrote:

>
> On 18 Aug 2017, at 15:39, David Nyman wrote:
>
> He points at a mug and says that 'representations' (meaning numbers)
> aren't to be confused with things themselves.
>
>
>
> He confuses a number and a possible representation of a number.
>
> LIke many people confuse the (usual, standard) arithmetical reality with a
> theory of the arithmetical reality. Yet after Gödel we know that no
> theories at all can represent or encompass the whole of the arithmetical
> reality.
>
> It is not much different that confusing a telescope and a star, or a
> microscope and a bacteria, or a finger and a moon, or a number and a
> numeral ("chiffre" in french).
> But in math, it is quite frequent. In logic, such distinction are very
> important. In Gödel's proof, we need to distinguish a mathematical being,
> like the number s(0), the representation of the number s(0), which is the
> sequence of the symbol "s", "(", "0", ")" (and that is not a number, but a
> word), and the representation of the representation of a number, which,
> when we represent things in arithmetic will be something like
> 2^3 * 3^4 * 5^5 *7^6, which will be some s(s(s(s(s(s(s(s(s(s(s(s(s(s(s(
> (0)...). (very long!).
>
>
> But what is the 'thing itself' at which he points?
>
>
> A mug. I guess.
>

​Just so.


The question will be "what is a mug in itself". A materialist would say
that it is a structured collection of atoms, but a mechanist has to say
something like "a common pattern pointed at by some normal (in Gauss sense)
machine sharing some long (deep) histories. Something like that.


Yeah, something like that. I enjoyed Frenkel's talk actually. I like his
enthusiasm for mathematics. It's funny though he doesn't seem to appreciate
his implicit assumptions, or indeed that he is in fact expressing a
particular metaphysical position. Is math real? I mean, really real?
Trouble is, people assume that the answer is obvious, whether they think
it's yes or no.

David



Bruno




David​

>
> Bruno
>
>
>
>
>
>
> https://futurism.com/the-most-important-question-underlying-
> artificial-general-intelligence-research-is-math-real/
>
> --
> You received this message because you are subscribed to the Google Groups
> "Everything List" group.
> To unsubscribe from this group and stop receiving emails from it, send an
> email to everything-list+unsubscr...@googlegroups.com.
> To post to this group, send email to everything-list@googlegroups.com.
> Visit this group at https://groups.google.com/group/everything-list.
> For more options, visit https://groups.google.com/d/optout.
>
>
> http://iridia.ulb.ac.be/~marchal/
>
>
>
>
> --
> You received this message because you are subscribed to the Google Groups
> "Everything List" group.
> To unsubscribe from this group and stop receiving emails from it, send an
> email to everything-list+unsubscr...@googlegroups.com.
> To post to this group, send email to everything-list@googlegroups.com.
> Visit this group at https://groups.google.com/group/everything-list.
> For more options, visit https://groups.google.com/d/optout.
>


-- 
You received this message because you are subscribed to the Google Groups
"Everything List" group.
To unsubscribe from this group and stop receiving emails from it, send an
email to everything-list+unsubscr...@googlegroups.com.
To post to this group, send email to everything-list@googlegroups.com.
Visit this group at https://groups.google.com/group/everything-list.
For more options, visit https://groups.google.com/d/optout.


http://iridia.ulb.ac.be/~marchal/



-- 
You received this message because you are subscribed to the Google Groups
"Everything List" group.
To unsubscribe from this group and stop receiving emails from it, send an
email to everything-list+unsubscr...@googlegroups.com.
To post to this group, send email to everything-list@googlegroups.com.
Visit this group at https://groups.google.com/group/everything-list.
For more options, visit https://groups.google.com/d/optout.

-- 
You received this message because you are subscribed to the Google Groups 
"Everything List" group.
To unsubscribe from this group and stop receiving emails from it, send an email 
to everything-list+unsubscr...@googlegroups.com.
To post to this group, send email to everything-list@googlegroups.com.
Visit this group at https://groups.google.com/group/everything-list.
For more options, visit https://groups.google.com/d/optout.


Re: Is math real?

2017-08-18 Thread David Nyman
On 18 August 2017 at 18:13, Bruno Marchal <marc...@ulb.ac.be> wrote:

>
> On 18 Aug 2017, at 15:39, David Nyman wrote:
>
> He points at a mug and says that 'representations' (meaning numbers)
> aren't to be confused with things themselves.
>
>
>
> He confuses a number and a possible representation of a number.
>
> LIke many people confuse the (usual, standard) arithmetical reality with a
> theory of the arithmetical reality. Yet after Gödel we know that no
> theories at all can represent or encompass the whole of the arithmetical
> reality.
>
> It is not much different that confusing a telescope and a star, or a
> microscope and a bacteria, or a finger and a moon, or a number and a
> numeral ("chiffre" in french).
> But in math, it is quite frequent. In logic, such distinction are very
> important. In Gödel's proof, we need to distinguish a mathematical being,
> like the number s(0), the representation of the number s(0), which is the
> sequence of the symbol "s", "(", "0", ")" (and that is not a number, but a
> word), and the representation of the representation of a number, which,
> when we represent things in arithmetic will be something like
> 2^3 * 3^4 * 5^5 *7^6, which will be some s(s(s(s(s(s(s(s(s(s(s(s(s(s(s(
> (0)...). (very long!).
>
>
> But what is the 'thing itself' at which he points?
>
>
> A mug. I guess.
>

​Just so.

David​

>
> Bruno
>
>
>
>
>
>
> https://futurism.com/the-most-important-question-underlying-
> artificial-general-intelligence-research-is-math-real/
>
> --
> You received this message because you are subscribed to the Google Groups
> "Everything List" group.
> To unsubscribe from this group and stop receiving emails from it, send an
> email to everything-list+unsubscr...@googlegroups.com.
> To post to this group, send email to everything-list@googlegroups.com.
> Visit this group at https://groups.google.com/group/everything-list.
> For more options, visit https://groups.google.com/d/optout.
>
>
> http://iridia.ulb.ac.be/~marchal/
>
>
>
> --
> You received this message because you are subscribed to the Google Groups
> "Everything List" group.
> To unsubscribe from this group and stop receiving emails from it, send an
> email to everything-list+unsubscr...@googlegroups.com.
> To post to this group, send email to everything-list@googlegroups.com.
> Visit this group at https://groups.google.com/group/everything-list.
> For more options, visit https://groups.google.com/d/optout.
>

-- 
You received this message because you are subscribed to the Google Groups 
"Everything List" group.
To unsubscribe from this group and stop receiving emails from it, send an email 
to everything-list+unsubscr...@googlegroups.com.
To post to this group, send email to everything-list@googlegroups.com.
Visit this group at https://groups.google.com/group/everything-list.
For more options, visit https://groups.google.com/d/optout.


Is math real?

2017-08-18 Thread David Nyman
He points at a mug and says that 'representations' (meaning numbers) aren't
to be confused with things themselves. But what is the 'thing itself' at
which he points?


https://futurism.com/the-most-important-question-underlying-artificial-general-intelligence-research-is-math-real/

-- 
You received this message because you are subscribed to the Google Groups 
"Everything List" group.
To unsubscribe from this group and stop receiving emails from it, send an email 
to everything-list+unsubscr...@googlegroups.com.
To post to this group, send email to everything-list@googlegroups.com.
Visit this group at https://groups.google.com/group/everything-list.
For more options, visit https://groups.google.com/d/optout.


Re: It from gBit

2017-08-01 Thread David Nyman
On 1 August 2017 at 01:14, Brent Meeker  wrote:

> Existence of an information unit as a postulate of quantum theory
>
> Lluis Masanes, Markus P. Mueller, Remigiusz Augusiak, David Perez-Garcia
> (Submitted on 2 Aug 2012 (v1), last revised 22 Oct 2013 (this version, v2))
>
> Does information play a significant role in the foundations of physics?
> Information is the abstraction that allows us to refer to the states of
> systems when we choose to ignore the systems themselves. This is only
> possible in very particular frameworks, like in classical or quantum
> theory, or more generally, whenever there exists an information unit such
> that the state of any system can be reversibly encoded in a sufficient
> number of such units. In this work we show how the abstract formalism of
> quantum theory can be deduced solely from the existence of an information
> unit with suitable properties, together with two further natural
> assumptions: the continuity and reversibility of dynamics, and the
> possibility of characterizing the state of a composite system by local
> measurements. This constitutes a new set of postulates for quantum theory
> with a simple and direct physical meaning, like the ones of special
> relativity or thermodynamics, and it articulates a strong connection
> between physics and information.
>


Interesting. And of course it implicitly sets up the question: what
​more ​
​could be said of
"systems themselves" beyond
​such an
​information-based account of
their "states"
​, should this indeed be determinative in the manner suggested​
? What, if anything,
​might be​

​"ignored" by, or left out of,
such an account?

David



>
> Journal reference:PNAS vol 110 no 41 page 16373 (2013)
> DOI:10.1073/pnas.1304884110
> Cite as:arXiv:1208.0493 [quant-ph]
>  (or arXiv:1208.0493v2 [quant-ph] for this version)
>
> --
> You received this message because you are subscribed to the Google Groups
> "Everything List" group.
> To unsubscribe from this group and stop receiving emails from it, send an
> email to everything-list+unsubscr...@googlegroups.com.
> To post to this group, send email to everything-list@googlegroups.com.
> Visit this group at https://groups.google.com/group/everything-list.
> For more options, visit https://groups.google.com/d/optout.
>

-- 
You received this message because you are subscribed to the Google Groups 
"Everything List" group.
To unsubscribe from this group and stop receiving emails from it, send an email 
to everything-list+unsubscr...@googlegroups.com.
To post to this group, send email to everything-list@googlegroups.com.
Visit this group at https://groups.google.com/group/everything-list.
For more options, visit https://groups.google.com/d/optout.


Re: Wallace on non-locality

2017-07-15 Thread David Nyman
On 15 July 2017 at 15:11, Bruno Marchal <marc...@ulb.ac.be> wrote:

>
> On 15 Jul 2017, at 14:20, David Nyman wrote:
>
> On 15 July 2017 at 09:26, Bruno Marchal <marc...@ulb.ac.be> wrote:
>
>>
>> On 15 Jul 2017, at 06:19, Brent Meeker wrote:
>>
>>
>>>
>>> On 7/14/2017 7:22 PM, Bruce Kellett wrote:
>>>
>>>> I think Brent has commented that consciousness needs a world to be
>>>> conscious of -- that doesn't, of itself, constrain that world in any way.
>>>> Even more so if the consciousness is a product of evolution in that world.
>>>> Evolution cannot produce anything that contradicts the laws of physics.
>>>>
>>>
>>> One way to see that consciousness and physics are not tightly
>>> constrained is to note that our consciousness seems adapted to living on a
>>> flat Earth in which Newtonian mechanics prevails.  The "laws of physics",
>>> as we've presently congratulated ourselves on inventing, are so far from
>>> intuitive that there are endless arguments over how to interpret them.
>>>
>>
>> John Clark misses this by stopping at UDA step 3 (not included), and
>> Bruce seems to stop at step 4 (not included).
>>
>
> ​Which particular 'this' do you have in mind here? The interpretation of
> the "laws of physics"?​
>
>
> By "this" I meant that consciousness and physics are tightly constrained.
>

​Ah, we agree then.


> Consciousness, including the observable (Physics) is made into a branch of
> machine arithmetical self-reference.
> ​ ​
> We, the universal machines, are distributed in an infinity of
> (arithmetical) computations below our substitution level on which our First
> Person experience is undetermined, and the laws to predict the observable
> must be given by a statistic on all computations, or it needs to invoke
> some metaphysical entity, be it Matter or God, doing a magical (non Turing
> emulable nor FPI recoverable) selection. That entails a "many computations"
> view of reality, and the quantum logical aspect of Nature is intuitively
> explained already.
>

>> Now, why negative amplitude of probability?
>
>
> Not sure I follow.  ​Negative amplitude of probability​
> ​is found in
>  harmonical functions​,
> ​is
> n't it​?
>
>
> Negative amplitude appears in all wave theory, but only in quantum
> mechanics do they have an interpretation in term of amplitude of
> probability. The whole "mystery" of quantum mechanics is there.
>
> I was alluding to the rather normal critics asserting that, a priori, in
> the UDA, or in classical probability theory, the probabilities on the next
> computational continuations are purely additive. There is no clue why the
> probabilities could be attached to some which undulate.
>

​Yes, now I see what you meant.
​

>
> But that critics makes a mistake. The probabilities are not on the 3p
> computations, but on the 1p experiences brought by those computations.
>

​A most crucial difference.
​

> So, to have the logic obeyed by those probabilities, we have to use a
> precise mathematical definition of "first person". This one is "offered" by
> the incompleteness theorem which makes sense of Theaetetus definition (in
> three different ways, giving three physical realms).
>

​Yes, and it puts into perspective the idea that the incompleteness
theorems might have been trivial in the sense that they are applicable only
to a small set of self-referential propositions. This is a criticism I have
encountered.
​


> Those notions of first person makes the antic dream argument valid,
>

​And thus reverses the apparent direction of explanation, at least at the
start, in a way that unfortunately seems to infuriate some people.
​

> explains entirely consciousness (if we agree to characterize it by the
> "non rationally justifiable yet undoubtable truth), associates an
> intuitionist subject to the knower, and, the main thing here, a quantum
> logic (and thus plausible negative amplitude of probability) for the
> observable.
>

​Yes, I see.

>
>
>> This remains counter-intuitive, but is explained by the fact that the
>> statistic are not on the 3p representational bodies in arithmetic, but on
>> the 1p views, which necessitate to be defined by machine-self-reference. In
>> that case, the non intuitive matter features are recovered by the
>> nonintuitive logic of self-reference and its 1p variants, and indeed, we
>> get a quantum logic which makes us expect a unique measure. It obeys, up to
>> now at least, everything needed to have those negative probability
>> amplitud

Re: Wallace on non-locality

2017-07-15 Thread David Nyman
On 15 July 2017 at 09:26, Bruno Marchal  wrote:

>
> On 15 Jul 2017, at 06:19, Brent Meeker wrote:
>
>
>>
>> On 7/14/2017 7:22 PM, Bruce Kellett wrote:
>>
>>> I think Brent has commented that consciousness needs a world to be
>>> conscious of -- that doesn't, of itself, constrain that world in any way.
>>> Even more so if the consciousness is a product of evolution in that world.
>>> Evolution cannot produce anything that contradicts the laws of physics.
>>>
>>
>> One way to see that consciousness and physics are not tightly constrained
>> is to note that our consciousness seems adapted to living on a flat Earth
>> in which Newtonian mechanics prevails.  The "laws of physics", as we've
>> presently congratulated ourselves on inventing, are so far from intuitive
>> that there are endless arguments over how to interpret them.
>>
>
> John Clark misses this by stopping at UDA step 3 (not included), and Bruce
> seems to stop at step 4 (not included).
>

​Which particular 'this' do you have in mind here? The interpretation of
the "laws of physics"?​


> We, the universal machines, are distributed in an infinity of
> (arithmetical) computations below our substitution level on which our First
> Person experience is undetermined, and the laws to predict the observable
> must be given by a statistic on all computations, or it needs to invoke
> some metaphysical entity, be it Matter or God, doing a magical (non Turing
> emulable nor FPI recoverable) selection. That entails a "many computations"
> view of reality, and the quantum logical aspect of Nature is intuitively
> explained already.
>
> Now, why negative amplitude of probability?


Not sure I follow.  ​Negative amplitude of probability​
​is found in
 harmonical functions​,
​is
n't it​?


> This remains counter-intuitive, but is explained by the fact that the
> statistic are not on the 3p representational bodies in arithmetic, but on
> the 1p views, which necessitate to be defined by machine-self-reference. In
> that case, the non intuitive matter features are recovered by the
> nonintuitive logic of self-reference and its 1p variants, and indeed, we
> get a quantum logic which makes us expect a unique measure. It obeys, up to
> now at least, everything needed to have those negative probability
> amplitudes, as required.
> ​ ​It is true that we can implement consciousness in a classical machine
> (assuming the substitution level enough high) but from its first person
> perspective, that machine will still experience the indeterminacy
> probabilities on all "classical and non classical" implementations in
> arithmetic.
>

> People are stuck on the brain-mind identity brain link, which is not
> available when we assume Mechanism. The first person is always associated
> with an infinity of implementations. We have to take into account the 1p/3p
> distinction. If a machine discovers that it run in a classical physical
> setting, the machine we interrogate will act like she knows that she
> belongs to a simulation or that computationalism is false.
>
> I understand that this is shocking, but normally, in this list, people
> should be used to it, because they accept Everett and the "everything" idea
> which should help them to be prepared for that admittedly unpleasant
> self-differentiation/self-multiplication. That is why I explain mechanism
> here, as people know already the work of Everett, and from a physicists
> standpoint, Mechanism generalizes its embedding of the subject's body in
> subject's matter of the theory, something already begun in arithmetic by
> Gödel, Kleene, Post, etc.
>

​Not only that, at first blush it looks as if it reverses entirely the
explanatory priority from mechanism-->mind to mind-->mechanism. But perhaps
in fact it creates a symmetry between the two modes of explanation. What I
mean is that if we were to simply accept mechanism on its own terms,
without extrapolating to mind, then we would have an 'explanation' entirely
in itself, or rather a machine that proceeds of itself without further need
of explanation. For example, if we conceive physics as such a mechanism,
the physical machine proceeds from state to state without the need of
invoking mind, or for that matter worlds, creatures, biological evolution,
or all the other mentalistic paraphernalia. On the other hand, should we be
'minded' to accept mind on its own terms, we are at once at a loss for a
mechanism for the phenomena that undoubtedly appear within its
observational spectra. As a slogan, we might assert that (from an
explanatory perspective) mechanism has no need for a mind, but mind is in
dire need of a mechanism. This, if you like, supplies a rationale for why
we look for such a mechanism and feel justified in deploying observational
self-selection as the filter on the generalised mechanistic base. Having
done so, we find ourselves left with the explanatory or implicative
symmetry mind<-->mechanism.

David


> Bruno
>
>
>
>
>
>
>
>> Brent
>>
>> --

Re: Wallace on non-locality

2017-07-15 Thread David Nyman
On 15 July 2017 at 05:19, Brent Meeker  wrote:

>
>
> On 7/14/2017 7:22 PM, Bruce Kellett wrote:
>
>> I think Brent has commented that consciousness needs a world to be
>> conscious of -- that doesn't, of itself, constrain that world in any way.
>> Even more so if the consciousness is a product of evolution in that world.
>> Evolution cannot produce anything that contradicts the laws of physics.
>>
>
> One way to see that consciousness and physics are not tightly constrained
> is to note that our consciousness seems adapted to living on a flat Earth
> in which Newtonian mechanics prevails.  The "laws of physics", as we've
> presently congratulated ourselves on inventing, are so far from intuitive
> that there are endless arguments over how to interpret them.


​You're bending the terminology away from the original argument. ​The
question isn't whether our naive beliefs about the nature of the world are
adapted precisely to its fundamental characteristics. Of course they aren't
- that's why scientific enquiry exists as a corrective. The question is
rather whether the fundamental nature of the physical world we observe with
the aid of scientific rigour is constrained to produce the conditions for
its own observation. This is very far from being a conjecture dreamed up by
yours truly, as you are of course aware.

David


>
> Brent
>
>
> --
> You received this message because you are subscribed to the Google Groups
> "Everything List" group.
> To unsubscribe from this group and stop receiving emails from it, send an
> email to everything-list+unsubscr...@googlegroups.com.
> To post to this group, send email to everything-list@googlegroups.com.
> Visit this group at https://groups.google.com/group/everything-list.
> For more options, visit https://groups.google.com/d/optout.
>

-- 
You received this message because you are subscribed to the Google Groups 
"Everything List" group.
To unsubscribe from this group and stop receiving emails from it, send an email 
to everything-list+unsubscr...@googlegroups.com.
To post to this group, send email to everything-list@googlegroups.com.
Visit this group at https://groups.google.com/group/everything-list.
For more options, visit https://groups.google.com/d/optout.


Re: Wallace on non-locality

2017-07-15 Thread David Nyman
On 15 July 2017 at 03:22, Bruce Kellett <bhkell...@optusnet.com.au> wrote:

> On 15/07/2017 10:34 am, David Nyman wrote:
>
> On 15 July 2017 at 00:46, Bruce Kellett < <bhkell...@optusnet.com.au>
> bhkell...@optusnet.com.au> wrote:
>
>>
>> On 15/07/2017 4:00 am, David Nyman wrote:
>>
>> ISTM, on reflection, that my tentative suggestion that the alternative
>> cases are in some deeper sense 'unphysical' have some sort of explanatory
>> force. It's circular of course, but not I think in a vicious way. IOW it
>> would be the case that there just were no well-formed physical
>> circumstances in which such encounters could occur.
>>
>>
>> Well yes, that is what the singlet state means -- there are no physical
>> circumstances in which the branches that form could violate QM. But that is
>> a statement of the fact, not an explanation of that fact.
>>
>
> ​Well, it's at least pointing in the direction​ of a mode of explanation
> in terms of a tight constraint on what could count as viably physical.
> Others have noted that physics (at least a physics adequate to support our
> kind of observer-hood) is suspiciously replete with such tight constraints.
> As I said, this would be a mystery if regarded merely as a brute given
> (i.e. it could simply have been otherwise) and hence it tends to point in
> the direction of a constraint on observational selection from a more
> general basis. And if so, it would of course then be the case that only
> appropriately-constrained phenomena, arising from such states of affairs as
> were physically capable of supporting observation, would thereby be
> observable.
>
>
> I think that this is looking at it in the wrong way. Things can look very
> constrained and unnatural if you reverse the direction of explanation. If
> you take the physical world, with all its complexity, as the starting
> point, and then understand human consciousness as having evolved in this
> environment, then there is no real mystery to be explained.
>

​I'm not sure I follow your logic here. The point I was making is ​that
"the physical world, with all its complexity" looks highly constrained,
something that is indeed widely noted - a put up job, to quote Fred Hoyle.
It has further been noted that such tightness of constraint might point to
its being a prerequisite for sentient life and hence for the conditions of
observation itself. Consequently, far from looking "very constrained and
unnatural if you reverse the direction of explanation", it's the opposite
that seems increasingly possible. IOW, things look less constrained and
unnatural if you take the view that the observed conditions are
observationally selected from a more generalised basis.

As to mysteries, take your pick. The one to which I was referring was
treating the complex physical world to which observation gives access as a
brute given. Of course this is a viable metaphysical position at which to
stop further enquiry. However, there remain motivations to look for a more
general basis from which such complexity might be derived, such as those we
are presently discussing. Such efforts cannot ultimately resolve the
mystery, of course, but they can perhaps simplify it.


>
>
>> ISTM however that any such notion implies a deeper level from which QM as
>> we observe it gets 'selected' on what is essentially an anthropic (hate
>> that term) basis.
>>
>>
>> If you want to go down this route you still have to give an account of
>> how this selection might come about -- why is it that violations of QM are
>> fatal to consciousness?
>>
>
> ​They would be fatal to consciousness if such violations precluded the
> formation of physical states of affairs adequate to comprise the vehicles
> of that consciousness and the environments on which their consistent
> experiences depended (as Brent has often noted).​
>
>
> I think Brent has commented that consciousness needs a world to be
> conscious of -- that doesn't, of itself, constrain that world in any way.
>

​I am speaking here of our actual experience, not some hypothetical
alternative. Consequently 'our' world is constrained to exhibit just those
phenomena that we actually observe and that support our physical existence
and that of our environment, and no others.
​

> Even more so if the consciousness is a product of evolution in that world.
> Evolution cannot produce anything that contradicts the laws of physics.
>

​My point precisely. We may also hope and trust that it hasn't produced in
us an intelligence that is condemned to be hopelessly deluded by its
scientific investigation of the nature of its physical environment.
​

>
>
>
>> This in turn would imply that an entangled Q

Re: Wallace on non-locality

2017-07-14 Thread David Nyman
On 15 July 2017 at 00:46, Bruce Kellett <bhkell...@optusnet.com.au> wrote:

>
>
> On 15/07/2017 4:00 am, David Nyman wrote:
>
> On 13 July 2017 at 12:12, Bruce Kellett < <bhkell...@optusnet.com.au>
> bhkell...@optusnet.com.au> wrote:
>
>
>> [Wallace's] summary is:
>> "The overall story about locality in Everettian quantum physics, then, is
>> this: the dynamics of the theory are local: there is no action at a
>> distance, and no clash with relativistic covariance. But quantum
>> entanglement means that a great deal of the information contained within
>> the quantum state is non-local, associated with large spatial regions but
>> not with any given subregion of those regions. As David Deutsch once put
>> it, quantum theory is a theory of local interactions and non-local states."
>>
>>  I cannot find anything in that summary to which I could object.
>>
>> Wallace than goes on to discuss some examples. His first example is of a
>> single particle system, such as Schrödinger's cat. He describes the initial
>> local interaction, and the spreading of the branching via decoherence. If
>> the cat is system A, and the environment with which it becomes entangled a
>> set of systems B_i, (i = 1,2,...), then the spread of the branching results
>> in a situation in which the individual states of the systems A and B_i
>> become mixed, but the combined state of (A ⋃ B) remains pure. "The state A
>> itself does not change at all in the process; what changes are the
>> non-local states of successively larger regions including A." The spreading
>> of entanglement by decoherence is thus a process that introduces a degree
>> of non-locality. Wallace illustrates this in his Figure 8.1 on page 307.
>>
>> Then we get to measurements on two independent particles, and, finally,
>> two entangled particles. Here Wallace really wimps out, and does not give
>> any systematic analysis. He does not consider the entangled singlet state
>> explicitly at all. All he says is that the entanglement between the
>> particle at A and the particle at B is a non-local property of A ⋃ B. "That
>> property propagates outwards, becoming a non-local property of the forward
>> light cone of A and that of B. Only in their interactions can it have
>> locally determinable effects--and it does, giving rise to the branch
>> weights which, in turn, give rise to the sorts of statistical result
>> recorded in Aspect's experiments and their successors: statistical results
>> which violate Bell's inequality."
>>
>> That is just the standard quantum account, since it is always accepted
>> that the correlations only become apparent when the results of measurements
>> by A and B are combined at some later time, when their light cones overlap.
>>
>> Wallace seems to find this relatively uninteresting. Bell's result
>> entails non-separability (non-locality), but  not action at a distance --
>> but then, no one said it did involve action at a distance. He then claims
>> that Bell's theorem does not apply to the Everett interpretation anyway,
>> because is assumes that experiments have unique, definite outcomes. That is
>> the usual MWI claim against Bell, but Bell's results are not specifically
>> quantum -- the inequalities obtain for any theory in which the measurements
>> at A and B are independent, so this passing swipe at Bell is rather
>> unnecessary.
>>
>> The upshot, it seems to me, is that Wallace acknowledges non-locality as
>> I have used the term, only he prefers to call it nonseparability. The
>> change in terminology does not change the physics, so Wallace accepts that
>> the Everettian interpretation of QM does not eliminate non-locality,
>> despite the claims of many MWI supporters.
>>
>
> ​The question then is what are we to make of such non-separability (I'll
> stick to this usage, for what it's worth) if it is not to imply a
> corresponding action at a distance?​ How can it be that the Alice who has
> observed, say, spin-up doesn't get to encounter the Bob who has also
> observed spin-up? In what does this so-called entanglement consist?
>
>
> This is where it is a pity that Wallace did not actually work through the
> entangled singlet state in detail. He gives a general expression for a
> state with two entangled particles in (8.14), but says: "In this case, the
> amplitudes of the four sets of branches into which C eventually branches
> are not determined simply by the separate weights of the branches at A and
> B. Not is this to be expected: as I stressed previously, in Everettian
> quantum mech

Re: Wallace on non-locality

2017-07-14 Thread David Nyman
On 13 July 2017 at 12:12, Bruce Kellett <bhkell...@optusnet.com.au> wrote:

I have recently acquired Wallace's book and read his section on
> non-locality, so I am re-opening this thread (under a different header) to
> discuss Wallace's views in more detail. More below..
>
>
> On 7/06/2017 9:24 am, David Nyman wrote:
>
> On 6 June 2017 at 01:46, Bruce Kellett < <bhkell...@optusnet.com.au>
> bhkell...@optusnet.com.au> wrote:
>
>> On 6/06/2017 10:21 am, David Nyman wrote:
>>
>>
>> ​Bruce, I'm reading The Emergent Multiverse by David Wallace at the
>> moment. He's well known as a prominent theorist of MWI. I don't know
>> whether he falls under your definition of competence in this area, but as
>> far as I've understood him, he fully accepts that MWI must be consistent
>> with QM in all respects, including of course nonlocality.​ The distinction
>> he makes is between nonlocality and the question of whether this requires
>> us to think in terms of instantaneous transfer of information at
>> greater-than-light speed, or "action at a distance". I can't say I've been
>> able to get my head around his full exposition of this yet, but I'm pretty
>> sure he doesn't  go along with your exposition of Price's seemingly faulty
>> version of this.
>>
>>
>> It is interesting that Wallace has come to this view. He, with Deutsch,
>> was one of those who attempted to argue that MWI restored full locality.
>> They also tried to derive the Born Rule from within MWI, and failed in that
>> too.
>>
>> I do not know the book you refer to, but if Wallace now accepts that QM
>> and Bell implies non-locality, then I fully agree. I have always argued, on
>> this list and elsewhere, that non-locality does not mean the instantaneous
>> transfer of physical information -- if you think about it, that would, in a
>> sense, be a local, albeit FTL, effect. The core of the quantum singlet
>> state is that it does not involve the physical positions of the particles.
>> It is expressed in configuration space, and the difficulties appear to
>> arise from interpreting configuration space as though it were the same as
>> ordinary 3-space. What has been said is that the singlet state is always
>> local in configuration space, which translates to non-locality in 3-space.
>> And this without some FTL information transfer. If there were FTL
>> information transfer, then that could be manipulated to give FTL
>> signalling, and there are all sorts of theorems in QM that show that FTL
>> signalling is not possible.
>>
>> But it seems as though Wallace is coming to see these things as do the
>> majority of other physicists -- non-locality is intrinsic to quantum
>> entanglement.
>>
>
> ​ Wallace uses the term non-separability. ​He makes an analogy, to a
> certain extent, with the ontology of field theories such as
> electromagnetism, about which he says "The structural complexity of a given
> electromagnetic field is represented not in the properties of very small
> spacetime regions (indeed in the limit as these regions become point sized,
> the field's structure becomes almost trivial) but in the way in which those
> properties vary across spacetime. Furthermore, this general model is
> characteristic of pretty much any classical field theory, except that
> vector fields seem mathematically tame compared to the sorts of
> mathematical objects used to represent the field values of many classical
> field theories.". He gives a number of examples of these latter objects
> including the affine connections of General Relativity. He then goes on
> from this analogy to propose an ontology for quantum field theory which he
> calls Spacetime State Realism. I can't really attempt to elaborate on this
> here.
>
> Moving on this basis to the question "Does Everettian quantum mechanics
> display action at a distance?" he answers in the negative. He justifies
> this by elaborating on the observation that "In a quantum field theory, the
> quantum state of any region depends on the quantum state of some cross
> section of the past light cone of that region. Disturbances cannot
> propagate into that light cone." To the question "Does Everettian quantum
> mechanics display non-separability?" he answers in the positive. He
> justifies this by elaborating on the observation that "Because of
> entanglement, knowing the density operators of regions A and B does not
> suffice to fix the density operator of (the union of) A and B. Some of the
> properties of (the union of) A and B are genuinely non-local: they have
> local physical manifes

Re: What lead to free-will denial?

2017-06-30 Thread David Nyman
On 30 Jun 2017 16:38, "Bruno Marchal" <marc...@ulb.ac.be> wrote:


On 29 Jun 2017, at 17:26, David Nyman wrote:



On 29 Jun 2017 15:58, "Bruno Marchal" <marc...@ulb.ac.be> wrote:


On 28 Jun 2017, at 16:03, John Clark wrote:

snip

>> Unicorns don't exist,
>
>

>In which theory?


Oh for christ's sake! This is really getting silly, now existence depends
on theories.



Of course not.


But *asserting* an existence depends on a theory or belief of some subject
doing the assertion.

 For example, you just said to Adrian Chira: "Assuming there is a largest
prime number leads to a logical contradiction, but assuming free will
exists is like assuming Klogknee exists."

But Robinson Arithmetic, a weak but already sigma_1 complete theory (and
thus Turing universal) is consistent with "there is a biggest prime
number". You can add that axiom to RA without getting any contradiction.
You were probably assuming Peano Arithmetic (which is Robinson arithmetic +
the induction axiom), or you were assuming the standard model of arithmetic
(which is a much stronger assumption than the axioms of PA).

Except for the here-and-now 1p experience (consciousness), all existence
assertions presupposes some belief, that is, an unconscious or a conscious
theory.

And the unicorn certainly exists. It is the national animal of Scotland!


On the contrary, as a proud Scot, I can assure you that the national animal
of Scotland is the haggis.


Well ...
Definitions of haggis
noun
a Scottish dish consisting of a sheep's or calf's offal mixed with suet,
oatmeal, and seasoning and boiled in a bag, traditionally one made from the
animal's stomach.
Modern haggis generally has beef suet rather than mutton fat, and cayenne
pepper or nutmeg are usual additions.

This is a merely reductive account that sweeps the true origin of the
haggis under the rug.

It is certainly more nutritive than a steak of unicorn (if that is legal).


It depends on the theory you're eating in.

David


Bruno








David


Bruno







John K Clark


-- 
You received this message because you are subscribed to the Google Groups
"Everything List" group.
To unsubscribe from this group and stop receiving emails from it, send an
email to everything-list+unsubscr...@googlegroups.com.
To post to this group, send email to everything-list@googlegroups.com.
Visit this group at https://groups.google.com/group/everything-list.
For more options, visit https://groups.google.com/d/optout.


http://iridia.ulb.ac.be/~marchal/




-- 
You received this message because you are subscribed to the Google Groups
"Everything List" group.
To unsubscribe from this group and stop receiving emails from it, send an
email to everything-list+unsubscr...@googlegroups.com.
To post to this group, send email to everything-list@googlegroups.com.
Visit this group at https://groups.google.com/group/everything-list.
For more options, visit https://groups.google.com/d/optout.



-- 
You received this message because you are subscribed to the Google Groups
"Everything List" group.
To unsubscribe from this group and stop receiving emails from it, send an
email to everything-list+unsubscr...@googlegroups.com.
To post to this group, send email to everything-list@googlegroups.com.
Visit this group at https://groups.google.com/group/everything-list.
For more options, visit https://groups.google.com/d/optout.


http://iridia.ulb.ac.be/~marchal/



-- 
You received this message because you are subscribed to the Google Groups
"Everything List" group.
To unsubscribe from this group and stop receiving emails from it, send an
email to everything-list+unsubscr...@googlegroups.com.
To post to this group, send email to everything-list@googlegroups.com.
Visit this group at https://groups.google.com/group/everything-list.
For more options, visit https://groups.google.com/d/optout.

-- 
You received this message because you are subscribed to the Google Groups 
"Everything List" group.
To unsubscribe from this group and stop receiving emails from it, send an email 
to everything-list+unsubscr...@googlegroups.com.
To post to this group, send email to everything-list@googlegroups.com.
Visit this group at https://groups.google.com/group/everything-list.
For more options, visit https://groups.google.com/d/optout.


Re: What lead to free-will denial?

2017-06-29 Thread David Nyman
On 29 Jun 2017 15:58, "Bruno Marchal"  wrote:


On 28 Jun 2017, at 16:03, John Clark wrote:

snip

>> Unicorns don't exist,
>
>

>In which theory?


Oh for christ's sake! This is really getting silly, now existence depends
on theories.



Of course not.


But *asserting* an existence depends on a theory or belief of some subject
doing the assertion.

 For example, you just said to Adrian Chira: "Assuming there is a largest
prime number leads to a logical contradiction, but assuming free will
exists is like assuming Klogknee exists."

But Robinson Arithmetic, a weak but already sigma_1 complete theory (and
thus Turing universal) is consistent with "there is a biggest prime
number". You can add that axiom to RA without getting any contradiction.
You were probably assuming Peano Arithmetic (which is Robinson arithmetic +
the induction axiom), or you were assuming the standard model of arithmetic
(which is a much stronger assumption than the axioms of PA).

Except for the here-and-now 1p experience (consciousness), all existence
assertions presupposes some belief, that is, an unconscious or a conscious
theory.

And the unicorn certainly exists. It is the national animal of Scotland!


On the contrary, as a proud Scot, I can assure you that the national animal
of Scotland is the haggis.

David


Bruno







John K Clark


-- 
You received this message because you are subscribed to the Google Groups
"Everything List" group.
To unsubscribe from this group and stop receiving emails from it, send an
email to everything-list+unsubscr...@googlegroups.com.
To post to this group, send email to everything-list@googlegroups.com.
Visit this group at https://groups.google.com/group/everything-list.
For more options, visit https://groups.google.com/d/optout.


http://iridia.ulb.ac.be/~marchal/



-- 
You received this message because you are subscribed to the Google Groups
"Everything List" group.
To unsubscribe from this group and stop receiving emails from it, send an
email to everything-list+unsubscr...@googlegroups.com.
To post to this group, send email to everything-list@googlegroups.com.
Visit this group at https://groups.google.com/group/everything-list.
For more options, visit https://groups.google.com/d/optout.

-- 
You received this message because you are subscribed to the Google Groups 
"Everything List" group.
To unsubscribe from this group and stop receiving emails from it, send an email 
to everything-list+unsubscr...@googlegroups.com.
To post to this group, send email to everything-list@googlegroups.com.
Visit this group at https://groups.google.com/group/everything-list.
For more options, visit https://groups.google.com/d/optout.


Re: “Could a Quantum Computer Have Subjective Experience?”

2017-06-25 Thread David Nyman
On 24 June 2017 at 11:36, Russell Standish  wrote:

> On Sat, Jun 24, 2017 at 06:29:54PM +1000, Bruce Kellett wrote:
> > On 24/06/2017 5:23 pm, Russell Standish wrote:
> >
> > OK, it was possibly the case that you gave arguments earlier in the
> > book. But I was going on the basis of the Appendix "Derivation of
> > Quantum postulates".
> >
> > But the problems only begin with the assumption of a probabilistic
> > model. Psi(t) is the set of possibilities consistent with what is
> > known at time t. But how do you limit this set? At the moment, I
> > could go to the pub for a drink, could open a bottle of wine at
> > home, stroke the cat, turn on the telly, talk to my wife, etc,
> > etc,. The possibilities consistent with what is known at this
> > time is not a well defined set, or limited in any way.
>
> The everything is the set of all infinite length strings, each of
> which describes a universe to infinite detail.


​I always wondered about this aspect of your theory Russell. I assume that
you mean "describes under some interpretation". If so, the only available
interpretation (whether explicitly computationalist, a la UDA, or
otherwise) would have to be intrinsic to the strings themselves. Hence some
interpretative scheme or other must also be implicitly assumed. Isn't that
so?

David

Some of these strings
> will describe universes compatible with our current observer moment -
> an infinite number even, as the information content of our OM is
> finite. Others will not. It is a well defined subset of the everything.
>
> >
> > Because you then go on to define projection operators in terms of a
> > sum over the members of this set of possible outcomes. That is
> > meaningless unless you are already assuming the the outcomes are
> > just possible results for a well-defined measurement, and that this
> > measurement process can be defined in a linear vector space.
> >
>
> Summation of the projection operators is defined in equation D.1 for
> disjoint observations a and b (ie where it is impossible to observe a
> and b simultaneously). Linearity is not assumed at this point.
>
> > Another problem occurs further down when you seem to have complex
> > numbers of observers observing an observer moment. Why you should
> > have more than one observer for any observer moment is a mystery yet
> > to be solved.
>
> It's more a measure over observer moments. In a branching multiverse,
> not all observer moments are equally likely, but one would expect
> across a branching point, measure should be conserved.
>
> Why the measure is complex, not real is more tricky. With the
> everything, subsets naturally induce a real valued measure. But we do
> know that complex measures are more general, and we need a good reason
> not to choose the most general. But complex measures are not the most
> general. I do say "more general division algebras cannot support
> equations of the form (D.7)", but I confess, I'm still not completely
> happy with that line.
>
> > But then you go on, in eq. B8 to define the inner
> > product in terms of the probability function. But you have merely
> > multiplied together two expansions in terms of projections over
> > possible outcomes -- assuming that there is a linear span over the
> > space in the process. This gives the Born rule, sure, because you
> > have built it into your derivation of the inner product.
> >
>
> By the time we get to equation D.8, we have proved that the set of
> observer moments is a vector space, so yes, this construction is
> allowed. We are entitled to define any real-valued bilinear operator on
> that space and call it an inner product.
>
> By using that particular inner product, you get the Born rule in the
> usual form. If we'd chosen another, we'd have a different expression
> that is equivalent to the Born rule.
>
> >
> > >>So you know about QM from the start, and devise a strategy to get
> > >>you there. One of the problems that many-worlders face in their
> > >>attempts to derive the Born rule from within MWI is that they cannot
> > >>independently justify a probabilistic model.
> > >Yes, but I don't start with the MWI (namely, I don't start with a
> > >Hilbert space and unitary equation of motion - ie Schroedinger's
> > >equation). I start with evolution in a generic multiverse.
> >
> > Why a multiverse? You no doubt argue for it elsewhere, but that is
> > not apparent in your quantum derivation.
> >
>
> Yes - of course. The whole book is premised on it.
>
> > And I do not understand why the most general equation for computing
> > psi as a function  of time is a first order differential equation.
> > The equation could clearly be non-linear in psi -- such things have
> > been postulated after all, as in general relativity and GRW for
> > instance.
> >
>
> A first order differential equation needn't be linear. Linearity comes
> from assuming that the laws of physics don't change every time you
> observe something, more specifically 

Re: “Could a Quantum Computer Have Subjective Experience?”

2017-06-22 Thread David Nyman
On 22 Jun 2017 2:46 a.m., "Bruce Kellett" <bhkell...@optusnet.com.au> wrote:

On 22/06/2017 10:32 am, David Nyman wrote:

On 22 Jun 2017 00:31, "Bruce Kellett" < <bhkell...@optusnet.com.au>
bhkell...@optusnet.com.au> wrote:

On 22/06/2017 1:44 am, Bruno Marchal wrote:

> On 21 Jun 2017, at 08:21, Bruce Kellett wrote:
>
>> On 21/06/2017 4:03 pm, Russell Standish wrote:
>>
>>> On Mon, Jun 19, 2017 at 12:15:31PM +1000, Bruce Kellett wrote:
>>>
>>>> On 19/06/2017 10:23 am, Russell Standish wrote:
>>>>
>>>>> I know Scott wouldn't go as far as me. For me, all such irreversible
>>>>> processes are related to conscious entities in some way. Whilst
>>>>> agreeing that Geiger counters are unlikely to be conscious, I would
>>>>> say that the output of Geiger counter is not actually discrete until
>>>>> observed by a conscious experimenter.
>>>>>
>>>> That sounds remarkably like the "many minds" interpretation of
>>>> quantum mechanics. This is disfavoured by most scientists because it
>>>> leaves the physics of the billions of years before the emergence of
>>>> the first "conscious" creature unresolved -- the first consciousness
>>>> would cause an almighty collapse on the many minds reading.
>>>>
>>>> Each consciousness causes "an almighty collapse" in er own mind
>>> independently of any other. It's a pure 1p phenomena.
>>>
>>
>> It is actually a 3p phenomenon because there is inter-subjective
>> agreement about the fact that measurements give definite results.
>>
>
> Inter-subjectivity does not imply 3p, as it can be "only" 1p plural. Let
> me illustrate this with a variant of the WM duplication.
>
> Imagine that Bruce and John are undergoing the WM-duplication *together*.
>
> By this I mean they both enter the scanning-annihilating box, and are both
> reconstituted in Washington and in Moscow.
>
> And let us assume they do it repetitively, which means they come back to
> Helsinki, and do it again together.
>
> Obviously, the line-life past that each copies describes in its personal
> diaries grows like H followed by a sequence of W and M. The number of
> copies grows exponentially (2^n). After ten iterations, we have 2^10 = 1024
> individuals, who share an indeterminate experiences. With minor exceptions,
> they all agree that the experience has always given each times a precise
> outcome, always belonging to {W, M}. Importantly  the duplicated couples
> agreed (which was the Washington or Moscow outcome) in all duplication.
> They mostly all agreed they did not found any obvious algorithm to predict
> the sequence (the exception might concerned the guys in nameable stories,
> like:
>
> WW
>
> MM
>
> Or the development of some remarkable real number in binary, like the
> binary expansion of PI, sqrt(2), sqr(n), etc. In this case, the computable
> is made rare (and more and more negligible when n grows, those histories
> are "white rabbits histories").
>
> That is what I mean by first person plural. It concerns population of
> machine sharing self-multiplication. it is interesting to compare the
> quantum linear self-superposition with the purely arithmetical one.
>

Sure, that would seem to be reasonably described as 1p-plural. except that
there is no need to have two people enter the duplicating machine and
undergo different teleportations afterwards. Surely it is sufficient to
consider one person doing a series of polarization measurements on a
sequence of photons from an unpolarized source. That person will record
some sequence of '+' and '-' results. If the experiment is repeated N
times, there will be 2^N sequences, one in each of the generated worlds.

But that has nothing to do with inter-subjective agreement between
different observers. To see that, consider just one polarization
measurement: In order for it to be said that the measurement gave a result,
there has to be decoherence and the formation of irreversible records. I
think it is Zurek who talks about multiple copies of the result entangled
with the environment. So many different individuals can observe the result
of this single experiment, and they will all agree that the result was what
the experimenter wrote in her lab book. That is inter-subjective agreement.
It clearly has nothing to do with 1p, or 1p-plural pictures.


I think there may be a terminological confusion here. IIUC, 1p-plural
denotes, amongst other things, just such inter-subjective agreement between
mutually entangled observers.


That seems a remarkably confusing terminology. The example Bruno gave

Re: “Could a Quantum Computer Have Subjective Experience?”

2017-06-21 Thread David Nyman
reement that is essential for
physics -- people have to agree that experiments have definite results, and
they have to agree what those results are. Inter-subjective agreement
occurs in just one world -- although there may be similar agreements
between copies of those people entangled be decoherence with the other
possible experimental results.  Each world is then characterized by
inter-subjective agreement about the result obtained in that world.

Again, this bears no relation to Tegmark's 'bird' view. You might well call
the bird view the 0p view, because there is no person or consciousness that
can ever experience that view.




There is no collapse at all at the 3p level, nor even decoherence as such.
>>>
>>
>> Decoherence is a well-understood physical phenomenon that has been widely
>> observed.
>>
>
> I can't agree more. It might be, and should be when assuming digital
> mechanism, a first person plurality phenomenon. In the (quantum) MW, is the
> fission/differentiation of histories brought by measurement, and the
> measurement itself is part of the histories.
>

As I have just explained at length, decoherence is not a 1p-plural view --
it is quite definitely a matter of entanglement in a single world giving
rise to inter-subjective agreement on the results of any particular
experiment.
Each 'world' in the many worlds picture is a separate decoherent history.



I do not know what you mean by saying "nor even decoherence as such."
>>
>
> Maybe Russell meant in the (3-1) view of the (assumed by Everett)
> Universal wave. Plausible. The universal wave describes a change of base.
> It is God's vision (in this still physicalist view).
>
> Everett, that is QM without the collapse axiom, looks already like a
> solution of the computationalist mind-body problem. But it works only if
> Everett QM is itself derivable from (intensional) arithmetic.
>

In that case you shouldn't be making pronouncements about what the physics
means until you have completed that derivation from arithmetic.


Also, you seem to be confusing the inter-subjective 3p view with Tegmark's
>> bird view. There is no person, body, or consciousness that ever has the
>> bird view -- the bird is a purely formal construct and has nothing to do
>> with mind or consciousness.
>>
>
> That is an interesting remark, but it is a highly debatable question. See
> my conversation with David Nyman, about the "the nature" of the 0p view: is
> it more 1p or 3p? Is it more like a thing or a person? Well, I don't know.
> Is the arithmetical reality conceivable as a person? You can see it has an
> infinite (and highly non mechanical) body of (arithmetical) knowledge, but
> this would be a poetical acknowledgment of our ignorance.
>

I can accept the characterization of the bird view as 0p -- but since it is
not experienced by anyone or anything, then it is neither 1p, 3p, nor
1p-plural.


Even though everything might remain unitary at that level, no one can ever
>> experience the consequences of that unitary evolution.
>>
>
> Hmm... You speculate that there is no global 1p for the global unitary
> evolution, which is an open problem to me. Hard to know.
>

Well, you can speculated about panpsychism if you wish, but since it would
have no observable consequences, the notion seems otiose to me.



Nevertheless, assuming QM, you do *experience* the *consequences* of the
> unitary evolution, right here and right now, directly, and indirectly, as
> you are using a machine whose miniaturization has been made possible by the
> QM laws + human inference of the QM laws.
>

Unitary evolution is a property of the equations, not of the experiences.
It is only ever inferred, not observed directly. The universal wave of the
multiverse is 0p -- there is no one or thing that ever experiences the
assumed unitary evolution of the universal wave, The fact that QM describes
many aspects of experience does not prove unitarity, because we interact
with quantum mechanical phenomena only at the 'classical' level, after
decoherence and FAPP collapse. Our experience is, in fact, entirely of
non-unitary behaviour -- experiments give unique results, not
superpositions in the measurement basis.

Bruce




> With mechanism, the QM laws have to be derived from the first person views
> emulated in elementary number theory, or from any first order
> Church-Turing-Post -Kleene equivalent theory.
>
> Bruno
>

-- 
You received this message because you are subscribed to the Google Groups
"Everything List" group.
To unsubscribe from this group and stop receiving emails from it, send an
email to everything-list+unsubscr...@googlegroups.com.
To post to this group, send email to everything-list@googlegroups.com.
Visit this group at https://groups.google.com/group/everything-list.
For more options, visit https://groups.google.com/d/optout.

-- 
You received this message because you are subscribed to the Google Groups 
"Everything List" group.
To unsubscribe from this group and stop receiving emails from it, send an email 
to everything-list+unsubscr...@googlegroups.com.
To post to this group, send email to everything-list@googlegroups.com.
Visit this group at https://groups.google.com/group/everything-list.
For more options, visit https://groups.google.com/d/optout.


Re: A thought on MWI and its alternative(s)

2017-06-11 Thread David Nyman
On 11 Jun 2017 16:44, "Bruno Marchal" <marc...@ulb.ac.be> wrote:


On 11 Jun 2017, at 12:24, David Nyman wrote:

On 11 June 2017 at 10:14, Bruno Marchal <marc...@ulb.ac.be> wrote:

>
> On 09 Jun 2017, at 20:21, David Nyman wrote:
>
> On 9 June 2017 at 12:34, Bruno Marchal <marc...@ulb.ac.be> wrote:
>
>>
>> On 08 Jun 2017, at 02:05, Bruce Kellett wrote:
>>
>> On 7/06/2017 10:38 pm, Bruno Marchal wrote:
>>>
>>>> On 07 Jun 2017, at 11:42, Bruce Kellett wrote:
>>>> On 7/06/2017 7:09 pm, Bruno Marchal wrote:
>>>>
>>>>> On 06 Jun 2017, at 01:23, Bruce Kellett wrote:
>>>>>>
>>>>>> I have been through this before. I looked at Price again this morning
>>>>>>> and was frankly appalled at the stupidity of what I saw.
>>>>>>> Let me summarize briefly what he did. He has a very cumbersome
>>>>>>> notation, but I will attempt to simplify as far as is possible. I will 
>>>>>>> use
>>>>>>> '+' and '-' as spin states, rather than his 'left', 'right'.
>>>>>>>
>>>>>>> He write the initial wave function as for the case when you and I
>>>>>>> agree in advance to have aligned polarizers:
>>>>>>>
>>>>>>> |psi_1> = }me, electrons,you> = |me>(|+-> - |-+>)|you>
>>>>>>>   = |me, +,-,you> - |me,-,+,you>
>>>>>>>
>>>>>>> He says that at this point no measurements have been made, and
>>>>>>> neither observer is split. But his fundamental mistake is already 
>>>>>>> present.
>>>>>>>
>>>>>>> A little test for you: what is wrong with the above set of equations
>>>>>>> from a no-collapse pov?
>>>>>>>
>>>>>>> skipping some tedium, he then gets
>>>>>>>
>>>>>>> |psi_3> = |me[+],+,-,you[-]> - |me[-],-,+,you[+]>
>>>>>>>
>>>>>>> where the notation me[+] etc means I have measured '+', you[-] means
>>>>>>> you have measured '-'.
>>>>>>>
>>>>>>> He then claims that the QM results of perfect anticorrelation in the
>>>>>>> case of parallel polarizers has been recovered without any non-local
>>>>>>> interaction!
>>>>>>>
>>>>>>> Spoiler -- in order to write the final line for |psi_1> he has
>>>>>>> already assumed collapse, when I measure '+', you are presented *only* 
>>>>>>> with
>>>>>>> '-', so of course you get the right result -- he has built that
>>>>>>> non-locality in from the start.
>>>>>>>
>>>>>>
>>>>>> ?
>>>>>>
>>>>>> From the start shows that it is local.
>>>>>>
>>>>>
>>>>> Your failure to see the problem here is symptomatic of your complete
>>>>> failure to understand EPR in the MWI.
>>>>>
>>>>
>>>> I could say the same, but emphatic statements are not helping. My
>>>> feeling is that you interpret the singlet state above like if it prepares
>>>> Alice and Bob particles in the respective + and - states, but that is not
>>>> the case. The singlet state describe a multiverse where Alice and Bob have
>>>> all possible states, yet correlated.
>>>>
>>>
>>> The singlet state is rotationally invariant, yes, and can be expanded in
>>> any basis of the 2-d complex Hilbert space. That has never been in doubt.
>>>
>>
>> OK.
>>
>>
>>
>>> Then in absence of collapse, all interactions, and results are obtained
>>>> locally, and does not need to be correlated until they spread at low speed
>>>> up their partners.
>>>>
>>>
>>> That does not follow. Although there are an infinity of possible bases
>>> for the singlet state, these are potential only,
>>>
>>
>> I don't understand this. Potential? That is no more the MW.
>>
>>
>>
>>
>>
>> and do not exist in any operative sense until the state interacts with
>>> something that sets a direction.
>>>
>>
>> That looks more like Bohr than Everett.
>>
>>
>>
>>
>> You appear to claim that

Re: A thought on MWI and its alternative(s)

2017-06-11 Thread David Nyman
On 11 June 2017 at 10:14, Bruno Marchal <marc...@ulb.ac.be> wrote:

>
> On 09 Jun 2017, at 20:21, David Nyman wrote:
>
> On 9 June 2017 at 12:34, Bruno Marchal <marc...@ulb.ac.be> wrote:
>
>>
>> On 08 Jun 2017, at 02:05, Bruce Kellett wrote:
>>
>> On 7/06/2017 10:38 pm, Bruno Marchal wrote:
>>>
>>>> On 07 Jun 2017, at 11:42, Bruce Kellett wrote:
>>>> On 7/06/2017 7:09 pm, Bruno Marchal wrote:
>>>>
>>>>> On 06 Jun 2017, at 01:23, Bruce Kellett wrote:
>>>>>>
>>>>>> I have been through this before. I looked at Price again this morning
>>>>>>> and was frankly appalled at the stupidity of what I saw.
>>>>>>> Let me summarize briefly what he did. He has a very cumbersome
>>>>>>> notation, but I will attempt to simplify as far as is possible. I will 
>>>>>>> use
>>>>>>> '+' and '-' as spin states, rather than his 'left', 'right'.
>>>>>>>
>>>>>>> He write the initial wave function as for the case when you and I
>>>>>>> agree in advance to have aligned polarizers:
>>>>>>>
>>>>>>> |psi_1> = }me, electrons,you> = |me>(|+-> - |-+>)|you>
>>>>>>>   = |me, +,-,you> - |me,-,+,you>
>>>>>>>
>>>>>>> He says that at this point no measurements have been made, and
>>>>>>> neither observer is split. But his fundamental mistake is already 
>>>>>>> present.
>>>>>>>
>>>>>>> A little test for you: what is wrong with the above set of equations
>>>>>>> from a no-collapse pov?
>>>>>>>
>>>>>>> skipping some tedium, he then gets
>>>>>>>
>>>>>>> |psi_3> = |me[+],+,-,you[-]> - |me[-],-,+,you[+]>
>>>>>>>
>>>>>>> where the notation me[+] etc means I have measured '+', you[-] means
>>>>>>> you have measured '-'.
>>>>>>>
>>>>>>> He then claims that the QM results of perfect anticorrelation in the
>>>>>>> case of parallel polarizers has been recovered without any non-local
>>>>>>> interaction!
>>>>>>>
>>>>>>> Spoiler -- in order to write the final line for |psi_1> he has
>>>>>>> already assumed collapse, when I measure '+', you are presented *only* 
>>>>>>> with
>>>>>>> '-', so of course you get the right result -- he has built that
>>>>>>> non-locality in from the start.
>>>>>>>
>>>>>>
>>>>>> ?
>>>>>>
>>>>>> From the start shows that it is local.
>>>>>>
>>>>>
>>>>> Your failure to see the problem here is symptomatic of your complete
>>>>> failure to understand EPR in the MWI.
>>>>>
>>>>
>>>> I could say the same, but emphatic statements are not helping. My
>>>> feeling is that you interpret the singlet state above like if it prepares
>>>> Alice and Bob particles in the respective + and - states, but that is not
>>>> the case. The singlet state describe a multiverse where Alice and Bob have
>>>> all possible states, yet correlated.
>>>>
>>>
>>> The singlet state is rotationally invariant, yes, and can be expanded in
>>> any basis of the 2-d complex Hilbert space. That has never been in doubt.
>>>
>>
>> OK.
>>
>>
>>
>>> Then in absence of collapse, all interactions, and results are obtained
>>>> locally, and does not need to be correlated until they spread at low speed
>>>> up their partners.
>>>>
>>>
>>> That does not follow. Although there are an infinity of possible bases
>>> for the singlet state, these are potential only,
>>>
>>
>> I don't understand this. Potential? That is no more the MW.
>>
>>
>>
>>
>>
>> and do not exist in any operative sense until the state interacts with
>>> something that sets a direction.
>>>
>>
>> That looks more like Bohr than Everett.
>>
>>
>>
>>
>> You appear to claim that A and B exist in separate worlds corresponding
>>> to each of this infinity of bases.
>>>
>>
>> Ye

Re: A thought on MWI and its alternative(s)

2017-06-09 Thread David Nyman
On 9 June 2017 at 12:34, Bruno Marchal  wrote:

>
> On 08 Jun 2017, at 02:05, Bruce Kellett wrote:
>
> On 7/06/2017 10:38 pm, Bruno Marchal wrote:
>>
>>> On 07 Jun 2017, at 11:42, Bruce Kellett wrote:
>>> On 7/06/2017 7:09 pm, Bruno Marchal wrote:
>>>
 On 06 Jun 2017, at 01:23, Bruce Kellett wrote:
>
> I have been through this before. I looked at Price again this morning
>> and was frankly appalled at the stupidity of what I saw.
>> Let me summarize briefly what he did. He has a very cumbersome
>> notation, but I will attempt to simplify as far as is possible. I will 
>> use
>> '+' and '-' as spin states, rather than his 'left', 'right'.
>>
>> He write the initial wave function as for the case when you and I
>> agree in advance to have aligned polarizers:
>>
>> |psi_1> = }me, electrons,you> = |me>(|+-> - |-+>)|you>
>>   = |me, +,-,you> - |me,-,+,you>
>>
>> He says that at this point no measurements have been made, and
>> neither observer is split. But his fundamental mistake is already 
>> present.
>>
>> A little test for you: what is wrong with the above set of equations
>> from a no-collapse pov?
>>
>> skipping some tedium, he then gets
>>
>> |psi_3> = |me[+],+,-,you[-]> - |me[-],-,+,you[+]>
>>
>> where the notation me[+] etc means I have measured '+', you[-] means
>> you have measured '-'.
>>
>> He then claims that the QM results of perfect anticorrelation in the
>> case of parallel polarizers has been recovered without any non-local
>> interaction!
>>
>> Spoiler -- in order to write the final line for |psi_1> he has
>> already assumed collapse, when I measure '+', you are presented *only* 
>> with
>> '-', so of course you get the right result -- he has built that
>> non-locality in from the start.
>>
>
> ?
>
> From the start shows that it is local.
>

 Your failure to see the problem here is symptomatic of your complete
 failure to understand EPR in the MWI.

>>>
>>> I could say the same, but emphatic statements are not helping. My
>>> feeling is that you interpret the singlet state above like if it prepares
>>> Alice and Bob particles in the respective + and - states, but that is not
>>> the case. The singlet state describe a multiverse where Alice and Bob have
>>> all possible states, yet correlated.
>>>
>>
>> The singlet state is rotationally invariant, yes, and can be expanded in
>> any basis of the 2-d complex Hilbert space. That has never been in doubt.
>>
>
> OK.
>
>
>
>> Then in absence of collapse, all interactions, and results are obtained
>>> locally, and does not need to be correlated until they spread at low speed
>>> up their partners.
>>>
>>
>> That does not follow. Although there are an infinity of possible bases
>> for the singlet state, these are potential only,
>>
>
> I don't understand this. Potential? That is no more the MW.
>
>
>
>
>
> and do not exist in any operative sense until the state interacts with
>> something that sets a direction.
>>
>
> That looks more like Bohr than Everett.
>
>
>
>
> You appear to claim that A and B exist in separate worlds corresponding to
>> each of this infinity of bases.
>>
>
> Yes. It is the rotaional invariance of the singlet states "taken
> seriously" when we drop the idea of collapse, or of special dualism between
> observer and the observed.
>
>
>
>
> But that is a misunderstanding. They are in superpositions in every base,
>> sure, but that does not mean that there are 'worlds' corresponding to each
>> possible base until some external interaction occurs.
>>
>
> This is even more fuzzy than the collapse. It looks like consciousness not
> only reduce the wave, but create the physical reality. That is correct in
> Mechanism, but that is another story.
>
>
>
> As you yourself have said, a world is something that is closed to
>> interaction. But superpositions are not closed to interaction, they can
>> interfere -- as in the two slit experiment, and essentially every other
>> application of QM.
>>
>
> Right.
>
>
>
>> So there are no separate worlds corresponding to every possible
>> orientation of the polarizers. Worlds can arise only after interaction and
>> decoherence has progressed so that the overlap between the branches of the
>> superposition is zero (FAPP if you like). It is only then that the branches
>> can no longer interfere (interact) and are closed to interaction, and thus
>> constitute different worlds.
>>
>
> We will have to disagree with this. I use the Y=II rules, like Deutsch. In
> this case the reading of the singlet state gives 2^aleph_zero constantly
> spreading histories figuring Bob and Alice. With mechanism, those
> worlds/histories are more like dreams. They will be epistemological
> personal (and plural in the spreading interaction based spheres).
>
>
>
>
>> The standard procedure in quantum mechanics 

  1   2   3   4   5   6   7   8   9   10   >